You are on page 1of 137

Nguyn Minh Tun

Lp 11A Trng THPT Bnh Minh

Tm ti
Sng to mt s cch gii
phng trnh v t

BNH MINH - 2017


Tm ti sng to mt mt s cch gii phng trnh v t

LI NI U.
Phng trnh v t l mt trong nhng vn quan trng ca i s s cp, hin nay c rt
nhiu ti liu ni v vn ny, nhng tuy nhin trong bi vit ny ti s gii thiu ti bn c
mt vi k thut rt hay bao gm k thut gii nhng bi ton khng cn CASIO v nhng bi
ton kt hp vi mt vi k thut CASIO nh gii quyt nhng bi ton hay v kh.
Trong bi vit ny s gm 5 ch :
Mt s k thut nh trong phng trnh v t
K thut nhn lin hp, phn tch nhn t mt s phng trnh v t c bn v tm trung
K thut chng minh v nghim.
K thut s dng tnh n iu ca hm s.
K thut s dng bt ng thc.
Bi vit l nhng kinh nghim, th thut m ti tch ly c trong qu trnh hc tp. Rt cm n
cc anh ch , thy c, din n cung cp mt s bi ton bi vit ny c hon thnh:
1. Anh Bi Th Vit.
2. Anh Lm Hu Minh
3. Thy L Duy Tin
4. Din n k2pi.net.
5. Din n VMF: diendantoanhoc.net
Hu ht tt c cc bi ton trong bi vit c gii theo cch gii ca ti nn khng th trnh khi
nhng thiu st, nn mong bn c gp bi vit c hon thin hn.
Mi kin ng xin gi v tc gi.

Nguyn Minh Tun - Lp 12A THPT Bnh Minh


Email: tuangenk@gmail.com
Facebook: https://www.facebook.com/khanhhuyenhth
a ch Blog: https://kinhnghiemhoctoan.wordpress.com/

Nguyn Minh Tun Trang 1


Tm ti sng to mt mt s cch gii phng trnh v t

MC LC

A. MT VI K THUT NH. --------------------------------------------------------------------------------------- 4


I. KIM TRA NGHIM BI. .....................................................................................................4
a) Kim tra bng o hm......................................................................................................4
b) Kim tra bng gii hn hm s. ........................................................................................4
Mt s mo khc. ....................................................................................................................5
II. TM NHN T. .....................................................................................................................6
Cch tm nhn t cha nghim hu t n duy nht. .......................................................6
Cch tm nhn t cha nghim hu t kp. ........................................................................6
Cch tm nhn t cha nghim bi cao................................................................................7
Cch tm nhn t cha nghim v t. ..................................................................................7
Cch tm nhn t cha nghim hu t v 1 nghim v t ................................................8
III. K THUT PHN TCH TNG BNH PHNG. .......................................................9
IV. K THUT HON I NHN T ...............................................................................11
V. K THUT N PH KHNG HON TON ..............................................................15
VI. K THUT CHIA CN. ...................................................................................................18
1. Cng thc chia 1 cn. .......................................................................................................18
2. Cng thc chia 2 cn. .......................................................................................................18
3. Cng thc chia 3 cn. .......................................................................................................20
VII. MT K THUT NH NH GI PHNG TRNH NGHIM KP. ................23
B. K THUT NHN LIN HP, PHN TCH NHN T MT S PHNG TRNH
V T C BN V TM TRUNG. --------------------------------------------------------------------------------- 26
I. BI. ...................................................................................................................................26
II. HNG DN GII.............................................................................................................28
C. K THUT CHNG MINH V NGHIM.------------------------------------------------------------------ 43
I. PHNG TRNH A THC. ............................................................................................43
Phng trnh bc 4. ...............................................................................................................43
Phng trnh bc 6. ...............................................................................................................45
Phng trnh bc chn khng cht. ....................................................................................46
Chng minh trn khong. ...................................................................................................48
Chng minh trn on. ........................................................................................................50
II. PHNG TRNH V T...................................................................................................55
1. bi..................................................................................................................................55
2. HNG DN GII. ........................................................................................................57
D. K THUT S DNG TNH N IU. -------------------------------------------------------------------- 78

Nguyn Minh Tun Trang 2


Tm ti sng to mt mt s cch gii phng trnh v t
I. Kin thc cn nh. .................................................................................................................78
nh l 1. ................................................................................................................................78
nh l 2. ................................................................................................................................78
nh l 3. ................................................................................................................................78
II. Bi ton minh ha. ...............................................................................................................79
E. K THUT S DNG BT NG THC -------------------------------------------------------------------- 97
I. CC BT NG THC C BN CN NH. ................................................................97
Bt ng thc Cauchy AM GM . ...................................................................................97
Bt ng thc Bunhiacopxki Cauchy Schwarz (BCS). ...............................................97
Bt ng thc Minkowski. ...................................................................................................97
Bt ng thc Holder. ..........................................................................................................97
II. BI. ..................................................................................................................................98
III. CC BI TON. ..............................................................................................................101

Nguyn Minh Tun Trang 3


Tm ti sng to mt mt s cch gii phng trnh v t
A. MT VI K THUT NH.
I. KIM TRA NGHIM BI.
Xt phng trnh f x 0 ta c th phn tch phng trnh thnh x x 0 k .g x 0 .
Nu k 1 khi phng trnh c nghim n.
Nu k 1 khi phng trnh c nghim bi k.
Sau y l cc cch kim tra 1 nghim xem c phi l nghim bi hay khng ca phng trnh.
a) Kim tra bng o hm.
f x 0
0
f ' x0 0

Xt phng trnh f x 0 c nghim x x 0 v nghim bi n khi v ch khi: .......... . Khi
n 1
f x0 0
f n x 0
0
phng trnh f x 0 c nghim bi n .
V d : Kim tra nghim bi x 1 ca phng trnh: x 4 4x 3 6x 2 4x 1 0
Ta c:
f ' x 4x3 12x2 12x 4 0
x1
2
f '' x 12x 24x 12 0
x 1

f ''' x 24x 24 x 1 0
f '''' x 24 0
Vy phng trnh c nghim bi 4 x 1 .
Nhn xt: Cch kim tra nghim bi bng o hm ny kh nhanh vi cc phng trnh a thc.
Nhng tuy nhin nu gp phi phng trnh v t c 2 cn tr ln m nghim bi cao v d nh
bi 5 th c m tnh o hm bng tay cht lun ri ch ng c ni l s quyt tm lm tip
. Do ta s s dng cch 2.
b) Kim tra bng gii hn hm s.
k
Xt phng trnh f x 0 ta c th phn tch phng trnh thnh x x 0 .g x 0 c nghim
bi k x x 0 .

f x
lim k 1
0

x x0
x x 0
Khi nu th phng trnh c nghim bi k .
lim f x 0
x x0 x x k
0

f i x 0 0i 0, k 1
V mt o hm th ta c i
f x 0 0i k
V d: Kim tra nghim bi x 0 ca phng trnh: x 2 x 1 x 2 1 x x 5 4x 0
Ta c:

Nguyn Minh Tun Trang 4


Tm ti sng to mt mt s cch gii phng trnh v t
x 2 x 1 x 2 1 x x 5 4x
lim 0
x 0 x
x 2 x 1 x 2 1 x x 5 4x
lim 0
x 0 x2
x 2 x 1 x 2 1 x x 5 4x
lim 0
x 0 x3
x 2 x 1 x 2 1 x x 5 4x
lim 0
x 0 x4
x 2 x 1 x 2 1 x x 5 4x
lim 0
x 0 x5
Li gii l:
x 2 x 1 x 2 1 x x5 4x 0
1 2

4
x1 1x 2 x1 1x
x 6x 8x 16
3 2

x 1 x 3 6x 2 8x 16 1 x 4 4 x 2 1 x 2 18 12x 2 0
Vic tnh ci lim cui cng bng my tnh c kt qu chnh xc th rt l kh. Nhng tuy
nhin y chng ta ch cn kim tra nghim bi ca phng trnh nn cho nn ch cn ci cui
khc 0 l c, khng cn quan tm n iu khc. Cn mt iu na, khi tnh lim ca bi ny
cc bn nn CALC X 0, 01 nu khng th kt qu s ra ton bng 0 lm cc bn rt kh suy
on. Vic tnh lim ny chng ta nn CALC nhiu gi tr khc nhau c th nh gi ng c
tnh cht ca nghim.
Mt s mo khc.
Do trong thi i hc th ti a c th cho nghim kp l ht c cho nn ta c th kim
tra nghim kp bng cch sau:
+ Dng MODE 7 ta c th thy hm khng i du khi i qua mc 0 cho nn s l
nghim kp.
d
+ Dng t hp phm SHIFT tc l tnh o hm. Tnh
dx
f x 0 th l
x x0

nghim kp.
Kim tra nghim bi 3:
Nu dng MODE 7 m thy hm i du khi qua mc 0, m khi thc hin php tnh
d
dx
f x 0 th chng t l nghim bi 3.
x x0

Ni chung ch c vy ai cng c th pht hin ra trong qu trnh lm bi!

Nguyn Minh Tun Trang 5


Tm ti sng to mt mt s cch gii phng trnh v t
II. TM NHN T.
y l iu rt quan trng trong vic gii phng trnh v t, lm tt bc ny th mi c th
chuyn sang cc bc sau c. Sau y ti s a ra cho cc bn cc loi nhn t c th gp
trong khi gii, v mt s khng bao gi c trong thi hc sinh gii hay THPT Quc Gia m ch
mang tnh cht tham kho. Ngoi ra ti nhc cc bn l phi nm chc cch tm cc loi nhn t
hay gp trong thi c th x l tt khi gp phi.
Cch tm nhn t cha nghim hu t n duy nht.
i vi phng trnh 1 cn:khi gp phi loi ny th cc bn thay trc tip nghim x x 0
vo cn thc.
+ Nu kt qu l nguyn th ta c lun nhn t cha nghim n .
+ Nu kt qu l v t th c th y l dng phi dng n ph khng hon ton gii quyt
n hoc dng cng thc chia 2 cn gii quyt, ni chung l cn rt nhiu cch.
i vi phng trnh 2 cn th lc ny nhn t c dng ti u nht l f x a g x b
. Cng tng t nh 1 cn, phng trnh 2 cn cng c khi cc h s l v t, v nhng
trng hp cc bn u phi suy on mt loi nhn t khc cha nghim ny hoc y
khng phi l bi cho c th phn tch thnh nhn t c, hoc nu thay kt qu vo cc
cn m thy n bng nhau th c th d dng suy ra nhn t. tm nhn t ta s cho a ty
tm ra b sao cho s a l mt s va nh, ti u nht. Vi cch lm nh trn th c th
c nhiu nhn t c sinh ra, kim tra nhn t no ti u hn th chng ta nn chn s
a ch l 1,-1 thi hoc ta s ly biu thc u chia cho nhn t ti ang suy on ri CALC
nhng gi tr b chy t -3 n 3, nu kt qu l nhng s hin th dng cn th l nhn t
p. Ch khi chn nhn t nh vy s rt c th c nghim ngoi lai lm chia cn b l.
Nu gp trng hp nh th th hy tm nghim ngoi lai trc ( c th bnh phng nhn
t tm nghim) ri nhn ci nhn t vo phng trnh u. Nu th ht mi cch m
khng c th chuyn sang lin hp chng minh v nghim ( c phn chng minh v
nghim th s thy thch cch ny J).
Cch tm nhn t cha nghim hu t kp.
Phng trnh v t 1 cn.
Nhn t 1 cn thc cha nghim kp c dng tng qut nh sau: f x ax b . Do cha nghim
d
f x ax b 0


a dx f x x x0
kp x x 0 nn c h phng trnh sau:

f x ax b ' 0 b f x d
f x
dx x x0

T ta c th suy ra nhn t ca bi ton.


Phng trnh v t 2 cn.
Cng tng t nh trn, nhn t cha nghim bi kp c dng f x a g x b . Do y l
nghim hu t kp nn o hm ca n cng cha nghim kp x x 0 nhng trong phng trnh
f ' x 0 th x x 0 ch l nghim n, c h sau:

Nguyn Minh Tun Trang 6


Tm ti sng to mt mt s cch gii phng trnh v t
d
dx
f xx x0
a
d
f x a g x b 0


dx
g x x x0
T ta c th suy ra

f x a g x b ' 0

d

dx f x

x x0
b f x g x
d



dx g x
x x0

nhn t ca bi ton.
Cch tm nhn t cha nghim bi cao.
Phng trnh v t 1 cn.
Xt phng trnh f x 0 c nghim x x 0 v nghim bi n n 2 . Khi tm nhn t
f x 0

f ' x 0
cha nghim bi n x x 0 ta lm nh sau: Gii h phng trnh: . Khi nhn t c
..........
f n 1 x 0

dng f x a.x n 1 b.x n 2 c.x n 3 ... dx e . V sau khi gii h ta tm c cc h s ca
nhn t.
Ngoi ra cn ch i vi nhn t nghim bi cao hn nh bc 4 hoc bc 5 th khng nn tm
nhn t theo nh trn m u tin nn kim tra xem n c th tch thnh nhn t nghim kp bnh
phng hay khng
( i vi nghim bi 4) hay c th tch thnh nhn t nghim bi 4 nhn vi nhn t cha nghim
n hay khng ( i vi nghim bi 5 ) gim bt cng knh ca nhn t.
Phng trnh v t 2 cn.
Thng thng khi gp phng trnh v t 2 cn hay nhiu cn cha nghim bi 2 tr ln th cch
lm m nhiu ngi s dng l tch cn sau tm nhn t lin hp tng cn ri nhn lin hp
sau chng minh phng trnh cn li v nghim. Tuy nhin chng ta vn c th tm nhn t 2
cn cha nghim bi nh sau.
+ Nu phng trnh c nghim bi 3 ( hoc c 3 nghim hu t), lc ny nhn t c dng nh
sau: f x a g x bx c . tm cc h s a,b,c ta gii h

f x a g x bx c 0


f x a g x bx c ' 0 Nhn t


f x a g x bx c '' 0

+ Nu phng trnh c nghim bi 4 th ta s tm nhn t cha nghim bi kp sau bnh


phng ln thnh nhn t cha nghim bi 4.
+ Nu phng trnh c nghim bi 5 th ta kim tra xem n c th tch thnh nhn t nghim bi
4 nhn vi nhn t cha nghim n hay khng.
+ Ngoi ra nu nghim bi cao hn nh th th ta vn t duy theo hng nh trn tm nhn t.
V nn lm theo phng php nhn lin hp.
Cch tm nhn t cha nghim v t.
Cch tm nhn t cha nghim v t dng a b c
y l dng nghim ca phng trnh bc 2 nn ta c cch tm nhn t nh sau:

Nguyn Minh Tun Trang 7


Tm ti sng to mt mt s cch gii phng trnh v t
+ i vi 1 cn th nhn t c dng f x ax b . Khi dng MODE 7 vi hm
f A XA vi A l nghim v t ca phng trnh u. Ta cho cc d kin my hi theo nh
nh ca ti min sao tm c nhn t l c ( tc l tm c gi tr X lm F X hu t).
Nu khng tm c nhn t khi cho 1 th ta tip tc nng h s ca ln cho ti khi no
tm c nhn t. Thng thng khi gii phng trnh m tm c nghim v t th u tin ta
nn ngh n trng hp ny v trong thi i hc hay mt s thi th th hu ht l cho
nghim dng ny. tm dng nghim chun xc a b c th ta dng MODE 7 vi hm
f X A 2 XA sau cng tm tng t nh trn ta c phng trnh bc 2 cha nghim v
t va tm c.
+ i vi phng trnh v t 2 cn ta th nhn t c dng nh sau f x a g x b sau
dng MODE 7 vi hm F X f A X g A sau cng tm tng t nh trn th ta
c nhn t cha nghim v t.
Nu trong bi thi ta gp trng hp nghim v t kp th vn lm tng t nh trn, ch khc l
sau khi tm nhn t cha nghim v t n th phi bnh phng nhn t ln.
Cch tm nhn t cha nghim hu t v 1 nghim v t
Ngoi nhng dng nhn t ti ni trn th dng ny cng l 1 trong nhng dng hay gp trong
thi. Tuy nhin cch lm tng qut m ngi ra mun nhm ti l chng ta s li c 1
trong hai nghim ra ( thng thng l nghim v t trc ) sau chng ta s phi dng hm
s kho st ch ra nghim . Mc d vy chng ta vn c th li c 2 nghim cng mt
lc m nhiu ngi cho rng khng th, c th nh sau:
Ta xt phng trnh tng qut f x 0 c th phn tch ra thnh x x 0 ax 2 bx c .g x 0
trong g x lun v nghim v ax 2 bx c l mt phng trnh bc 2 cha 1 nghim v t
ca phng trnh u. Khi tm nhn t cha 2 nghim ny( ch p dng cho 1 cn) ta s lm
theo nhng bc sau:
Bc 1: Ta s tm nhn t bc 2 cha nghim l X A bng MODE 7 vi hm
f X A 2 XA .
Bc 2: Ta s tm nhn t cn cha nghim l X A bng MODE 7 vi hm
f X n f A XA .
Bc 3: Khi nhn t c dng: n f A ax b k cx 2 dx e 0
n f x 0 ax 0 b
Ta s thay nghim hu t vo v tm ra c k, khi k
cx 0 2 dx 0 e
Vy l ta tm xong nhn t cha 1 nghim v t bc 2 v 1 nghim hu t n.

Nguyn Minh Tun Trang 8


Tm ti sng to mt mt s cch gii phng trnh v t
III. K THUT PHN TCH TNG BNH PHNG.

Vd 1: Gii phng trnh: 3x3 2x2 2 3x3 x 2 2x 1 2x2 2x 2

thi th THPT Quc Gia 2016 ln 1 THPT Chuyn H S phm H Ni

Li gii ngn gn ca bi ny nh sau:


3x 3 2x 2 2 3x 3 x 2 2x 1 2x 2 2x 2
2 2
1 1 1

2
3x 3 2x 2 2 1 2
3x 3 x 2 2x 1 1 2
2
x 1 0
3x 3 2x 2 2 1 0


D thy rng VT 0 nn du " " ch xy ra khi 3x 3 x 2 2x 1 1 0 x 1
x 1 0

Vy x 1 l nghim duy nht ca phng trnh.
Th no, sau khi c xong c thy di hn cch lm Cauchy khng? Thc ra bn cht ca
cch ny cng chnh l dng bt ng thc, tn ting Anh l Sum of square hay ta gi l
SOS.
u tin lm theo cch ny ta s lm theo cc bc sau:( ch rng ang p dng cho
a s nhng bi cc cn ang ng n l,a thc trong cc cn cng bc v c nghim
kp)
1. Tm nghim ca phng trnh
2. Tm nhn t cha nghim n cho tng cn
3. Xc nh du ca phng trnh u bng CASIO
4. Khi phn tch phng trnhthnh:
2 2 2
f x a
f1 x b ... fn x z 0

Vi , ,..., cng du, f x a; f1 x b l nhng nhn t cha nghim n va


tm c
p dng vo bi
3x 3 2x 2 2 1
1. Ta c khi x 1 th
3 2
3x x 2x 1 1
2. Nhn thy rng VT 0 nn ta s tch phng trnh thnh:

3x 3 2x 2 2 3x 3 x 2 2x 1 2x 2 2x 2
2 2
1 1 1

2

3x 3 2x 2 2 1
2
2
3x 3 x 2 2x 1 1 x 1 0
2

Vd 2: Giiphngtrnh: 2 x2 2x 1 2 x2 2x 2 3 x2 2x 3 0

p dng cch lm trn ta c:


2 x 2 2x 1 2 x 2 2x 2 3 x 2 2x 3 0
2 2 2
1 3
x 2 2x 1 1 2
x 2 2x 2 2
2
x 2 2x 3 1 0
n l xong ri .

Nguyn Minh Tun Trang 9


Tm ti sng to mt mt s cch gii phng trnh v t
Vd 3: Chng minh rng:

x 4 x 1 32 4 x 3 4x 2 7x 12
f x 1 0x 3;
x 4 x 2 16x 11
Trch t cun Nhng vin kim cng trong bt ng thc ton hc Trn Phng

Cu ny th chc chn phi lm theo AM GM nhng c th dng SOS gii


2 2 2


4

4 2
1 2 2 x 3 x x 4 32 x 3 1 2
Ta c: f x
x2 x 4 4 0
2 x 4 x 2 16x 11
2 4 x 3 4 x 2 x 4 0

Do du " " xy ra x 3 1 0 x3
2
x x 4 4 0
Vy bi ton c gii quyt!

Bi tp tng t.
1. x 2 4 x x 2 6x 11
1 2 1 2
/s:
2

x2 1
2
2
4 x 1 x 3 0
2. 2x 1 2 x 3 7 2x 2 12 5 x 2x 41
1 2 1 2 7 2 2
/s:
2

2x 1 1
2

x3 2
4
2x 2 2 3 5x 2 0

3. x 4 4x 3 6x 2 3x 3 2x 2 4 2x 1
2
1 2 4
2x 1 1 2x 1 2 4 2x 1 3 0
/s: x 1
2

4
2x 2 2
4

4 2 3 3
4. x 3x 2 x 4 4 x 1 2 x 4x
x
2 2 1

/s: x x 3 4 x 2 x 1
x
x 3 4x 2x 2x 3
x 3 4x 2x 2 6x 0

5. x2 12x 1 4 x3 x 2x 4x 1 2 4x x2
2
2 2 x 2
4x 1
/s:
4x x 2 1 x 4x 1
x 2 4x 1 4 x 3 x
0

Nguyn Minh Tun Trang 10


Tm ti sng to mt mt s cch gii phng trnh v t
IV. K THUT HON I NHN T
K thut hon i nhn t l k thut nhm tm nghim ca phng trnh i du trc cn sau
suy ra nhn t ca phng trnh ban u. C l nhiu bn s khng hiu v sao sau khi i du ta
tm c nhn t v nghim, rt n gin php i du trc cn l mt php bin i ging nh
php bnh phng, ti s chng minh cho cc bn thy bng mt v d sau y.
V d 1 : Gii phng trnh: 10x 60 x 1 x 2 37x 60 0
Theo nh nhng cch lm bn trn th u tin ta s tm nghim sau s tm nhn t ng
khng? Nhng tuy nhin vi bi ny ta s khng lm c g do n v nghim m. Tuy nhin
theo nhng cch cc thy c gio dy th ta c th khng cn quan tm ti nghim ca phng
trnh m c vic bnh phng ln bc 4 sau gii bnh thng do ti s bnh phng phng
trnh u ln, ta c:
10x 60 x 1 x2 37x 60 0
10x 60 x 1 x 2 37x 60
x 1 x 1

x 2 37x 60 10x 60 0 x 2 37x 60 10x 60 0 PTVN
4
x x 3 x 8 x 15 0
3 2
x 26x 189x 360x 0
Bn cht ca vic bnh phng l vic gii phng trnh f x 0 vi f x l biu thc lin hp
ca v tri, c ngha l f x 10x 60 x 1 x 2 37x 60 cho nn khi gii ra phng
trnh bc 4 c bng kia nghim th tc l phng trnh f x 0 s vit li thnh:
f x 10x 60 x 1 x 2 37x 60 0
x11 x1 2 x1 3 x14 0
Mt khc do f x l biu thc lin hp ca v tri cho nn tt c nhn t phi i thnh biu thc
lin hp ca chnh cc nhn t v tr thnh:
x1 1 x1 2 x1 3
x1 4 0
Vy n y ta phn tch phng trnh u thnh cc nhn t v nghim ri .
Tuy nhin thc cht khng phi bi no cng lm c nh vy, c nhng bi khi bnh phng
cn 1 phng trnh bc 3 c nghim l th sao, chng l li dng cch tm nhn t cha nghim
ca phng trnh bc 3 tm ? Thi tt nht n ch m khng tm c nhn t ca phng
trnh i du th i chng minh v nghim cn hn . cn mt iu na, khi i du chng ta
cn c th tm c nghim lin hp tm nhn t m khng cn phi dng MODE 7, c th ta
c v d sau

V d 2: Gii phng trnh: x 8 x 2 x 36 x 2 7 x 2 4 2x 63 0


on Tr Dng
Gii
Bc 1: Tm nghim, SOLVE ta c nhng nghim nh sau:

Ta s gn 2 nghim l chot kia vo A,B. ng nh cng thc ta s i du trc cn. Nhng


tuy nhin y c 2 cn nn ta s i du tng cn mt.
Bc 2: Tm nghim i du:
+ i du x 2 ta tm c nghim sau:
Tm ti sng to mt mt s cch gii phng trnh v t

+ i du x 2 ta tm c nghim sau:

+ i du 2 cn ta c phng trnh v nghim.


289 190
AD 32, 1 9 B C 9
Bc 3: Ta nhn thy rng ;
A D 16, 6 50 BC 425
3 9
Cho nn ta c cc nghim i du l nghim lin hp ca phng trnh u.Khi nhn t
c dng x 2 a x 2 b cha ln lt cc nghim trn.
+ Vi 2 nghim B,C ta c h phng trnh:
B 2 a B 2 b 0




a B 2 C 2 B 2 C 2 0

a 2
C 2 a C 2 b 0 b B 2 a B 2 b 5
NT : x 2 2 x 2 5
+ Vi 2 nghim A, D ta c h phng trnh:
1
a
A 2 a A 2 b 0


a A 2 D 2 D 2 A 2

2
D 2 a D 2 b 0 b A 2 a A 2 b 3
2
NT : 2 x 2 x 2 3
S d ta c h phng trnh nh trn l do ta i du cn no khi tm nghim th khi lp h, phn
h s trc cn phi i li du, cc bn thy ch? Qua v d ny cc bn li c thm 1 cch
tm nhn t cha nghim v t bc 2 na ri, nhng tuy nhin ti khuyn cc bn nn dng
cch c bi c khi i du ta s khng tm c nghim hoc nghim l th cch ny ht ngon n!
Khi phng trnh u s c phn tch thnh:
x 8 x 2 x 36 x 2 7 x 2 4 2x 63 0


2 x2 x2 3 x2 2 x2 5
x2 x2 1 0

V d 3: Gii phng trnh: x 6 13x 2 6x 16 2x x 4 x 3 10 x 2 0


Bi ny c th thy bc kh cao nn ta c th tm nghim i du tm thm nhn t.
Bc 1: Tm nghim + Nghim i du ta c:
Tm ti sng to mt mt s cch gii phng trnh v t
C th nhn thy x 2 l nghim ca phng trnh u cn x 1 l nghim ca phng
trnh i du.
Bc 2: Kim tra nghim bi. Ta c:
x 6 13x 2 6x 16 2x x 4 x 3 10 x 2
lim 0
x 2 x2
x 6 13x 2 6x 16 2x x 4 x 3 10 x 2
lim 2
0
x 2
x 2
x 6 13x 2 6x 16 2x x 4 x 3 10 x 2
lim 0
x 1 x1
x 13x 6x 16 2x x 4 x 3 10 x 2
6 2

lim 2
0
x 1
x 1
Vy c 2 nghim u l 2 nghim kp ca 2 phng trnh tng ng.
Bc 3: Tm nhn t.
+ Nhn t cha nghim kp x 2 ca phng trnh u l 4 x 2 x 6
+ Nhn t cha nghim kp ca phng trnh i du l 2 x 2 x 3 Nhn t ca phng
trnh u l 2 x 2 x 3 .
Bc 4: Chia cn ta c kt qu l:
x 6 13x 2 6x 16 2x x 4 x 3 10 x 2
2x x 2 x 4 x 3 x 2 x 8

2 x2 x3 4 x2 x6
Bng php th ta c th thy cc kia lun m, mt khc ta thy c dng 2ab nn s th thm bt
2
to hng ng thc a b ( do ang cn chng minh m) ta c:
2x x 2 x 4 x 3 x 2 x 8
2

x x2 x 4 x 3 2x 2 2x 6
2 2
2 1
2 9 27 38 58

x x 2 x x x
2 5 2 27 675
0

Vy bi ton c gii quyt!
2x 5 3x 4 14x 3 2
V d 4: Gii phng trnh: 4x 4 14x 3 3x 2 2 1
x2 x2
thi th ln 2 THPT L Thi T - Bc Ninh 2016
Gii
nh th ny th hi kh lm do ta quy ng ht ln, ta c phng trnh mi:

2x 5 3x 4 14x 3 4x 4 14x 3 3x 2 2 x 2 2
n y chc chn phi i tm nghim ca phng trnh SOLVE ta tm c 2 nghim l:

Ngoi 2 nghim nh trn ta cn tm c 1 nghim i du l x 1 .


Tm nhn t cho phng trnh ta c 3 nhn t l

x2 2 ; x2 1 ; x2 x1 .
n y chia cn ta c:
Tm ti sng to mt mt s cch gii phng trnh v t
2x 5 3x 4 14x 3 2
4x 4 14x 3 3x 2 2 1
x2 x2


x2 2 x2 1
x 2 x 1 2x 3 7x 2 2 2x x 2 0
x2
x2 2 0 x 2

1 5
x2 x1 0 x
2

x 2 1 0 PTVN 0
3 2
2x 7x 2 2x x 2 0 *
Gii phng trnh * :
2x 3 7x 2 2 2x x 2 0
2
x 2 4x 2 x 3 x 4 x 2 16 4 0 PTVN 0

Nguyn Minh Tun Trang 14


Tm ti sng to mt mt s cch gii phng trnh v t
V. K THUT N PH KHNG HON TON
Ta xt phng trnh tng qut: g x f x u x 0
Khi ta s t f x t v bin i phng trnh thnh:
t 2 g x t u x .f x 0
Cng vic cn lm by gi l tm h s c th biu din f x t qua x . lm
vic ny cn ti s tr gip ca CASIO .
Sau y l cc bc lm tng qut:
2
Bc 1: Ta c bit thc g x 4 u x .f x
Bc 2: Ta s cn tm sao cho l mt s hu t. Vi l do ta s dng
2
MODE 7 vi hm: F X g 1000 4X u 1000 X.f 1000 . Nn
Start 15

nh gn 1000 A trc khi lm. Ta s cho Step 14 . V tm cc gi tr lm f x
End 1

nguyn.
Bc 3: Theo nh l thuyt th ta s lun tm c mt s sao cho f x t lun biu
g x
t f x
din c qua bin x . V khi ta s c 2
g x
t f x
2
Sau khi n y ta s phn tch c phng trnh u thnh nhn t!

V d 1: Gii phng trnh:


2x 4 x3 2x2 x 2 x 1 x8 x7 2x6 2x5 4x 4 2x3 2x2 2x 2 0
Gii

ng nh kch bn ta s vit phng trnh v dng t 2 g x .t u x .f x 0 .


g x 2x 4 x 3 2x 2 x 2

Vi u x x 8 x7 2x6 2x 5 4x 4 2x 3 2x 2 2x 2

f x x 1
Ta s gn 1000 A , sau gim bt s cng knh khi vo MODE 7 th ta s tnh
g A B

u A C
f A D

MODE 7 vi hm:
F X B 2 4X C DX
Start 15

- Vi Step 14
End 1

- Sau khi tin hnh d tm ta s c kt qu l
Tm ti sng to mt mt s cch gii phng trnh v t
X 1
3
F X 999999000 x x

2x 4 x 3 2x 2 x 2 x 3 x
t1 x 4 x 3 x 2 1
Khi ta s c: 2
2x x 2x x 2 x 3 x
4 3 2
t
2 x 4 x2 x 1
2
Vy phng trnh u s c phn tch thnh:
x 1 x 4 x3 x2 1
x 1 x4 x2 x 1 0
Cn cng vic chng minh phng trnh v nghim th qu d cn g ni na ! T
chng minh nh!
Tng t m rng hn vi h phng trnh ta cng c th lm nh trn

1 y x 2 2y 2 x 2y 3xy

V d 2: Gii h phng trnh:
2 2
y 1 x 2y 2y x
thi th THPT Quc Gia - Chuyn Hng Yn 2015
Gii
Xut pht t phng trnh th nht thy c dng gn ging vi dng tng qut ca
phng trnh 1 cn, nn ta cng c th bin i phng trnh v thnh:
t 2 1 y t x 2y 3xy x 2 2y 2 0

Trong t x 2 2y 2 & l h s cn tm.


Ta s gn 100 B, 1000 A vi B l y cn A l x. Ta s c biu thc sau:

f X 1 B
2

4X A 2B 3AB X A 2 2B 2
Sau khi d bng MODE 7 ta s tm c X 1; F X 2301 .
2 2 1 y 2x 3y 1
x 2y 2
x y 1
Vy tm li
x 2 2y 2 1 y 2x 3y 1 x 2y
2
Khi li gii l:
Ta c:
PT 1 x 2 2y 2 x y 1
x 2 2y 2 x 2y 0

x 2 2y 2 x y 1 0 PTVN
o

x 2y x 2y
2 2

Tm ti sng to mt mt s cch gii phng trnh v t
1 5
y 1 2x x
Kt hp vi phng trnh 2 ta c: 4
2 2
x 2y x 2y y 1 5
2

Nguyn Minh Tun Trang 17


Tm ti sng to mt mt s cch gii phng trnh v t
VI. K THUT CHIA CN.
1. Cng thc chia 1 cn.
g x f x h x
Gi s c php chia S g 1 x f x h 1 x . Khi tm cc h s ta
a f x bx c
c cc bc sau.
Gn 1000 X .
g x f x h x
Nhp biu thc v thay X 1000 vo ta c A
a f x bx c
g x f x h x
i du trc cn: B.
a f x bx c
AB
g 1 x
2 f x
Khi .
AB
h 1 x 2

9x 4 12x 3 7x 2 6x 16 4 x 3 x 2 x 2
V d: Thc hin php chia sau: S
2
x 2 x 2 x 4 4 x 2 x 2 3x 5
Gn 1000 X .
Nhp biu thc v thay X 1000 vo ta c:

9x 4 12x 3 7x 2 6x 16 4 x 3 x 2 x 2
A

2 x 2 x 2 x 4 4 x 2 x 2 3x 5 xX

9x 4 12x 3 7x 2 6x 16 4 x 3 x 2 x 2
i du trc cn: B

2 x 2 x 2 x 4 4 x 2 x 2 3x 5 xX
2
Khi thng c dng f x x x 2 g x .
AB 6x 6
f x 5994
7
Vi 2 x 2 x 27
2
g x A B 2143287.143 15x 3x 10

2 7
Nhn xt: thy khi tnh g x ta c kt qu lth lm sao bit c mu l 7. Cu tr li
n gin thi, v khi tm h s trc cn ta tm c mt phn thc c mu l 7 m nhng h
s ban u u nguyn nn chng c ngha l no h s cn tm cn li cng nguyn c, n phi
c mu chung vi ci va tm c.
Ngoi ra nu gp phi nhng bi khi thay X 1000 vo m vi phm KX th chuyn sang
MODE 2 CMPLX ri tnh nh bnh thng.

2. Cng thc chia 2 cn.


Ta x tphp chia tng qut sau:

Nguyn Minh Tun Trang 18


Tm ti sng to mt mt s cch gii phng trnh v t
g x u x v x h x f x t x v v f x
P
d x v x q x f x p x
u1 x v x h 1 x f x t1 x v v f x g 1 x
Khi tm c cc h s trc cn ta lm nh sau:
Cha i du, CALC X 1000 ri gn vo A.
g x u x v x h x f x t x v v f x
A
d x v x q x f x p x
i du v x , CALC X 1000 ri gn vo B
g x u x v x h x f x t x v v f x
B
d x v x q x f x p x
i du f x , CALC X 1000 ri gn vo C
g x u x v x h x f x t x v v f x
C
d x v x q x f x p x
i du 2 cn, CALC X 1000 ri gn vo D
g x u x v x h x f x t x v v f x
D
d x v x q x f x p x
Khi thng ca php chia c dng nh sau:
P u1 x v x h 1 x f x t 1 x v v f x g 1 x
ABCD
u 1 x
4 v x
ABCD
h 1 x
4 f x
Vi:
ABCD
t 1 x
4 v x f x
ABCD
g 1 x
4

V tng t nh phn chia a thc 1 cn i khi gp phi nhng trng hp vi phm KX ca


bi ton ,khi ta s chuyn sang ch s phc MODE 2 CMPLX chia nh bnh thng
v p dng cng thc l xong

V d : Thc hin php chia sau: S



3x 2 1 x 1 x 2 2 3x x 2
2
1 x 2 3x 1
Chuyn sang mi trng s phc MODE 2

Ban u:

3x 2 1 x 1 x 2 2 3x x 2 A
2
1 x 2 3x 1 X 1000

Nguyn Minh Tun Trang 19


Tm ti sng to mt mt s cch gii phng trnh v t

i du 1 x :

3x 2 1 x 1 x 2 2 3x x 2 B
2
1 x 2 3x 1 X 1000

3x 1 x 1 x 2 2 3x x 2
2

i du 2 3x : 2
C
1 x 2 3x 1 X 1000

3x 1 x 1 x 2 2 3x x 2
2

i du 2 cn: 2
D
1 x 2 3x 1 X 1000

Thng c dng: f x 1 x g x 2 3x h x 3x 2 5x 2 v x
ABCD
f x 4 1x
1

g x A B C D 1
4 2 3x
Vi
h x A BCD 1

4 3x 2 5x 2 2

v x A B C D 2 3x
4 2

3. Cng thc chia 3 cn.


Xt php chia sau:
u v t u v u t v t u v t k x
f x
m u n v p t q x
a u b v c t d u v e u t f v t g u v t h
t:
u v t u v u t v t u v t k x
1. A
m u n v p t q x X 1000

2. i du trc u :
u v t u v u t v t u v t k x
B
m u n v p t q x X 1000

3. i du trc v :
u v t u v u t v t u v t k x
C
m u n v p t q x X 1000

4. i du trc t :
u v t u v u t v t u v t k x
D
m u n v p t q x X 1000

5. i du u & v :
u v t u v u t v t u v t k x
E
m u n v p t q x X 1000

Nguyn Minh Tun Trang 20


Tm ti sng to mt mt s cch gii phng trnh v t
6. i du u & t :
u v t u v u t v t u v t k x
F
m u n v p t q x X 1000

7. i du v & t :
u v t u v u t v t u v t k x
M
m u n v p t q x X 1000

8. i du 3 cn:
u v t u v u t v t u v t k x
Y
m u n v p t q x X 1000

ABCDEFMY ABCDEFMY
a 8 u e
8 u t

b A B C D E F M Y A B C D E F M Y
8 v f 8 v t
Khi & .
c A B C D E F M Y g A B C D E F M Y
8 t 8 u v t
A B C D E F M Y
d h A B C D E F M Y
8 u v
8

V d: Thc hin php chia: S


5x 4 3x 2 5 2 x 6x 1 x 3
3x 2 2 x x 3
Chuyn sang MODE 2 CMPLX

Ban u :
5x 4 3x 2 5 2 x 6x 1 x 3
A
3x 2 2 x x 3 x 1000

5x 4 3x 2 5 2 x 6x 1 x 3
i du trc 3x 2 : B
3x 2 2 x x 3 x 1000

i du trc 2 x :
5x 4 3x 2 5 2 x 6x 1 x 3
C
3x 2 2 x x 3 x 1000

i du trc x 3 :
5x 4 3x 2 5 2 x 6x 1 x 3
D
3x 2 2 x x 3 x 1000

5x 4 3x 2 5 2 x 6x 1 x 3
i 3x 2 ; 2 x : E
3x 2 2 x x 3 x 1000

5x 4 3x 2 5 2 x 6x 1 x 3
i 3x 2 ; x 3 : F
3x 2 2 x x 3 x 1000

i 2 x; x 3 :
5x 4 3x 2 5 2 x 6x 1 x 3
M
3x 2 2 x x 3 x 1000

5x 4 3x 2 5 2 x 6x 1 x 3
i 3 cn: Y
3x 2 2 x x 3 x 1000

Nguyn Minh Tun Trang 21


Tm ti sng to mt mt s cch gii phng trnh v t
Khi
ABCDEFMY ABCDEFM Y
a 8 3x 2
0 e 1
8 3x 2 x 3

b A B C D E F M Y 0 f A B C D E F M Y 0
8 2x 8 2x x3
&
c A B C D E F M Y 0 g A B C D E F M Y 0
8 x3 8 3x 2 2 x x 3

d A B C D E F M Y 1 h A B C D E F M Y 3x 3
8 3x 2 2 x 8

Nguyn Minh Tun Trang 22


Tm ti sng to mt mt s cch gii phng trnh v t
VII. MT K THUT NH NH GI PHNG TRNH NGHIM KP.
Bi 1: Gii phng trnh: x 4 x 3 6x 4 2 x6 x 1 0
- Bi Th Vit Vted.vn -
Gii
u tin ta thy phng trnh c nghim kp x 1 v nhn t l 2 x 6 x 1 5x 3 0
v cng du vi bi ton. By gi s cn chng minh 2 x 6 x 1 5x 3 . Tuy nhin
kh l kh chng minh v ta s quy v bi ton sau:
Ly 2 x 6 x 1 2x 3 ax b trit tiu x6 bi v nu bin i trc tip th s hi di nn
d
ta s quy v mt bi ton n gin hn. Ta c
dx

2x6 x 1 2x 3
x 1
1 nn a 1 .

Ly 2x 6 x 1 2x 3 x 1 nn c b 1 . Vy cn chng minh
x1

2 x 6 x 1 2x 3 x 1 .
2
4 x x 1 2x x 1 4x 4x x 2x 3
6 3 4 3 2


Ta c 2 .
2 1
4x 4 4x 3 x 2 2x 3 x 1 4 x 2 0
2
Nn 2 x 6 x 1 2x 3 x 1
2
Khi VT x 1 x 2 3x 5 0
Du = xy ra khi x 1

Bi 2: Gii phng trnh: x9 2x 2 x8 x 1 2 x x 3 2x 2 2x 3


Nguyn Minh Tun
Gii
u tin ta c th nhn thy v tri lun dng nn dn ti iu kin ko theo ca phng trnh
s l: 2 x x 3 2x 2 2x 3 0 . Nhng tuy nhin nghim ca a thc bc 3 l nghim l nn
ta phi lm nh sau . Xt hm f x x 3 2x 2 2x 3 , ta c f ' x 3x 2 4x 2 0x nn hm
f x ng bin trn v c ti a duy nht mt nghim. n y ta c 2 hng l ch ra
nghim v xc nh gn chnh xc nghim. u tin vi cch lm th nht ta ch ra nghim
l:

2 3 404 4 10233 3 404 4 10233


x x0
3 6
Hoc nhn thy f 0,7 .f 0, 8 0 nn phng trnh c 1 nghim x x0 0,7; 0, 8 . n y ta
c iu kin ko theo s l x x0 ; 2 .
thy:
f x
2

1 1 1 2 1 1
1. x8 x 1 x 4 x x 1 4x 3 4x 2 4x 3 x 4 x
2 2 4 2 2
2
1 1 1 2
2. x 2x 2 x 4 x x 1 4x7 4x 6 4x 5 4x 4 8x 3 8x 2 8x 7 f x
9

2 2 4

Nguyn Minh Tun Trang 23


Tm ti sng to mt mt s cch gii phng trnh v t
2 2
1 1 1 1 1 5
3. x x x 2 x
4
0x
2 2 4 2 2 16

2
Nn VT x 1 3x 2 6x 7 0 . Du " " ch xy ra khi x 1

Bi 3: Gii phng trnh: 2 x 9 4x 4 4x8 4x 1 x 5 2x 3 x 2 8x 9 0


Nguyn Minh Tun
Gii
8 9
t f x 4x 4x 1; g x x 4x 4 .
4
x 8 x1
1 9
Ta c : f ' x 32x7 4 0 x 7 x x 0 g ' x 9x8 4 0
8 4
x 8 x2
9
Lp bng bin thin cho 2 hm f x ; g x ta c:
1
x 7
8
f ' x 0

f x

f x0

4 4
x 8 8
9 9
g ' x 0 0
g x1
g x
g x2

Ta c: f x0 1.60044 0 phng trnh f x 0 c ti a 2 nghim.


f 0, 25 .f 0, 3 0
Mt khc . Nn phng trnh c 2 nghim thuc 2 khong
f 0, 95 .f 0, 96 0
0, 25; 0, 3 ; 0, 95;0, 96
g x 1 7, 21; g x 2 0, 78 nn phng trnh g x 0 c nghim duy nht thuc
1, 3; 1, 2 do g 1, 3 .g 1, 2 0 .
Gi x 3 ; x 4 l 2 nghim ca phng trnh f x 0 v x 5 l nghim ca phng trnh
g x 0 . Lp bng xt du cho 2 hm f x ;g x .

Nguyn Minh Tun Trang 24


Tm ti sng to mt mt s cch gii phng trnh v t
x x5 x3 x4
f x 0 0
g x 0 +
KX l x x 5 ; x 3 x 4 ; .
Xt x x 5 ; x 3 1, 3 x 0, 3 .
2
7 2 91
t : h x x 5 2x 3 x 2 8x 3 h ' x 5x 4 6x 2 2x 8 5 x 2 x 1 0
10 20
h x ng bin trn 1, 3; 0, 3 h 1, 3 h x h 0, 3 17, 03 h x 6, 561 .
Li c 2 g x 5 0; 2 g x 1 3, 69; 2 g x 2 1, 54; 2 g 0, 3 2, 366
17, 03 h x 2 g x 1 0 . Nn phng trnh v nghim trn x 5 ; x 3 .
Xt: x x 4 ; x 0, 95
Ta c:
2 2
2 x 9 2x 2 2x 4
3x 3 x 1 4x7 4x 6 4x 5 4x 4 16x 3 16x 2 16x 7
2x 4 3x 3
Khi :
VT x 5 2x 4 2x 3 x 2 5x 6 4x8 4x 1
2
x 1 x 3 2x 2 x 1 2x 4 6x 7 4x 8 4x 1
2
2
x 1 x 2x x 1
3 2
x 1 24x 3
20x 2 16x 48
2x 4 6x 7 4x8 4x 1
Nhn thy:
1. v x 2x 4 6x 7 v' x 8x 3 6 0 v x v 0, 95 0
x 3 2x 2 x 1 0
2. 3 2
24x 20x 16x 48 0
Nn VT 0 . Du = xy ra khi x 1

Nguyn Minh Tun Trang 25


Tm ti sng to mt mt s cch gii phng trnh v t
B. K THUT NHN LIN HP, PHN TCH NHN T
MT S PHNG TRNH V T C BN V TM
TRUNG.
I. BI.
1. Gii phng trnh: x 3 2x 2 10x 6 x 2 8x 10 x 1 2 x 1 x 3 1 0
5x 13 57 10x 3x 2
2. Gii phng trnh: 2 x 3 x 2 2x 9
x 3 19 3x
3. Gii phng trnh: 2 x 2 5 x 2 x 1 x 2 5 7x 5
4. Gii phng trnh: x 2 16x 32 2x x 1 4 x 6 x 2 0
2 1 2
5. Gii phng trnh: 2

x 1 x 2x 1 1 x 2x 1

6. Gii phng trnh: 3 x 4 3 5x 4 4x 2 18x 12 0


7. Gii phng trnh: 2 5x 3 x 1 5 x 1 3 x 3 5x 1


8. Gii phng trnh: x 3 x 2 x 1 x 4 x 2 1 x 1 x2 x 1 x2 x 1
9. Gii phng trnh:
x 4 x2 1

2 2x 4 x 2
4x x5 x1

10. Gii phng trnh: 1 x 1 2x 2 2x 1 x 1 x x
11. Gii phng trnh:
3x 4 1 x 2 5x 3 x 2 15x 2 29x 12 1 x 2 x 2 0
6 x x3 x2 x
12. Gii phng trnh: 8x 14 2 x2 x2
x 1 x 2 2 x 1
13. Gii phng trnh: 7x2 20x 86 x 31 4x x2 3x 2
14. Gii phng trnh: 5x 2 5x 10 x 7 2x 6 x 2 x 3 13x 2 6x 32

15. Gii phng trnh: x x 2 x3 4x2 5x x3 3x2 4


x 3 4x 2 3x 2 x 3 3 2x 3
16. Gii phng trnh: x 4 1 x2
3
2x 3 3 x4 1
17. Gii phng trnh:
2x 1 x3
1

2 x 2 x 1x 1x
7x 2 2x
18. Gii phng trnh: 2 2x x 2
x 3 x 1 2x x 2
4 3
x 3x
19. Gii phng trnh:
x3 1

x 2 1 x 2 5x 6 x 2
x 2 3x x 1 3 x4
20. Gii phng trnh: 4 2
x5 x4 x1 x 1

Nguyn Minh Tun Trang 26


Tm ti sng to mt mt s cch gii phng trnh v t
3 9x 2 8x 4
x

21. Gii phng trnh: 2 2 x 1 1
3x 2 2x 1

Nguyn Minh Tun Trang 27


Tm ti sng to mt mt s cch gii phng trnh v t
II. HNG DN GII.
Bi 1: Gii phng trnh:
x 3 2x 2 10x 6 x 2 8x 10 x 1 2 x 1 x 3 1 0
Gii
Bc 1: Tm nghim, SOLVE c 2 nghim l A,B:

A B 8
- D dng nhn thy nn 2 nghim ny cng thuc 1 phng trnh bc 2. Do
AB 10
ta ch cn quan tm n 1 nghim tm nhn t.
Bc 2: Tm nhn t :
1. MODE 7 vi hm F X A 2 A 1 X A 1 .
2. Sau khi d tm ta s tm c nhn t l 3 x 1 x 2 x 1

Bc 3: Chia cn: 1000 X . Ta c kt qu l:


x 3 2x 2 10x 6 x 2 8x 10 x 1 2 x 1 x 3 1
3 x 1 x2 x 1
2 x x 1 x x 2 x 1 x 3 1 3 3x

Bc 4: Xt bt phng trnh: 2 x x 1 x x 2 x 1
- Nu x 2 th bt phng trnh lun ng.
- Nu x 2 ta c:
2 x x 1 x x2 x 1
x 4 6x 2 8x 4 0
2
2 4
x 4 6 x 0 (ng)
3 3
Nn 2 x x 1 x x 2 x 1 x 3 1 3 3x 0x 1;
Vy li gii l:
x 3 2x 2 10x 6 x 2 8x 10 x 1 2 x 1 x 3 1 0


3 x 1 x 2 x 1 2 x x 1 x x 2 x 1 x 3 1 3 3x 0

x 4 6
3 x 1 x2 x 1 0
x 4 6
y do mnh trnh by tt cho nn khi vo bi vn phi chng minh v nghim cho
ngi chm thy nh.
Tm ti sng to mt mt s cch gii phng trnh v t
5x 13 57 10x 3x 2
Bi 2: Gii phng trnh: 2 x 3 x 2 2x 9
x 3 19 3x
thi th THPT Quc Gia 2016 THPT Ph C
Gii
Bc 1: Nhn th ny c v kh kh lm, nhng chu ta thy rng:
5x 13 57 10x 3x 2
2 x 3 19 3x
x 3 19 3x
C th dng my tnh chia nu thch. Do phng trnh tr thnh:
4 x 3 19 3x x 2 2x 9 0
Bc 2: Tm nghim, SOLVE c 2 nghim l:

Bc 3: Kim tra nghim bi. Ta c:


4 x 3 19 3x x 2 2x 9
lim 0
x 1 x1
4 x 3 19 3x x 2 2x 9
lim 0
x 2 x2
Nn phng trnh khng c nghim bi.
Bc 4: Tm nhn t c dng x 3 a 19 3x b cha 2 nghim x 1; x 2
1 3 a 19 3 b 0 a 1
C h NT : x 3 19 3x 6 .
2 3 a 19 6 b 0 b 6
Bc 5: Chuyn sang MODE 2 CMPLX . Chia nh bnh thng c kt qu:
4 x 3 19 3x x 2 2x 9
x 3 19 3x 6
1
12 x x 3 x 6 19 3x 3 57 10x 3x 2 3x 31
4
+ Vi iu kin cho d dng chng minh c biu thc cn li dng.
Vy bi ton c gii quyt!
3x 3 21x 2 58x 56
Bi tp tng t: Gii phng trnh: x 2 x2 x 6
2
3x 6x 19 3 x 1

Bi 3: Gii phng trnh: x 2 16x 32 2x x 1 4 x 6 x 2 0


Bc 1: Tm nghim ta c nghim x 2
Bc 2: Kim tra nghim bi:
x 2 16x 32 2x x 1 4 x 6 x 2 0
lim 0
x 2 x2
x 2 16x 32 2x x 1 4 x 6 x 2 0
lim 2
0
x 2
x 2
Tm ti sng to mt mt s cch gii phng trnh v t
x 2 16x 32 2x x 1 4 x 6 x 2 0
lim 3
0
x2
x 2
x 2 16x 32 2x x 1 4 x 6 x 2 0
lim 4
0.03
x2
x 2
Nn phng trnh c nghim bi 4 x 2 .
2
Bc 3: Tm nhn t c dng x1 a x2 b
d
dx
x1
x2

a 2 2
Vi d
x2 NT : x 1 2 x
2 3
dx x2

b x 1 2 x 2 3
Bc 4: Chia cn, ta c:
x 2 16x 32 2x x 1 4 x 6 x 2 1
2
4 x 2 x 2 5x 16
9

x1 2 x2 3
Vy li gii l:
x 2 16x 32 2x x 1 4 x 6 x 2 0
1 2

9
x1 2 x2 3 4
x 2 x 2 5x 16 0

x1 2 x2 3 0 x 2

2 1 2
Bi 4: Gii phng trnh: 2

x 1 x 2x 1 1 x 2x 1
thi th THPT Quc Gia ln 2 THPT Minh Chu Hng Yn 2015
Gii
u tin nu nh th ny th chc khng lm c g, ta s phi quy ng ln.
Bc 1: Tm nghim. Ta c x 1 l nghim kp.
Bc 2: Tm nhn t: 2 x 2x 1 1.
Bc 3:Chia cn ta c kt qu:
2 x 2x 1 x 1 2 1 x 2x 1 2 x 1 2 x 2x 1

2 x 2x 1 1
1
4x 1 2x 1 2x 3 x 5 x 2x 1 6x 1
2
Vy bi ton c gii quyt!

Bi 5: Gii phng trnh: 3 x 4 3 5x 4 4x 2 18x 12 0


Gii
bi ny ta s tm c 2 nghim l x 0; x 3,791287847 .
i vi nghim l th s tm c nhn t l x 4 5x 4 2 . Nhng vi nghim
x 0 th sao? Thng thng nu khng tm c nghim i du th s gi s nhn t c
dng x 4 5x 4 k; x 4 5x 4 k . Ta s thay nghim x 0 vo ri chia
cn.
1. TH1: Nhn t l x 4 5x 4 , thc hin php chia thy l chot.

Nguyn Minh Tun Trang 30


Tm ti sng to mt mt s cch gii phng trnh v t
2. TH2: Nhn t l x 4 5x 4 4 , thc hin php chia cng thy l chot. Nhng
hy rng phng trnh x 4 5x 4 4 0 c 1 nghim ngoi lai l x 12
m phng trnh u khng h c nhn t ny cho nn ta s cn nhn thm mt lng
vo. Thng thng th s nhn x 12 vo phng trnh u th cng khng sao,
nhng ta nn nhn vo 1 biu thc v nghim phi chia trng hp. Li nhn
thy x 12 x4 4
x 4 4 , nn s nhn thm
x 4 4 vo l s chia
cn c.
Khi kt qu php chia l:

x 4 4 3 x 4 3 5x 4 4x 2 18x 12
x 4 5x 4 4 x 4 5x 1 2
1

2

7x 12 2x 6 x 4 9 5x 4 3 x 4 5x 4
Cch 2: Lin hp:
+ Xt x 5; x 9 ta c:
3 x 4 3 5x 4 4x 2 18x 12 0
3
x 4 x 2 4x 2 3 5x 4 x 2 6x 6 0
3x x 2 3x 3 5 x x x 2 3x 3 9 x
0
x 4 x 2 4x 2 3 5x 4 x 2 6x 6

x 0

3 21
x
x 2 3x 3 0 2
3 21
x
2
35 x 9x
f x 0
x 4 x 4x 2 3 5x 4 x 2 6x 6
2

9 129 9 129
+ iu kin c nghim ca phng trnh l x ; .
4 4
+ Xt x 0, 6; 5 . Nhn thy
5 x 0 3 5 x
1. 2 0
x 4x 2 0 x 4 x 2 4x 2
9 x 0 9x
2. 2 2
0.
x 4x 2 0 3 5x 4 x 6x 6
3 5 x 9x
2
0x 0, 6; 5 .
x 4 x 4x 2 3 5x 4 x 2 6x 6

+ Xt x 5;6 . t y x 4 x 2 4x 2 y'

2 1x x 4 4 3 x 0x 5; 6 .
2 x4 2

Nguyn Minh Tun Trang 31


Tm ti sng to mt mt s cch gii phng trnh v t
y nghch bin trn 5; 6 . M y 5 0 y 0 . Mt khc
3 5 x 9 x 0 9x
5x 0 2
0 . Cng c 2 2
0.
x 4 x 4x 2 x 4x 2 0 3 5x 4 x 6x 6
Nn f x 0x 5; 6 pcm.
3 21
Th li thy x 0; x tha mn phng trnh.
2
Cch 3: Lin hp + hm s:
3 x 4 3 5x 4 4x 2 18x 12 0
3 x4 x1 3
5x 4 x 1 4 x 2 3x 3 0
3 x 2 3x 3 3 x 2 3x 3
4 x 2 3x 3 0
x4 x1 5x 4 x 1
2
x 3x 3 0
3 3
f x 40
x 4 x1 5x 4 x 1
6 x4 3 6 5x 4 15
+ C f ' x 2
2
0.

2 x 4 x 4 x1
2 5x 4 5x 4 x 1
5
Nn f x ng bin trn ; . Li c f 0 0 nn x 0 l nghim duy nht ca phng
4
trnh f x 0 .
3 21
Vy phng trnh c 2 nghim x 0; x
2

Bi 6: Gii phng trnh: 2 5x 3 x 1 5 x 1 3 x 3 5x 1


Gii
u tin ta tm c nghim l x 3; x 0, 44; x 0, 2 . Khi gi s nhn t c dng
x 1 a 3 x bx c . Thay 3 nghim vo ta tm c nhn t l
8 x 1 4 3 x 5x 1 .
Tin hnh chia cn ta c :
2 5x 3 x 1 5 x 1 3 x 3 5x 1
8 x 1 4 3 x 5x 1
2 x1 3x 1
Vy bi ton c gii quyt.

Bi 7: Gii phng trnh:



x3 x2 x 1 x4 x2 1 x 1 x2 x 1 x2 x 1
http ://k2pi.net.vn
Gii
u tin ta s i tm nghim ca phng trnh, nhng tuy nhin c nghim rt l, khng
th tm nhn t c ta s ngh ti vic tm nghim i du. i du 1 trong 2 cn u thu
c nghim x 0 . Nn c nhn t l x2 x 1 x 2 x 1 .
Thc hin php chia ta c:

Nguyn Minh Tun Trang 32


Tm ti sng to mt mt s cch gii phng trnh v t


x3 x2 x 1 x 4 x2 1 x 1 x2 x 1 x2 x 1 x x2 x 1 x 1
2 2
x x1 x x1
n y s gii phng trnh bc 4 nghim cn trong cn. Ta c:
x x2 x 1 x 1 0
x x2 x 1 x 1
x x 1 0
x x 1 0
4 3
2 1 5 1 5 2 1 5 1 5
x x 2x 1 0 x 2 x 2 x 2 x 2 0

x x 1 0

1 5 1 3 5
1 5 1 3 5
2 2
x 2 2
2 x
2
1 5 1 3 5

x 2 2

2
Ngoi cch chia cn nh trn nu ai tinh c th bin i nh sau:

x3 x2 x 1 x 4 x2 1 x 1 x2 x 1 x2 x 1

x x2 x 1 x 2
x x1
x 1 x 2 x 1 x 1 2

x2 x 1

x x2 x 1 x x 1 x x 1 x 1 x x 1
2 2 2
x x 1
2

x x 1 x x 1 x x x 1 x 1 0
2 2 2

1 5 1 3 5

x x2 x 1 x 1 0 x 2 2
2

Bi 9: Gii phng trnh:


x 4 2 2x 4 x 2
x2 1

4x x5 x1
ngh Olympic 30/4/2014 Chuyn L Qu n Bnh nh
Gii
Cch 1: Phn tch nhn t trc tip.
Ta vn s lm nh bnh thng, quy ng ln v phn tch nhn t ta c:
x 4 x 2 1 2 2x 4 x 2
4x x5 x1

x 1 x 4 x 2 1 2 2x 4 x 2 4x x5
1

2
x2 4x 2 2x 1 x 2 4 x 4 x x 2 4 x x 2 6x 14 0
x2 4x 2 0 x 3
Cch 2: Phn tch nhn t tng thnh phn a v phng trnh n gin hn.
Ta c:

Nguyn Minh Tun Trang 33


Tm ti sng to mt mt s cch gii phng trnh v t
x 4 x2 1

2 2x 4 x 2
4x x5 x1


x 3 x2 x2 1
x 2 1 2 x 2 2x 2
4x x5 x1
x 3 x 2 2 x 2 2x 2

4x x5 x1

x 1 x 3 x 2 4 x x 5 2 x 2 2x 2 0
1

2
x2 4 x 2 x 5 x 2 3 x 4 x x 2 4x x4 0
Cch 3: t n ph.
a x 2
t a2 b2 2
b 4 x
Phng trnh tr thnh:
1 4 x x 2 2 2 x 2 x 2 1 ab 2 2 2a 3

1 4 x 4 x x 2 1 1 b2 b a2 1
1 ab 2 a 2 1 2 2a 3 b 2 b 1
a 2 1 a 3 b 2 ab 2 2b 2 2b 2 2a 3 b 2 2a 3 b 2a 3
a 3 a 3 b 2 2a 3 b 2b 2 2b 1 a 3 a 2 ab 2 0
a 3 b 2 2b 1 b 2 2b 1 b 2 ab 2 a 3 a 2 0
b 2 2b 1 a 3 1 2 a 2 a 2 a 2 a 3 a 2 0 b 2 2 a 2
2
b 1 a3 1 2 a 1 a2 1 0
2 2
b 1 a3 1 2 a 1 a 1 0 *
a 1 0 b 1 x 2 1
Do a, b 0 3 , nn * x3
a 1 0 a 1 4 x 1
Bi 10: Gii phng trnh: 1 x 1
2x 2 2x 1 x 1 x x

Gii
Tin hnh lin hp ngc ta c:
1 x 1 2x 2x 1 x 1 x x
2

1 x 1 2x 2x 1 x 1
2
x11 x1 1 x

2x 2x 1 x 1 x 1 1 x
2

2x 2 2x 1 x 2 x x x 1 0
x 2 3x 1
x x 1 0
2x 2 2x 1 x 2 x

Nguyn Minh Tun Trang 34


Tm ti sng to mt mt s cch gii phng trnh v t


x
x 1 x x 1 x x 1 0
2 2
2x 2x 1 x x
3 5
x x 1 0 x
2
2x 2x 1 x x x x 1 *
2 2

Gii * ta c:
2x 2 2x 1 x 2 x x x 1
2x 2 2x 1 x x x 2 x 1
2


2x 2 2x 1 x x x 2 x 1
2 x x2 x
x x1 0 x 0

Bi 11: Gii phng trnh:


3x 4 1 x 2 5x 3 x 2 15x 2 29x 12 1 x 2 x 2 0
Gii
Nhn pht thy ngay phng trnh v nghim do KX khng tha mn.
Vy cu hi t ra l c th phn tch phng trnh v t ny hay khng. Cu tr li l c!
Trc tin ta nh rng z zi , y l mt kin thc ca s phc, ng dng ca n rt
hay trong
vic tm nhn t khng tha mn KX.
By gi n gin l ta s i du trong cn ca phng trnhu. Trc tin c i du ca
x 2 2 x . SOLVE ta c nghim x 1 . i du trc cn 2 x ta c
14
nghim x . Do nhn t ca phng trnh i du trong cn l 2 x 3x 4 . Tr
9
li cn ta c nhn t ban u l x 2 3x 4 .
Chia cn ta c kt qu:
3x 4 1 x 2 5x 3 x 2 15x 2 29x 12 1 x 2 x 2
1 x 2 5x 3
x 2 3x 4
Vy li gii l:
3x 4 1 x 2 5x 3 x 2 15x 2 29x 12 1 x 2 x 2 0
1 x 2 5x 3 x 2 3x 4 0
1 x 2 5x 3 0 PTVN
0

x 2 3x 4 0 PTVN 0

6 x x3 x2 x
Bi 12: Gii phng trnh: 8x 14 2 x2 x2
x 1 x 2 2 x 1
thi th THPT Quc Gia S GD&T tnh Bnh Phc
Gii
Ta c:

Nguyn Minh Tun Trang 35


Tm ti sng to mt mt s cch gii phng trnh v t
6 x x3 x2 x
8x 14 2 x2 x2
x 1 x 2 2 x 1
x2 2
x 2 2 x3 x2 x

x 1 x 2 2 x1
8x 14 x2 2 x2

x3 x2 x
x2 2 8x 14 x 2 0
x 1 x 1
x2 2 x 6

x3 x2 x
x 1 x 1 8x 14 x 2 *

Gii * ta c:
2
x 3 x2 x
8x 14 x 2
x3 x2 x 8x 14 2 x 2
3
x 1 x 1 x 1
3x 2 4x 8 21x 4 26x 3 53x 2 42x 49 0
22 7
x t/m
2
3x 4x 8 0 3
22 7
x L
3
2 2
13 3 41 63 49
V f x 21x 26x 53x 42x 49 21 x 2 x x
4 3 2
0
21 2 21 82 82
22 7
Vy phng trnh c 2 nghim: x 6 x
3

Bi 13: Gii phng trnh: 7x2 20x 86 x 31 4x x2 3x 2


thi th THPT Quc Gia Chuyn Phan Bi Chu Ngh An Ln 1 - 2016
Gii
Ta c:
7x 2 20x 86 x 31 4x x 2 3x 2
7x 2 20x 86 3x 2 x 31 4x x 2
2


7x 2 20x 86 3x 2 x 31 4x x 2
31 4x x 2 1 31 4x x 2 4 31 4x x 2 x 2 7 0
x 2 34 t/m
31 4x x 2 1 0
x 2 34 L

x 2 19 t/m
31 4x x 2 4 0
x 2 19 L
Vy phng trnh c 2 nghim x 2 34 x 2 19

Nguyn Minh Tun Trang 36


Tm ti sng to mt mt s cch gii phng trnh v t
Bi 14: Gii phng trnh: 5x 2 5x 10 x 7 2x 6 x 2 x 3 13x 2 6x 32
thi th THPT Quc Gia 2016 ln 2 THPT Lc Ninh Bnh Phc
Gii
Cch 1: Lin hp + Chng minh v nghim.
Ta c:
5x2 5x 10 x 7 2x 6 x 2 x3 13x2 6x 32
5x 2 5x 10
x 7 3 2x 6
x 2 2 x 2 x2 5 0
5x 2 5x 10 2x 6
x 2 x2 5 0
x7 3 x2 2
x 2
2
f x x 2 5 5x 5x 10 2x 6 0
x7 3 x2 2
thy: x 7 3 5 3 5 v x 2 2 2 .
5x 2 5x 10 2x 6
Khi : f x g x x 2 5 0
5 2
Nn f x 0 PT : f x 0 v nghim!
Ngoi ra cn mt cch chng minh na c phn chng minh v nghim!
Cch 2: Phn tch nhn t.
Nh phn trn ta bit cch tm nhn t v nghim bng phng php i du trong
cn. p dng vo bi ny ta s tm c 1 nhn t l x 7 2 . Do s phn tch
phng trnh thnh:
5x 2 5x 10 x 7 2x 6 x 2 x3 13x2 6x 32
1

2
x 7 2 x 7 x 2 1
x 11 x 2 x 4 x 7 3
2 2
x 2 x 7 x 2 3x 2 0
x2
thy:
x2 11 x 2 x2 4 x 7 3 x 2 x 7 x2 3x 2
x 2 4 x 7 x 2 3x 2 2 x 2 4 x 2 3x 2 0
Vy bi ton c gii quyt!

Bi 15: Gii phng trnh: x x 2 x3 4x2 5x x3 3x2 4


Gii
Ta c:
x x 2 x 3 4x 2 5x x 3 3x 2 4
x 1 x 2 x 1 x 2
x 2 x 4x 5x
x 1
3 2 3
x 3x 4 2

x 1 x 2 0 x 4

x 1 x 2
1 *
x 3 4x 2 5x x 3 3x 2 4

Nguyn Minh Tun Trang 37


Tm ti sng to mt mt s cch gii phng trnh v t
Gii * ta c iu kin c nghim l x 0; 1 . Ta c:

1
x 1 x 2
x 3 4x 2 5x x 3 3x 2 4
x x 2 4x 5 1 x 1 x 2
2
x2 0
2


x 2 x
0
x x 2
2
x 4x 5 1 x11

x2 x
x 2 x 0x0
x 2 4x 5 1 x 1 1

0 x 0 ;1

x 3 4x 2 3x 2 x 3 3 2x 3
Bi 16: Gii phng trnh: x 4 1 x2
3
2x 3 3 x4 1
thi th THPT Quc Gia 2016 H Huy Tp Ngh An
Gii
Ta c:
x 3 4x 2 3x 2 x 3 3 2x 3
x 4 1 x2
3
2x 3 3 x4 1
x 3 4x 2 3x 2 x 3 3 2x 3
x 4 1 x4 1 x2 3
2x 3
3 2
x 4x 3x 18 6 x 3 2 x 3 3 2x 3
x 3 x 2 3
2x 3 3
3 2
x 4x 3x 18
x 3 x2 2 3
2x 3 3
x 3
2
x2 2 x2 2
x 3 x2 2 3
2x 3 3
x 3 x2 2
1 3
2x 3 3
2x 3 2x 3 x 3 x 2
3

3 3
3 2x 3 3
2x 3 x2 x2 f 3

2x 3 f x2
3 2
Xt f t t t lin tc trn . C f ' t 3t 1 0 . Nn f t ng bin trn .
Khi :
Bpt 3 2x 3 x 2
x 3 2x 2 1 0
3 1 5
2x 3 x 2 3 x 1;
x 2
2
x 2 x 2 3 2x 1
Bi tp tng t [THPT Triu Sn 1 Thanh Ha]: x1 3
2x 1 3

Nguyn Minh Tun Trang 38


Tm ti sng to mt mt s cch gii phng trnh v t

Bi 17: Gii phng trnh:


2x 1 x3
1
2 x 2 x 1x 1x
Gii
1
Xt x , hin nhin phng trnh v nghim.
2
1
Xt x ; 1 ta c:
2
2x 1 x 3 1

2 x 2 x 1x 1x
2x 1 x 3 2 x 2 x 1x 1x
2
1x 4x 2
12x 3

2x 1 x3 2 x

1x 2 x 1x
4x 2 12x 3 1 x
1x 2 x 1x 0
2x 1 x 3 2 x
1x 0 x 1
4x 12x 3 1 x
2
1
Ch : 2 x 1 x 0x ;1
2x 1 x 3 2 x 2
Vy phng trnh c nghim duy nht x 1 .

7x 2 2x
Bi 18: Gii phng trnh: 2 2x x 2
x 3 x 1 2x x 2
Gii
Bi ny nhn thong qua thy khng b nhng rt l d, nh!
Ta c:
7x 2 2x
2 2x x 2
x 3 x 1 2x x 2

2 2x x 2

x 2 2x x 2 2 2x x 2
x 3 x 1 2x x 2
x 3 x 1 2x x 2 x 2 2x x 2

x3 x1 2x x 2 x 2 2x x 2 0
x1 1 x1 2 x 2 2x x 2 2x 3x 1 0
x 1 2 3x 1 2x x 2
x 2 0
x1 1 2x x 2
x 2

x 1 2 3x 1 2x x 2
f x x1 1

2x x 2
0

thy:

Nguyn Minh Tun Trang 39


Tm ti sng to mt mt s cch gii phng trnh v t
x1 2 x1
6. 2 0.
x1 1 x1 1
3x 1 2x x 2 3x 1 2x x 2 2 2x 2 x 2
7. 2
2x x 2 2x x 2
8. Xt bt ng thc: 2x
x 2 2 2 x 2 3x 1
9x 2 2x 7
+ Do 2 x 2 3x 1 0x 1; . Nn vi x 2 th bt lun
2 x 2 3x 1
ng.
+ Xt x 1; 2 c:
2x
x 2 2 2 x 2 3x 1

2 x 2 3x 1
2x
x2 2
7x 2 2x 9 4x 4 x 2
49x 4 44x 3 66x 2 116x 49 0
4 3 2
49 x 1 152 x 1 228 x 1 80 x 1 4 0
+ D thy bt cui lun ng nn c pcm. Vy bi ton c gii quyt!

x 4 3x 3
Bi 19: Gii phng trnh:
x3 1

x 2 1 x 2 5x 6 x 2
Gii
Ta c:
x 4 3x 3
x3 1
x2 1 x 2 5x 6 x 2
x 3 1 x 3 1
x 4 3x 3 x 2 1 x2
3
x 3x x 1 x 2
4 3 2

x 2 1 x 2 2 x x 2 x 3x 3 2
x2 0
4 3 3
iu kin c nghim ca phng trnh l x 3x 0 x x 3 0 x 0
Vi K trn ta c x x 2 x 3 3x 2 x 2 0 . Vy phng trnh c nghim duy nht
x 2.

x 2 3x 3 x 1 x4
Bi 20: Gii phng trnh: 4 2
x5 x4 x1 x 1
Gii
1 13
Nhn bi ny hnh thc kh l chy ci. Ta s tm c nghim x , nhng tuy
2
nhin x 4 khng th biu din qua x c nn chc chn n y s b tc do bi ny
khng nh nhng bi trn c th lin hp ngc lm gim cng knh. Hy thy
1 13 x 2 3x 3 x 1
vi x th 1 , lc ny kh c x 4 nn ta c
2 x5 x4 x1
li gii sau:
Ta c:

Nguyn Minh Tun Trang 40


Tm ti sng to mt mt s cch gii phng trnh v t
x 2 3x 3 x 1 x4
4 2
x5 x4 x1 x 1
x 2 3x 3 x 1 x4
11 4 2
0
x5 x4 x1 x 1


x 1 x4 x4 x2 x1 x 4
0
x5 x4 x1 x1 x1 4 x4
1 13
x 1 x 4 0 x
2
x4 x2 1
0
x 5 x 4 x 1 x 1 x 1 4
x 4

0 x 1;

3 9x 2 8x 4
Bi 21:Gii phng trnh: 2


2 x1 1
x

3x 2 2x 1
thi th THPT Quc Gia 2016 THPT Trn Hng o knng
Gii
Ta c:
3 9x 2 8x 4


2
x
2 x 1 1
3x 2 2x 1

3
3x 2 2x 1 3x 2 2x 1
x

2 2 x1 1
3x 2 2x 1
3

2 2 x 1 1 3x 2 2x 1
x
2x 3 2 x 1 1 3x 2
2x 2x 1

2 2x 3 x 1 2x 2x 1 3x 2 2x 3 0
2 2x 3 x 1 2x
2x 1 1 x 1 3x 3 0
4x x 1
x 1 2 2x 3 3x 3 x 1 0
2x 1 1
x 1
4x x 1
f x 2 2x 3 3x 3 x 1 0
2x 1 1
thy 2x 1 x 1 nn :

f x 2 2x 3
4x x 1
3x 3 x 1
5x 9 x 1 3x 2 4x 3
x1 1 x1 1
9
- Vi x th hin nhin f x 0 .
5
9
- Vi x ta c:
5

Nguyn Minh Tun Trang 41


Tm ti sng to mt mt s cch gii phng trnh v t
9x 4 49x 3 149x 2 195x 90
f x
x1 1 5x 9 x 1 3x 2 4x 3
2
49 44 45563 2 2719 38826
9 x 2 x x x
18 25 900 25 625
0
2
x 1 1 5x 9 x 1 3x 4x 3
Vy phng trnh c nghim duy nht x 1
Cch 2:
Ta c:
3 9x 2 8x 4


2
x
2 x 1 1
3x 2 2x 1

2x 3 2 x 1 1 3x 2 2x 2x 1
2 2

2 x1 x 1 x 2x 1 2 x1 x1 0
V tri lun dng nn bi ton c gii quyt!

------------------------------------------------ Ht ---------------------------------------------

Nguyn Minh Tun Trang 42


Tm ti sng to mt mt s cch gii phng trnh v t
C. K THUT CHNG MINH V NGHIM.
I. PHNG TRNH A THC.
Phng trnh bc 4.
a) S dng tnh cht tam thc bc 2.
2
ax
Nn tng: Ta s phn tch phng trnh ban u thnh x 2 m f x trong f x l
2
mt tam thc bc 2 lun ln hn 0 vi mi x
V d: Chng minh phng trnh sau v nghim: x 4 x 3 3x 2 x 7 0
Bc 1: u tin ta bin i phng trnh theo tham s m nh sau:
x 4 x 3 3x 2 x 7 0
2
x 11
x 2 m 2m x 2 1 m x 7 m 2 0
2 4
x
Nhiu bn s t ra cu hi ti sao li l x 2 m . Rt n gin, khi ta khai trin biu thc
2
2
2 x 3 2 x 3
x m s xut hin ngay x v 2.x . x . Hiu ri ch, cc bi khc cng tch tng t
2 2
c nh vy, ch c iu ta phi a n v dng tng qut: x 4 2ax 3 bx 2 cx d 0 th mi
tch thnh nh trn.
Bc 2: Ta tnh theo tham s m:
2 11
1 m 4 2m 7 m 2
4
Bc 3: Ta thy phng trnh ban u v nghim th phng trnh

11 2 2
2m x 1 m x 7 m 0
4
0

Phi v nghim. phng trnh ny v nghim th 11
4 2m 0

Dng MODE 7 ,nhp hm sau vo my:


2 11
F X 1 X 4 2X 7 X 2
4
Start 10
End 10
Step 1
11
Sau ta tm cc gi tr X lm F X 0 & 2X 0
4
Nhn vo bng ta thy rt nhiu gi tr lm F X 0 , nhng tuy
11
nhin ta phi chn lm sao cho 2X 0 v phi l mt gi tr
4
b d rt gn. Vi l do nh th ti s chn X 0 hay m 0
Bc 4: Do bit m 0 nn phng trnh s tr thnh:
Tm ti sng to mt mt s cch gii phng trnh v t
x 4 x 3 3x 2 x 7 0
2
x 11
x 2 x2 x 7 0
2 4
2 2
x 11 2 76
x2 x 0
2
4
11 11

0 x
Nn phng trnh v nghim!
b) S dng o hm.
Ta xt phng trnh tng qut: x 4 ax 3 bx 2 cx d 0
Bc 1: o hm v tri: f ' x 4x3 3ax2 2bx c
Bc 2: Gii phng trnh f ' x 0 . Nu :
1. Phng trnh c 1 nghim th y l im ri ca bi ton.
2. Phng trnh c nhiu nghim th th xem nghim no lm v tri nh nht
Bc 3: Tm k sao cho:
2
ax
+ x 4 ax 3 bx 2 cx d x 2 k 0x
2
a
+ k x 0 2 x 0 nht.
2
Mc tiu ca phng php ny tng t nh phng php trn nhng c vi im ti u hn.
Bc 4: Sau khi ta tm c k th ch vic ly :
2
a
x ax bx cx d x 2 x k mx 2 nx p 0x
4 3 2

2
h x 0
Do f x g x h x m trong nn f x 0 . Th l xong bi!
g x 0

V d 1: Chng minh rng: f x x 4 x2 x 2 0x


Bc 1: o hm v tri f ' x 4x3 2x 1
Bc 2: Gii phng trnh f ' x 0 x x0 0..8846461771

Bc 3: Tm k:
a 4
k x 0 2 x 0 0.7825988 k 0.8
2 5
2
4 3
Bc 4: Ta ly: x 4 x 2 x 2 x 2 x 2 x 1, 36 0x
5 5
Do phng trnh ban u v nghim! Nhanh hn ch!
V d 2: Chng minh rng : f x 2x 4 x 3 2x 2 x 3 0

Bc 1: o hm: f ' x 8x3 3x2 4x 1 x x0 1


a 3
Bc 2: Tm k x 0 2 x 0
2 4
Tm ti sng to mt mt s cch gii phng trnh v t
2
1 3 7 2
Bc 3:Ly f x 2 x 2 x x 1 1 . Xong!
4 4 8
V d 3: Chng minh rng: f x 4x 2x 2x2 x 14 0
4 3

Bc 1: o hm f ' x 16x 3 6x 2 4x 1 0 x x0 0,7909677904


a 1
Bc 2: Tm k x 0 2 x 0
2 2
2 2
1 1 7 4 87
Bc 3: Ly f x 4 x 2 x x 0x
4 2 4 7 7
Phng trnh bc 6.
Ta xt phng trnh tng qut sau: f x x6 ax5 bx 4 cx3 dx 2 ex f 0
2
a
Ta s thm bt biu thc: x 3 x 2 mx n
2
2
a a2
Ly f x x 3 x 2 mx n b 2m x 4 ...
2 4
Gii phng trnh f ' x 0 x x0 tha mn min f x f x 0

m
a2
Tm m tha mn a 2 , thng thng ta s cho b 2m 1
b 2m 0 4
4
n

Tm n tha mn 3 a 2
x 0 x x0 mx 0 n 0
Khi tm c m,n bi ton coi nh c gii quyt!
Sau y l 2 v d tm hiu r cch lm.
V d 1: Chng minh rng: f x x6 2x5 x 4 4x2 2x 1 0
Ta c f ' x 6x5 10x 4 4x3 8x 2 0 x x0 0, 25219838

2
Ly f x x 3 x 2 mx n 2 2m x 4 ...
3
Ta tm m tha mn 2 2m 1 m
2
3 1
Ta tm n tha mn x 0 3 x 0 2 x 0 n 0 n
2 4
2 2
3 1 5 1 11 11
Ly f x x 3 x 2 x x 2 x x 2 0
2 4 4 2 16 16
Vy bi ton cgii quyt!

V d 2: Chng minh rng: f x x6 2x5 2x 4 4x3 8x2 2x 12 0


Ta c f ' x 6x 5 10x 4 8x3 12x2 16x 2 0 x x0 0, 115820665
Tm ti sng to mt mt s cch gii phng trnh v t

2
Ly f x x 3 x 2 mx n 3 2m x 4 ...
Ta tm m tha mn 3 2m 1 m 2
1
Ta tm n tha mn x 0 3 x 0 2 2x 0 n 0 n . thy f x0 11, 58 0 rt
4
nhiu nn y l mt bi ton kh lng lo. Do ta c th coi n 0 tin rt gn bng
my tnh.
2
Ly f x x 3 x 2 2x x 4 4x 2 2x 12 0
Vy bi ton c gii quyt hon ton

Phng trnh bc chn khng cht.


Phng php ny ch hu ch cho 2 dng my VINACAL 570es PLUS II v CASIO 570VN
PLUS bi v 2 dng my ny c tnh nng tnh min max ca 1 tam thc bc 2. i vi my
VINACAL th ta s bm q66 my s hin ln nh sau:

Cn my CASIO VN th tch hp trong chc nng gii phng trnh bc 2.

Ni dung
Phng php ny s dung tnh cht c bn ca tam thc bc 2 nh sau:Xt tam thc
2
2 b
f x ax bx c th ta lun c f x a x . Tng chng n gin nhng li
2a 4a
gip ch kh nhiu!

V d minh ha.
3 3
V d 1: Chng minh rng: f x x 4 3x 3 3x 2 x 0x
4 16

Gii
1. Nu quen vi phng php ny th s cho ra kt qu khong 15s .
2. Do ti dng my VINACAL nn s khi ng tnh nng tm min max.
3. Nhp vo my 1 3 3 , my s cho ra kt qu:

2
3 3x 2
Vy ta s c x 4 3x 3 3x 2 x 2 x .
2 4
3 3 3
4. Tip tc nhp ta li c kt qu:
4 4 16
Tm ti sng to mt mt s cch gii phng trnh v t

2
3x 2 3 3 3 1
Vy ta s c x x .
4 4 16 4 2
2 2
2 3 3 1
5. Vy ta c f x x x x 0 . Bi ton c gii quyt!
2 4 2

Nhanh ch!. y vn l bnh thng ta s chin mt v d tip theo!

V d 2: Chng minh rng: f x x6 3x5 3x 4 x3 2x2 x 1 0

Gii
1. Nhp 3 h s u vo my ta s c kt qu:

2
3 3
Vy s c x 3x 3x x x x 4
6 5 4 4

2 4

2. Nhp vo my 3 h s tip theo s c kt qu:

2
3 4 3 2 5
Vy s c x x 3 2x 2 x 2 x x 2
4 4 3 3

3. Nhp vo my 3 h s cui s c kt qu:

2
5 5 3 17
Vy s c x 2 x 1 x
3 3 10 20
2 2 2
3 3 2 5 3 17
4. Vy f x x x x 2 x x
4
0x . Vi diu cha .
2 4 3 3 10 20

By gi s chin nt v d cui cng!


Tm ti sng to mt mt s cch gii phng trnh v t
2 7 4 14 1
V d 3: Chng minh rng: f x x 8 x 3x 6 x 5 x 4 2x 3 3x 2 x 0x
3 3 3 3

Gii
Ch cn bm my khong 1 pht ta s c kt qu di y:
2 2 2 2
1 26 3 176 2 39 489 88 119
f x x6 x x 4 x x x x
3 9 13 39 176 176 489 489

T lm nh! Cui cng hy th sc vi bi sau y.


Chng minh:

f x x12 2x11 18x10 11x9 18x8 16x7 22x6 17x5 31x 4 10x3 20x2 10x 21 0

Ch rng:
1. Nu bn no khng c 2 dng my trn th vn c th tnh c nh trn nhng
b
mt thi gian tnh & . Nhng ng ngi nh lm nhiu s tin b .
2a 4a
2. Nu bn no c VINACAL hay VN PLUS th ng vi mng, nhiu khi gp phi
nhng bi h s xu th cng phi tnh tay thi v my tnh khng hin th c, th
l bng nhau. Tiu biu l bi bn trn ti cho, vui v nh.

Chng minh trn khong.


u tin xt dng tng qut cho cc bi ton c im ri khng cht.
Gi s cn chng minh phng trnh f x 0 v nghim trn b; ; ; a . Ta s CALC
sao cho X a 1000; X b 1000 sau khai trin nh bnh thng. hiu r hn ta s cng
chin mt v d ly .
V d 1: Chng minh rng : f x 3x 4 2x 3 2x 2 10x 4 0x 2;
1. Cch 1: Hm s
Ta c f ' x 12x3 6x2 4x 10 f '' x 36x2 12x 4
1 5
x
f '' x 0 6
1 5
x
6
Lp bng bin thin cho f ' x ta c:

1 5 1 5
x
6 6
f '' x 0 0
83 5 5

9
f ' x
83 5 5

9

Nguyn Minh Tun Trang 48


Tm ti sng to mt mt s cch gii phng trnh v t
Nhn vo bng bin thin ta c th thy phng trnh f ' x 0 c 1 nghim duy nht thuc vo
khong 0,9; 0, 8 do f ' 0,9 .f ' 0, 8 0 . Gi v nghim l x x0 0, 8997774777 .
Li tip tc lp bng bin thin cho f x ta c.
x x0 2
f ' x 0

f x 48
f x0
Nhn vo bng d thy f x 0x 2; . Vy l ht bi!
2. Cch 2: Nhm thnh tng da vo iu kin.

Ta d dng nhn thy x 2 x 2 0 nn ny ra tng vit f x di dng :


4 3 2
f x a x 2 b x 2 c x 2 d x 2 e
V cng vic ny s nh ti s tr gip ca th thut CASIO.
Ta s CALC X sao cho X 2 1000 X 1002
4
CALC X 1002 ta c kt qu 3, 022058 10 12 3 x 2
4 3
Ghi vo sau 3 X 2 , CALC X 1002 ta c kt qu 2, 205807 10 10 22 x 2
3 2
Ghi vo sau 22 X 2 , CALC X 1002 ta c kt qu 58074048 58 x 2
2
Ghi vo sau 58 X 2 , CALC X 1002 ta c kt qu 74048 74 x 2 48
Th li vi X ta c kt qu l 0. Vy kt qu lun ng
4 3 2
Vy f x 3 x 2 22 x 2 58 x 2 74 x 2 48 0x 2;
Th l bi ton c gii quyt!

V d 2: Chng minh rng: f x x5 x 4 x3 4x2 6x 1 0x 1


Gii
ng nh nhng bc lm bn trn ta s tch thnh:
5 4 3 2
f x x 1 6 x 1 13 x 1 17 x 1 20 x 1 10
x 1 5 0
3
thy vi x 1 13 x 1 0 f x 0x ; 1

20 x 1 0
V d 3: Chng minh rng: f x x7 x6 x5 2x 4 8x3 8x 2 10x 2 0x 1
Gii

Do bi ny bc tng i cao nn ta s c lm nh bnh thng v tm cc h s cn li


bng ng nht h s.
Ta c:
7 6 5 4 3 2
f x x 1 6 x 1 16 x 1 23 x 1 a x 1 b x 1 c x 1 f 1
Lp h, cho x ln lt bng 1,2,3 s tm c a,b,c.
Ta c:
7 6 5 4 3 2
f x x 1 6 x 1 16 x 1 23 x 1 25 x 1 20 x 1 16 x 1 7
Bi ton c gii quyt

Nguyn Minh Tun Trang 49


Tm ti sng to mt mt s cch gii phng trnh v t
Chng minh trn on.
Bi 1: Chng minh rng: f x x5 x 4 2x3 2x2 5x 3 0x 1;
Gii
thy:
5 4 3 2
1. f x x 1 4 x 1 4 x 1 4 x 1 4 x 1 4
2 2
4 3 2 3x 1 1 7 9 58
2. 4 x 1 4 x 1 4 x 1 4 x 1 4 4 x 2 x 0
2 5 20 7 175
3. Nn do f x 0x 1; (pcm). Xong! Ht bi.

Hng dn
Do ta ang cn chng minh f x 0x 1 nn ny ra tng tch thnh:
5 4 3 2
x 1 a x 1 b x 1 c x 1 d x 1 e

tch thnh nh vy ta s s dng my tnh cm tay gii quyt. thy vi


x 1 x 1 0 nn ta s nhp vo my v CALC sao cho X 1 1000 X 1001 v
s dng k thut xp x nh khai trin a thc ta s tch thnh dng nh trn. C th cc
bc lm nh sau:
1.1. Nhp vo my biu thc trn, CALC X 1000 ta c kt qu l
5
1.0... 10 15 x 1 .
5 4
1.2. Ghi vo sau X 1 CALC X 1000 ta c kt qu l 4.0... 10 12 4 x 1 .
4 3
1.3. Ghi vo sau 4 X 1 CALC X 1000 ta c kt qu 3.99... 10 9 4 x 1
3 2
1.4. Ghi vo sau 4 X 1 CALC X 1000 ta c kt qu 4003996 4 x 1
2
1.5. Ghi vo sau 4 X 1 CALC X 1000 ta c kt qu 3996 4 x 1 4 .
1.6.Nh rng tm h s t do ta s CALC gi tr mc tc l 1 v c kt qu l 4.
5 4 3 2
Vy ta c kt qu f x x 1 4 x 1 4 x 1 4 x 1 4 x 1 4 , th
li vi x ta thy kt qu lun ng. n y vn t ta l tt c khng phi du
" " nn ta cn phi x l thm 1 bc na. Tht may l biu thc bc 4 ng sau lun
dng nn ta s quy n v bi ton chng minh phng trnh bc 4 v nghim vi n
y x 1 . S dng th thut SOS ta s tch n thnh:

4 3 2
4 x 1 4 x 1 4 x 1 4 x 1 4
2 2
3x 1 1 7 9 58
4 x 2 x 0
2 5 20 7 175

Khi bi ton c gii quyt hon ton!

Nhn xt

Bi ton trn ch l dng c bit do biu thc f x kh l lng. Vy i vi nhng bi ton cht
khc m khi tch ra dng nh trn ton du " " th phi lm nh th no? Sau y s l cch gii
quyt.

Nguyn Minh Tun Trang 50


Tm ti sng to mt mt s cch gii phng trnh v t
1.1. Th nht ta s cn ni rng khong cn chng minh ta, c ngha l nu bi ton cho
x 1 th ta s chng minh hn n ln hn 0 vi x 3 chng hn, sau s chng
minh n ln hn 0 vi x 1; 3 .
1.2. chng minh f x 0x 1; 3 ta s s dng k thut chia tr DAC. ( p
dng chng minh v nghim trn on).
1.3. Ni dung phng php DAC: B : Cho hm s f x, y lin tc v xc nh trn
D a; b a; b Hm s f x, y ng bin theo x v nghch bin theo y . Khi nu
f a, b 0 th f x, y f a, b 0 .
1.4. Chng minh b :
+ Do hm s ng bin theo x , x a nn f x, y f a, y 1
+ Do hm s nghch bin theo y , y b nn f a, y f a, b 2
+ T 1 & 2 c iu phi chng minh.

p dng

1. i vi bi ny ta c gi v tch n di dng x 3 ta s c:

5 4 3 2
f x x 3 14 x 3 76 x 3 196 x 3 236 x 3 108 0x 3
2. Xt x 1; 3 . y l iu quan trng nht. Do b xt ti hm 2 bin nn ta s bin
f x thnh hm 2 bin da vo tnh ng bin nghch bin. Ta c:
2.1. x 5 ' 5x 4 0 Ch ny ng bin ta s t l x .
2.2. x 4 ' 4x 3 0 Ch ny nghch bin nn t l y.
2.3. Tng t vi cc ch cn li. Cui cng ta s t
g x, y x5 y 4 2y3 2y 2 5x 3 , hm ny chc chn ng bin theo x v
nghch bin theo y vi x, y D 1; 3 1; 3 .
3. Sau khi t xong hm g x, y ta cn phi chng minh n ln hn 0. Do b pht
biu nu f a, b 0 f x, y 0 th nhiu ngi s tng lun x 1& y 3 , nhng
ch tru l n m chot ra do ta nh gi qu mnh tay, v nhiu bn s ngh b sai,
nhng hy rng phng php ny c tn chia tr nn cc bn cn phi chia
1; 3 1; a a; b ... z; 3 v xt tng khong khi thay 2 cn vo n lun
dng, hiu ch?. tm cc khong kia phi s dng n ti sn qu bu l chic
my tnh.
3.1. Nhp hm g x, y vo my: X 5 Y 4 2Y 3 2Y 2 5X 3 . u tin bm
CALC v nhp X 1 trc do y l cn nh nht, sau ta th thay Y 3 vo
thy m th s chuyn Y 2 thy vn m. Chuyn tip Y xung 1, 5 th thy vn
369
m, lc ny ng hong ta s tm c Y 1, 2 th g x, y 0 , th l
625
tm c 1 khong u tin.
3.2. tm tip cc khong tip theo ta li cho X 1, 2 v tm Y. C lp li qu trnh
trn ta s chia c:
1; 3 1; 1, 2 1, 2; 1, 3 1, 3; 1, 39 1, 39;1, 46 1, 46; 1, 51
1, 51; 1, 56 1, 56; 1, 6 1, 6; 1, 64 1, 64; 1, 67 1,67;1,7 1,7;1,73

Nguyn Minh Tun Trang 51


Tm ti sng to mt mt s cch gii phng trnh v t

1,73; 1,76 1,76; 1, 8 ; 1,8; 1,84 1,84; 1, 88 1,88; 1, 93 1, 93;1, 99 1, 99; 2 .


Woa! Tht p mt. Lc n 2 l cc bn s gp kh khn do cc khong cng ngy cng hp. Ta
li ny tng chng minh f x 0x 2 . Ta s c:

5 4 3 2
f x x 2 9 x 2 30 x 2 42 x 2 21 x 2 5 0
Vy li gii s lc ca bi ny s nh sau:

1. Xt x 2 ta c
5 4 3 2
f x x 2 9 x 2 30 x 2 42 x 2 21 x 2 5 0
2. Xt x 1; 2 .
+ Ta c b sau: Cho hm s f x, y lin tc v xc nh trn D a; b a; b Hm s
f x, y ng bin theo x v nghch bin theo y . Khi nu f a, b 0 th
f x, y f a, b 0 .
+ Chng minh:

- Do hm s ng bin theo x , x a nn f x, y f a, y 1
- Do hm s nghch bin theo y , y b nn f a, y f a, b 2
- T 1 & 2 c iu phi chng minh.

+ Xt hm g x, y x5 y 4 2y3 2y 2 5x 3 f x g x, x .Hm s ng bin


theo x , nghch bin theo y , lin tc trn
1; 1, 2 ; 1, 2;1, 3 ; 1, 3; 1, 39 ; 1, 39;1, 46 ; 1, 46; 1, 51
; 1, 51; 1, 56 ; 1, 56; 1, 6 ; 1, 6; 1,64 ; 1, 64; 1, 67 ; 1, 67; 1,7 ; 1,7;1,73
; 1, 51; 1, 56 ; 1, 56; 1, 6 ; 1, 6; 1,64 ; 1, 64; 1, 67 ; 1, 67; 1,7 ; 1,7;1,73 .

369
g 1; 1, 2 625 0

+ Li c ... nn theo b ta s c
g 1, 99; 2 4, 1579601 0


f x g x, x 0x 1; 1, 2

...
f x g x, x 0x 1, 99; 2

T suy ra iu phi chng minh!

* Lu : Mt iu ng bun l khi vit trong bi khng c ghi m phi ghi ht ra


ngi ta cng nhn khng s b bt b ngay lp tc. Ni chung cch lm tng qut bao gi
cng di hn cch lm dng IQ m . Sau y l mt s bi c th lm theo DAC.
Bi 2: Chng minh rng: f x x8 x5 x2 x 1 0x

Gii
1. Cch 1: To dng hng ng thc

Nguyn Minh Tun Trang 52


Tm ti sng to mt mt s cch gii phng trnh v t
2 2
1 3 2 2
Ta lun c: f x x x x x 1 x 4 x x 0
8 5 2

2 4 3 3
T suy ra iu phi chng minh.
2. Cch 2: DAC
Nhn cch 1 c v rt ngn gn nhng s nhiu bn c th khng nhn thy du hiu tch hng
ng thc th ta vn c th lm nh sau:
2
2 1 3
x x 1 x 0
Xt x 0 khi x 5 0 li c 2 4 nn c iu phi chng
x 8 0

minh.
Xt x 1 khi x 8 x 5 x 5 x 3 1 x 5 x 1 x 2 x 1 0 ta cng c iu phi
chng minh.
Xt x 0; 1 y l khu quan trng nht. Cch lm DAC s nh sau:
+ Bc 1: Pht biu, chng minh b .
+ Bc 2: t hm g x, y sao cho hp l m bo lun ng theo b ( rt quan
trng!). t hm g x, y ta s o hm tng bin mt v xt tnh ng bin, nghch
bin.Nh l ch no ang ng s t l x, nghch bin l y.
x 8 ' 8x7 0

x 5 ' 5x 4 0
C: . Nn s t hm g x, y x8 y 5 x2 y 1 .
x ' 2x 0
2


x ' 1 0
+ Chia tr: chng minh v nghim c ta s phi chia thnh cc khong nh
a;m ; m; n ;...; y; b lm sao cho khi ta thay cn min bng x v cn max bng y th
g x, y 0 . Cng vic ny c casio h tr.
Nhp vo my X 8 Y 5 X 2 Y 1 . Ta s CALC X 0 trc v th cho vi Y 0
lun xem c dng khng. Nhng tic l biu thc b m do ta nh gi qu tri, v v
th cn thu nh khong li. Th CALC tip v cho Y 0, 5 xem.Ln ny dng,
nhng ta c th ni rng khong hn na th cho Y 0, 7 ln ny cng dng nhng nu
ni rng ra hn na s b m.Th l tm c mt khong. Ta s lp li qu trnh trn
vi X 0,7 v s phi tm Y. Ln lt tm c 2 khong na l 0,7;0, 9 & 0, 9; 1 .
+ Bc 3: Li gii:
- Vit li bc 1.
- t g x, y x8 y 5 x2 y 1 lin tc trn cc khong
0; 0,7 ; 0,7; 0, 9 ; 0,9;1 . ng bin theo x, nghch bin theo y, c f x g x, x
.
g 0; 0,7 ... 0 f x g x, x 0x 0; 0,7

- Li c g 0, 7; 0, 9 ... 0 f x g x, x 0x 0,7; 0, 9 .

g 0, 9; 1 ... 0 f x g x, x 0x 0, 9; 1
- Suy ra iu phi chng minh.
Vy bi ton c gii quyt! Hay ch . Chin 1 ci na no!

Nguyn Minh Tun Trang 53


Tm ti sng to mt mt s cch gii phng trnh v t
1 1 4
V d 2: Chng minh rng: f x x 6 x 5 x 4 2x 2 x 0x ;
10 9 3

Gii
y l 1 bi ton kh l cht nn chc chn s phi chia tng i nhiu khong.

1 1
1. Xt x ; 0 . t g x, y x6 x 5 x 4 2y 2 x . Ta s chia c cc khong
9 10
1
l ; 0, 1 ; 0, 1; 0, 05 ; 0, 05; 0
9
4 1
2. Xt x 0; . t g x, y y 6 x 5 y 4 2x 2 x . Ta s chia c cc khong l
3 10
4
1; 1, 1 ; 1, 1; 1, 2 ; 1, 2; 1, 25 ; 1, 25; 1, 29 ; 1, 29; 1, 31 ; 1, 31; 1, 32 ; 1, 32; . Gh
3
cha!
Vy bi ton c gii quyt!
p dng lm bi sau: Chng minh rng: f x x 4 3x2 6x 1 0x 0, 2; 1,1
Th p dng cch lm trn lm cc bi sau
Chng minh rng:
1 1 4
1. f x x 6 x 5 x 4 2x 2 x 0x ;
10 9 3
8 5 2
2. f x x x x x 1 0x ( th dng DAC nh).
3. f x x5 x 4 x3 4x2 6x 1 0x ; 1
4. f x x7 x6 x5 2x 4 8x3 8x 2 10x 2 0x 1
5. f x x 4 3x2 6x 1 0x 0, 2; 1,1

Nguyn Minh Tun Trang 54


Tm ti sng to mt mt s cch gii phng trnh v t
II. PHNG TRNH V T.
1. bi.
Bi 1: Gii phng trnh: x 2 8x 16 x 3 3x 1 2 x 7x 2 0
3
Bi 2: Gii phng trnh: x2 x 1 x 1
2
x 2 x 6 2x 3 2x 2 9x 2

Bi 3: Gii phng trnh: x 5 7x 4 12x 3 x 2 3x 6 12 6 5x 0


20x x 1 20 x 1 9
Bi 4: Chng minh rng: f x 4 x 1 x 1 1 0x 1;
5x 1 2 9 5x 2 5
2
5x 5x 10 2x 6
Bi 5: Chng minh rng: f x x 2 5 0x 2;
x7 3 x2 2
Bi 6: Chng minh rng:
x1 x 1
f x 2 3 0x ;1
2 x 3x 1 2
1x 3 2x 1 3
3 2 3
1
Bi 7: Chng minh rng: x 4 2x 3 3x 2 6x 7 x 2 2x 5 x 1 4x 2 0x
2
x1 2x 1
Bi 8: Chng minh rng: f x
x2 3 1

x2 5 3
2 0x 3 ;
Bi 9: Chng minh rng: f x 4 x 4 1 x 2 3x 1 2x 10 v nghim.
x1 x 11 x3
Bi 10: Chng minh rng: f x
x2 1 1

x2 3 3 3 6
0x 3 ;

x1 2x 1 19
Bi 11: Chng minh rng: f x
x2 1 2

x2 2 1 7
0x 2 ;
4 x 1 3x 2 2 2 4
Bi 12: Chng minh rng: f x x 0x ;
2
x 2 3 2
x 4 8 5 2 5
x3 1 2x 1 1
Bi 13: Chng minh rng: f x 3 0x ;
8x 5 x 2 6x 2 x 1 3
Bi 14: Gii phng trnh: 9x 4 32x 2 5 18 4 3x 0
1 1 x2 1
Bi 15: Gii phng trnh:
2x 3 x4 x 2 2x 5 1
Bi 16: Chng minh rng:
f x x x 2 3x x 4 x 3 x 2 x 1 3x 6
3x 5 3x 4 3x 2 x 2 x 2 3 0x 1
4 x2 x 1
2x 1 1
Bi 17: Gii h phng trnh: y y2 3
3
x 3y 11 2 xy x 2 x 2
Bi 18: Gii phng trnh: 2 x 3 3x 1 x 1
3x 1 x 3 0
3x 1 6 1
Bi 19: Chng minh rng: f x x 2 x 4 0x ;
2
3x 2x 3 2 3x 1 2 3
Bi 20: Chng minh rng:
2 x2 x 1 4 x2 x 1
f x x 2 2x 0x 0
4 2 2 4 2 2
x x 4 x x2 x 20x 4 x 2x 2

Nguyn Minh Tun Trang 55


Tm ti sng to mt mt s cch gii phng trnh v t
Bi 21: Gii phng trnh: x 5 5x 4 2x 3 2x 2 4x 8 x 4 1 x 6 x 2 0


Bi 22: Gii phng trnh: 1 x x 2 1 x 2 x 1 1 x 2 x 2
3
a2 b2 c2
Bi 23: Cho 3 s a, b, c 0 . Chng minh rng: 3 a b c 2abc 11
3 3 3

3

Nguyn Minh Tun Trang 56


Tm ti sng to mt mt s cch gii phng trnh v t
2. HNG DN GII.
Bi 1: Gii phng trnh: x 2 8x 16 x 3 3x 1 2 x 7x 2 0
Nguyn Minh Tun
Gii
Bi ny c rt nhiu cch gii khc nhau nhng ta c lin hp ri chng minh v nghim
xem sao.
Ta c:
x 2 8x 16 x 3 3x 1 2 x 7x 2 0
x 4 x 1 x 3
3x 1 2 2 x
7x 2 3 0
3 x 3 x 1 7 2 x x 1
x 4 x 1 0
3x 1 2 7x 2 3
x 1

f x x 4 3 x 3 7 2 x 0
3x 1 2 7x 2 3
2
Nhim v l chng minh f x 0x .
7
3 x 3 0

Vi x 3 2 x 0 . Khi c:

3x 1 2 5
3 x 3 7 2 x 2x 29 7x 2 29x 157
f x x 4
5 7x 2 3 5 7x 2 3
157
1. Vi x f x 0 .
29
157 28x 3 21x 2 15225x 22967 157
2. Vi x th f x 0x
29 5 7x 2 3 2x 29 7x 2 29x 157 29

2 2 x 0
Vi x ; 2 ta c: . Khi ta c:
7 7x 2 3 7
3 x 3 6 3x 1 12 3 x 3 6 3x 1 21 3x
f x 6 0
3x 1 2 3x 1 2 3x 1 2
3 x 3 0

Vi x 2; 3 , ta c: 2 x 0 . Khi tng t nh trn ta c:

7x 2 3 7
3 x 3 6 3x 1 12 3 x 3 6 3x 1 21 3x
f x 6 0
3x 1 2 3x 1 2 3x 1 2
Vy bi ton c gii quyt hon ton!.
Nhn xt
Ngoi cch nh trn ta vn c th tinh nhm nh sau:
3 x 3 7 2 x 1 7 2 x 1 3 x 3
f x x 4 x x4
3x 1 2 7x 2 3 2 7x 2 3 2 3x 1 2


x 7x 2 4x 14

x 8 3x 1 x 25
2 7x 2 3 2 3x 1 2
Nguyn Minh Tun Trang 57
Tm ti sng to mt mt s cch gii phng trnh v t
Khi thch dng o hm hay dng bt phng trnh ph th ty, ti s dng o hm.
f x x 7x 2 4x 14
t .
g x x 8 3x 1 x 25
8 7x 2 21x 4 280 275968
- Ta c: f ' x 0 0x . f ' x i du t
2 7x 2 882
280 275968 280 275968
khi qua nn t cc tiu ti
882 882
280 275968
f x f 12, 99 0
882
- Chng minh tng t ta cng suy ra g x 0 .
Vy bi ton c gii quyt!

3
Bi 2: Gii phng trnh: x2 x 1 x 1
2
x 2 x 6 2x 3 2x 2 9x 2
on Ch Dng
Gii
Ta c:
3

x 1 x x 6 2x 2x 9x 2
x2 x 1 2 2 3 2

x x 1 x x 1 2 x 1 x x 6 3 2x 5 x x 3 0
2 2 2 2 2

x2 x 1 x2 1
x2 x 3 2x 5 0
2
x x12 x2 x 6 3
2 2
x x1 x 1
t f x 2x 5 .
2 2
x x1 2 x x6 3
x2 x 1
2 xa
x x12
Ch rng: .
x2 1
xb
x 2 x 6 3
x2 x 1 3
a lim x
x 2
x x12 2
Khi dng lim ta tm c
b lim x2 1 7
x
x 2
x x6 3 2
Ta c:
3 2 x x 1 3 2x x x 1
2 2
x2 x 1 3 x2 x 1
1. x 0
x2 x 1 2 2 2 x2 x 1 2
2 x2 x 1 2
7 2 x 1 7 2x x 1
2 2
x2 1 7 x2 1
2. x 0
x2 x 6 3 2

2 x2 x 6 3 2 x2 x 6 3
Vy bi ton c gii quyt!
Bi 3: Gii phng trnh: x 5 7x 4 12x 3 x 2 3x 6 12 6 5x 0
Nguyn Minh Tun
Gii

Nguyn Minh Tun Trang 58


Tm ti sng to mt mt s cch gii phng trnh v t
Ta c:
x 5 7x 4 12x 3 x 2 3x 6 12 6 5x 0
x 1 x 4 8x 3 20x 2 21x 18 12 6 5x 1 0
x 1
60
f x x 3 x 3 5x 2 5x 6 0
6 5x 1
thy:
6
1. Vi x ; x 0 3; f x 0 . Vi x 0 tha mn x 0 3 5x 0 2 5x 0 6 0
5
2. Vi x x0 ; 3 ta c:
60 300
- g x g ' x 2
0 g x g x0 9
6 5x 1 2 6 5x 6 5x 1
2
2 13 39
- Khi : f x x 2 4x 1 2 x 0
4 8
Vy bi ton c gii quyt!

Bi 4: Chng minh rng:


20x x 1 20 x 1 9
f x 4 x 1 x 1 1 0x 1;
5x 1 2 9 5x 2 5
Gii
thy:

9 20 x 1
1. Do x 1; nn 9 5x 2 4 5 x1
5 9 5x 2
20x x 1
2. Khi f x g x 4 x 1 x 1 5 x1 1
5x 1 2
2 2
x 1 1 2 5x 1 x1 5x 1 2 9
3. Li c: g x 2
0x 1;
5x 1 2 5

9
4. Nn f x 0x 1; ( pcm). Xong!
5
Hng dn
Bi ny nhn hnh thc kh khng b, trng c v ri rm nhng kh l n gin.
1. u tin ta thy c qu nhiu cn trong bi, iu ny lm ta ny ra tng nh gi
bt 1 cn i a v dng n gin hn.
2. Hai l bi ny l mt bi khng c cht cho lm vcc biu thc trong cn bc
nhtnn ny ra tng nh gi vi 1 hng s no .
3. Ba l thy trong bi c 2 phn thc nn s th nh gi mt em xem sao, ti s chn
ci th 2.
20 x 1
D thy 9 5x 2 4 5 x 1 . Lc ny bi ton ch cn 2 cn. Dng
9 5x 2
MODE 7 nhn thy f x 1 . iu ny chng khc no cho ta bit phng trnh:

Nguyn Minh Tun Trang 59


Tm ti sng to mt mt s cch gii phng trnh v t
20x x 1
4 x 1 x 1 5 x 1 0 c nghim kp. M may mn thay ta li rt
5x 1 2
1
c x 1 ra, lc ny ch l gii phng trnh 1 cn c nghim kp x (t tm nh)
4
v 1 nghim x 1 (khng quan tm) qu d dng, s dng lin hp ngc hoc chia cn
cho nhanh l s ra!
2

Ta c: 4 x 1 x1
20x x 1
5 x1

x 1 1 2 5x 1 x1
0
2
5x 1 2 5x 1 2
Vy bi ton c gii quyt!

Bi 5: Chng minh rng:


5x 2 5x 10 2x 6
f x x2 5 0x 2;
x7 3 x2 2
Gii
Hng lm vn nh bi trc, ta s nh gi 2 ci mu c: x 7 3 5 3 5
v x 2 2 2 .
5x 2 5x 10 2x 6
Khi : f x g x x 2 5 0
5 2
Nn f x g x 0x 2; (pcm). Xong!

Hng dn
Bi ny khng c ci g ni ht, trong cn cha a thc bc nht nn ta s nh gi vi
1 hng s no v l xong!
Ngoi ra nu thch DAC th c vic dng th nh! Ngoi cch ny ra cn c mt cch
khc.

2
5 x2 x 2 2 x 3
Ta c: f x x x 2 x 3
x 7 3 x2 2

5 2 9
x2 x 2 1 x 3 1 0x 1; 5
x7 3 x2 2
n y da vo iu kin t gii thch nh!

Bi 6: Chng minh rng:


x1 x 1
f x 2 3 0x ; 1
2 x 3x 1 2
1x 3 2x 1 3
3 2 3
Gii
Nhn thy
2 x 1 3
1. g x 1 x 3 nghch bin trn ; 1 nn g x g 1
3 2 2 6
3x 1 1 1 3
2. v x 2x 1 3 ng bin trn ; 1 nn v x v
3 2 2 6
x1 x
Khi f x 2 3 0
3 3
6 6

Nguyn Minh Tun Trang 60


Tm ti sng to mt mt s cch gii phng trnh v t
Vy ta c iu phi chng minh.

Hng dn
Bi ny khng c g phi bn c. Do di mu ang c cn cha a thc bc nht nn ta
s nh gi tng cn vi mt hng s, nu khng nh gi c nh vy th dng DAC
khng c g kh c.
trn ti trnh by hi tt, ng l phi c phn chng minh o hm mang 1 du na,
nhng thi thi gian c hn mong thng cm .

Bi 7: Chng minh rng:


1
x 4 2x 3 3x 2 6x 7 x 2 2x 5 x 1 4x 2 0x
2
Gii
1 9
2x 5 2 x 4
Vi x 2 ta c 2 b sau (t chng minh nh): .Khi ta c:
9
4x 2 x 8
4 5
2 2
9 16 6 1 4 427
f x x 2x x 2 x 9, 9 x 2 x x
4 3
0
10 5 5 2 5 50
1 2x 5 2
Vi x ; 2 ta c: . Khi :
2 4x 2 4
2
f x x 4 2x 3 3x 2 7 x 2 x 1 2x 2 7 0
Vy bi ton c gii quyt!
Nhn xt.
trn ti c nu ln 2 b m nhiu ngi c s chng hiu c kim u ra. Sau y
ti xin trnh by cc bc lm.
1. D kim tra thy f x 0 nn n s c gi tr nh nht, vic ca ta l tm c khi x
bng bao nhiu th t cc tiu. R rng nghim ca phng trnh f ' x 0 chnh l gi
tr cn tm do ta s gii phng trnh f ' x 0 , ta c:
3x 3 6x 4
f ' x 4x 3 6x 2 6x 6 0 x x 0 0, 3100436394
2x 5 4x 2
d
2. Vic lm tip theo l tm nhn t cha nghim x x 0 , ta s dng phm . Nhn t
dx
d
c dng
a dx 2x 5
2x 5 ax b . Vi
x x0 . Do kt qu l nn ta s tm mt
b 2x 5 a
0

s gn n v phi p v thay vo phng trnh u phi tha mn. Vi l do chn


1
a 2 9 8
. Tng t cho cn cn li ta c nhn t 4x 2 x .
b 9 4 5
4

Nguyn Minh Tun Trang 61


Tm ti sng to mt mt s cch gii phng trnh v t
1 9
2x 5 2 x 4
3. thy , m f x 0 nn cn chia lm 2 trng hp mi nh gi
4x 2 9 x 8
4 5
c . n ch vic nh gi phng trnh bc 4 l xong!.

x1 2x 1
Bi 8: Chng minh rng: f x
2
x2 5 3
x 3 1
2 0x 3 ;

Gii
2
4 3 28x 144x 189
Ta c: x2 3 x 0x 3 ;
3

2 6 6 x 2 3 8x 3

Khi :

f x
x1

2x 1
2
12 10x x 2 5 16x2 28x 33
4
3
3
x 1
2
x2 5 3 8x 3 x2 5 3
thy :
1. 12 10x x 2 5 16x 2 28x 33

2
5 23
2 x 2 5 x 1 16 x 2 5 x 2 33x 12
2 4
2
23 2 23 66 222
2. 16 x 2 5 x 33x 12 x 0x
4 4 23 23
3. Vy f x 0

Hng dn
u tin ta s tm im ri ca bi ton chnh l nghim ca o hm. Ta c:
x2 3 x 3 6 x 2 5 x 10
f ' x 2
2
0 x A 2, 677764402
2
x 3 2
x 3 1 2
x 5 2
x 53
d
Tm 2 nhn t cha im ri bng cch s dng phm . Ta s a v 1 cn v s
dx
dng nhn t lin quan ti x 2 3 v iu kin gn cht vi con cn ny. Nhn t c dng
nh gii phng trnh v t l x2 3 ax b .

d
+ Ta c
a dx f x
x A . p

dng vo bi ta c

b f x ax

d 4 4
a
dx

x2 3
x A

1.311... . Vn d c th ly em l v ta ang cn mt s
3 3
p tin bin i v y cng l bi ton cht nhng khng phi cht xt c do

Nguyn Minh Tun Trang 62


Tm ti sng to mt mt s cch gii phng trnh v t
13 59
f A 0, 188421028 . Ngoi nu thch lm cht na ta c th ly l a ;a ... i
10 4
vi bi cht hn na th ta cn phi lm nh vy ch cn bi ny th khng cn, ch vy l !
3 4 3

+ C tip b f x ax 1.528191379 . Lc ny nhn thy l x 2 3 x
2 3 2
ng chiu ta cn do bi ton cn chng minh ln hn 0 v t ang ln hn 0. Nu nhn t
cha ng nh ta cn th ta cn phi lm trn h s t do lm sao cho ng chiu du chng ta
cn.
2
n y mi ch i qua chng ng, cn mt phn na l chng minh phn sau cha
3
mt cn ln hn 0 bng SOS.
Tm im ri ca biu thc g x 12 10x x 2 5 16x 2 28x 33 . Ta c:
32x 28 x 2 5 20x 2 12x 50
g ' x 0 x x0 1, 961407277
x2 5
2
Nhn thy ta cn tm nhn t c dng x 2 5 ax b 2ax x 2 5 ... ng trc
2
ang c 10x x2 5 nn tm a tha mn khi ta ly g x x 2 5 ax b phi trit
34
tiu c 10x . Vi l do trn ta s chn a 5 b . T c nhn t l
5
2
2 34
x 5 5x . Nhng tuy nhin nhn t cha tha mn do thay vo b m v ta
5
nng h s trc cn ln 2. Vi cch lm tng t ta c nhn t s l
2
2 5
2 x 5 x 1 . n y khng cn g phi bn nh!
2

Bi 9: Chng minh rng: f x 4 x 4 1 x 2 3x 1 2x 10 v nghim.


Bi Th Vit
Gii
3 5 3 5
Dng Casio nhn thy f x 0x 5; v f x 0x ; .
2 2
3 5
Trng hp 1: Xt x 5; .
2
1. thy:


4 4 3 5
x 1 x 4 x 4 x 0x 5;
2
2
2 3 7 5 x 3 3 5
x 3x 1 x 0x 5;
2 2 3 7 2
4 x 2 3x 1 x
2 2

2x 10 1 x 7 1 2x 10 2 2 0x 5; 3 5
2 2 4

2


3 7 1 7
2. Do f x x x x 0
2 2 2 2

Nguyn Minh Tun Trang 63


Tm ti sng to mt mt s cch gii phng trnh v t
3 5
Trng hp 2: x ;
2
2x 2
1. thy 4
x 4 1 x2 1 0
4
x 4 1 x2 1 x 4 1 x2 1
2. Khi :

5x 10 2 x 2 3x 1 2x 10
f x x 2 1 x 2 3x 1 2x 10 0
x 2 1 x 2 3x 1 2x 10
Vy phng trnh f x 0 v nghim. (pcm). Xong!

Hng dn
Bi ny phng php ging ht vi bi 4. Ta cng s i tm im ri ca bi ton.
3 5 3 5
Do thy f x lun dng trn 5; v lun m trn ; nn ta s
2 2
chia lm 2 trng hp.
i vi trng hp 1 th khng c g phi bn, nhng trng hp 2 th c iu phi ch !
3 5
Do trn ; th ta s khng tm c nghim ca o hm hay lc ny hm
2
nghch bin, nn ta s khng i chng minh o hm m m s quy v nh gi 4 x 4 1
4 x 4 1 x
vi mt g x no . Ci ny hi kh nhng chu . Nu nh
2
x 1 x
4
x 4 1 x2 1 th thay vo thy tha mn. Vy ta s i chng minh
4
x 4 1 x2 1 , y khng phi 1 cu kh, ti cng chng minh ri. Khi ch vic
lin hp ln l c iu cn chng minh.

Bi 10: Chng minh rng:


x1 x3 x 11
f x
2
x 1 1

2
x 3 3

3 6
0x 3 ;
Gii
Trng hp 1: Xt x 3 ; 3 .
3
2 x
1 4
x 1 x 0x 3 ; 3
2 2 1
1. thy: x 1 x
2
2
2 3 x 2
x 3 2x 3 0x 3 ; 3
x 2 3 2x 3
2. Khi :

3 x x 11 2x 14x 9 x 1 8x 8x 9
2 2 2
x1
f x
x1
1
2
2x 3 6 6x x 2 1 1
3. tip:

Nguyn Minh Tun Trang 64


Tm ti sng to mt mt s cch gii phng trnh v t
+ 2x 2 14x 9 0x 3 ; 3 2x 2 14x 9 x 2 1 8x 2 8x 9 0
+ Nhn lin hp ta c:
4x 6 56x 5 172x 4 180x 3 105x 2 108x
2
4 21 7299
x x 4x 13 x 43x 172x 108 110 x
2 2
0
44 88
4. Nn 2x 2 14x 9 x 2 1 8x 2 8x 9 0
Trng hp 2: Xt x 3; .
1. Nhn thy x2 3 3 5
2
x1 3 x x 11 4x 37 x 1 34x 7
2. Khi f x
x2 1 1 5 3 6 30 x 2 1 1
34 x 2 1 8x 2 37x 4
3. t g x 4x 37 x 2 1 34x 7 g ' x .
x2 1
2
3 119
8 x
16 32
4. thy g ' x 0 nn g x ng bin trn 3 ; . Suy ra
2
x 1
g x g 3 15, 9430585 0 .
Vy f x 0x 3 ; .

x1 2x 1 19
Bi 11: Chng minh rng: f x
2
x 1 2

2
x 2 1

7
0x 2 ;
Bi Th Vit
Gii
2
D nhn thy x 1 x . Khi ta lun c:
x
g
2 2

f x
x1

2x 1

19

12x 31 x 2 14x 23x 17
x2 x 2 1 7
2
7 x 2 x2 2 1
thy:
2
75 22 523 2 37
1. g x x 2 2 x x 23x 31 x 2 2
11 25 275 11
0.261129x 2 1.621612x 2.521284
2. x 2 2 1.123x 0.722 0
x 2 2 1.123x 0.722
3. Khi :

523 2 37 523 2 37
x 23x 31 x 2 2 x 23x 31 1.123x 0.422 0
275 11 275 11
4. Vy bi ton c gii quyt!

Bi 12: Chng minh rng:


4 x 1 3x 2 2 2 4
f x x 0x ;
2
x 2 3 2
x 4 8 5 2 5
Nguyn Minh Tun

Nguyn Minh Tun Trang 65


Tm ti sng to mt mt s cch gii phng trnh v t
Gii
thy:
2139 2 9139
x 1.40448x
1.
19
x2 4 x
231
2500 15625 0x 2 ; 4
19 231
50 125 x2 4 x 2 5
50 125
gx

475x 2 24890x 11968 475x 2 8750x 12421 x 2 2
2. Khi f x

5 95x 2462 x 2 2 3
u x

1424x 3 17500x 2 14324x 17500
3. C g ' x 950x
24890

v x x2 2
2
min v x v 25561,75144
22 ; 45 2

4. Nhn thy
max u x u 4 25165, 00167 min v x
2 4
5 2 4
;
2 ; 5
2 5

2 4
5. Nn g x ng bin trn ; .
2 5
2 2 4
6. T c g x g 2.957051285 0 f x 0x ; .
2 2 5

Bi 13: Chng minh rng:


x3 1 2x 1 1
f x 3 0x ;
8x 5 x 2 6x 2 x 1 3
Nguyn Minh Tun
Gii
Trng hp 1: Xt x 1
x 3 5x 4 3 8x 5
1. Khi f x 0x 1;
8x 5 x 2
1 1
Trng hp 2: Xt x ;
3 2
8x 5 x 2 3

1. Khi 2
6x 2 x 1
3
2. Nn ta c:
3
1 1
2 18 7
x 1 3 2x 1
3 3
2x 18x 7 3 8 25
f x 3 0
3 2 6 6 162
1
Trng hp 3: Xt x ; 1
2

Nguyn Minh Tun Trang 66


Tm ti sng to mt mt s cch gii phng trnh v t
6x 2 x 1 5
1. Khi
8x 5 x 2 5.5
2 x 3 1 2x 1 10x 3 22x 144 1
2. Nn f x 3 0x ; 1
11 5 55 2
Vy bi ton c gii quyt!

V d 14: Gii phng trnh: 9x 4 32x 2 5 18 4 3x 0


L Duy Tin
Gii
Ta c:
9x 4 32x 2 5 18 4 3x 0
54
x 1 9x 3 9x 2 23x 23 0
4 3x 1
x 1
54
f x 9x 3 9x 2 23x 23 0 *
4 3x 1
Gii * :
23

1. D dng nhn thy vi x ; 1; th ta thy f x 0 .
3

23 1 19
2. Xt x ; 1 , ta c b sau: 4 3x x (bn c t chng minh).
3 2 5
3. Khi :
54 45x 4 477x 3 317x 2 1219x 1644
f x 9x 3 9x 2 23x 23 0
1 19 5x 48
x1
2 5
Vy PT x 1 .

Nhn xt
Cu ny mc d mu cha cn bc nht nhng ta khng th nh gi c do
lim 4 3x . V th ta s a v bi ton ging vi bi 4.
x

Khi ta o hm tm im ri th s tm c 3 nghim, nhiu bn s khng bit chn im


ri no cho ph hp. Vy khi gp nhng trng hp th ny th ta s th 3 nghim ca
f ' x vo, nghim no lm f x min th ta s chn im ri .
Th 2 vi vic tm nhn t, ta c im ri l x x0 1, 350561897 khi thay vo
1
4 3x x th s c kt qu l 3.512830189 v nu ai ly l 3.5 th bi ton s sai
2
1 7
do 4 3x x 0 s ngc chiu vi bi ton. Nu ly khong 3, 6; 3, 7 th vn
2 5
cha c do s b dng mt vi gi tr. Mt khc bi ny kh l lng nn ta s ly hn
19
ln v kim tra bng MODE 7 c th thy lun m v thay vo thy f x 0 cho nn
5
y l nhn t cn tm.
Tm ti sng to mt mt s cch gii phng trnh v t
Vic chng minh g x 45x 4 477x 3 317x 2 1219x 1644 0 nhn c v kh
nhng rt d, ta s tch nh sau: g x x 3 45x 477 317x 2 1219x 1644 , c
x 3 0

; 1 th
23
th thy rng vi x 45x 477 0 cho nn g x 0 .
3

317x 1219x 1644 0
2

V th l ht bi!

1 1 x2 1
Bi 15: Gii phng trnh:
2x 3 x4 x 2 2x 5 1
Gii
Do x 1 l nghim ca phng trnh nn ta tin hnh lin hp.

1 1 x2 1

2x 3 x4 x 2 2x 5 1
x 4 2x 3 x2 1

2x 3 x 4 x 2 2x 5 1

x1 1
x 1 0
x 2x 5 1

2

2x 3 x 4 2x 3 x 4

x 0
x1 1
f x
0


2
x 2x 5 1
2x 3 x 4 2x 3 x 4
2

Xt x 1; . Ta c: 2
x 2x 5 1
x 1 3
0 . Nn bi ton lun ng.
2
x 2x 5 1
3
Xt x ; 1 .
2
thy:
1. 2x 3 x 4 x 4 2x 3 2x 3 x 4 3
x 2 2x 5 2x 2 5x 3 x 4 3x 2
2. Khi f x
x 2 2x 5 1 2x 3 x4 3
3
3. x 2 2x 5 2x 2 5x 3 x 4 3x 2 2x 3 x 1 x 4
2

u x

3x 9 3 3 6 10
4. u' x 0 nn u x ng bin trn 2 ; 1 u x u 2 0
x4 4
5. Vy bi ton c gii quyt!
Tm ti sng to mt mt s cch gii phng trnh v t
Bi 16: Chng minh rng:
f x x x 2 3x x 4 x 3 x 2 x 1
3x 6
3x 5 3x 4 3x 2 x 2 x 2 3 0x 1
Din n k2pi.net
Gii


y x 2 y 3

t

4 3 2
z x x x x 1 z 5

Khi :
2
3 y x 2 z
f x xy 3xz 3x z 2 2
y 2
x 2
3 0
xy 3xz 3x z
2 2
y 2 x 2 3
thy:
x
g
8 7 6 5 4
x x x x x x2
1. x 2 x2 x 4 x3 x2 x 1
x 2 x2 x4 x3 x2 x 1
8 7 6 5 4
2. g x x 1 9 x 1 36 x 1 84 x 1 126 x 1
3 2
125 x 1 80 x 1 29 x 1 2 0
3. Nn g x 0x 1 . Vy du " " khng th xy ra.
4. Vy bi ton c gii quyt!

4 x2 x 1
2x 1 1
V d 17: Gii h phng trnh: y y2 3
3
x 3y 11 2 xy x 2 x 2
Gii
KX: 2x y 1 0; x 1; 1 ; 1 2x2 y 0
2

Ta c:
2x 1 y
1 f 2x 1 f y
2 2
2x 1 3 y 3
3
Xt f t lin tc trn . C f ' t 0t . Nn f t ng bin trn .
t 2
3 t2 3
Khi phng trnh f 2x 1 f y y 2x 1
3
Th vo phng trnh 2 ta c: 6x 4 8x 3 2 2x 2
2
3
t f x 6x 4 8x 3 2 2x 2
2
thy:
2
25 3
1. f x 5 2x 2 2 5x 4 8x 3 x 2 2x 2 2 x 2
4 2
2
3
g x 2x 2 2 x 2
2

Nguyn Minh Tun Trang 69


Tm ti sng to mt mt s cch gii phng trnh v t
10 1
2. g ' x x 20x 2 24x 2 x.h x .
2
2x 2 2x 2 2
120x 3 96x 2 84x 48 v x
3. h ' x
2x 2 2 2x 2 2
2
4 292
4. v' x 360 x 0 . Suy ra phng trnh h ' x 0 c ti a 1 nghim l
15 5

6784 2338848 3 6784 2338848


3
4 125 625 125 625
x x0
15 6
min h x h x 0 1.31 0 . Khi phng trnh g ' x 0 x 0 h x 0 .

lim g x

Mt khc ta li c: 25 20 2 g x 0x f x 0 .
g 0 0
4
Vy bi ton c gii quyt!

V d 18: Gii phng trnh: 2 x 33x 1 x 1 3x 1 x 3 0


Gii
Nhn thy phng trnh ang c 3 cn nn c th t a x gim bt cng knh ca
phng trnh.
t a x a 0 phng trnh tr thnh

2 3a 4 10a 2 3 a 1
3a 2 1 a 2 3 0 .

3a 2 1 3 a 1
Ta tm c nghim kp a 1 nn s tm c 2 biu thc lin hp l 2 2 . Khi
2 1 3
a 3 a
2 2
tin hnh nhn lin hp ta c:
2 3a 4 10a 2 3 a 1
3a 2 1 a 2 3 0
1 3
2 3a 4 10a 2 3 2a 2 4a 2 a 1 3a 2 1 a a 1 a 2 3 a 0
3 1
2 2 2 2
8 a 1 a 2 1 3 a 1a 1 3 a 1a 1
2 2 2

0
2 3a 4 10a 2 3 2a 2 4a 2 3 1 1 3
4 3a 2 1 a 4 a 2 3 a
2 2 2 2
a 1

8 a 2 1 3 a 1 3 a 1

0 *
2 3a 4 10a 2 3 2a 2 4a 2 3 1 1 3
4 3a 2 1 a 4 a 2 3 a
2 2 2 2
Nhim v l s i chng minh phng trnh * v nghim.
thy:
Tm ti sng to mt mt s cch gii phng trnh v t
1 3 a 1 48a 2
1. 0 a 0
2 4 3a2 1 6a 2

4 3a 1 4 8 3a 1 12a 4
2 2

1 3 a 1 48
2. 0 a 0
2 4 a 2 3 2a 6

4 a 2 3 4a 8 a2 3 4a 12
3. Cn li ta c:
8 a 2 1
1
2 3a 4 10a 2 3 2a 2 4a 2
24a 2 24a 1
2

0

2 3a 4 10a 2 3 2a 2 4a 2 6a 2 4a 6 2 3a 4 10a 2 3
Nn do phng trnh * v nghim.
Vy phng trnh c nghim x 1
Bi 19: Chng minh rng:
3x 1 6 1
f x x 2 x 4 0 x ;
3x 2 2x 3 2 3x 1 2 3

Gii
Cch 1:
Ta c:
3x 2 2x 3 ' 0
6x 2
2 3x 2 2x 3
0x
1
3
. Suy ra

3
3x 2 2x 3 .
2
Khi :

f x x x 4
2 6x 2

6

7x 2 13x 26 3x 1 14x 2 26x 10
7 3x 1 2 7 3x 1 2
thy:
14x 2 26x 10 0
1
1. Vi x ; 0 th
2
2 . Do c iu phi
3
7x 13x 26 7 x 559 0
13

14 28

chng minh.

2. Vi x 0 th:
13
2
501
21 x
14 28
7x 2
13x 26 3x 1 7x 2 13x 26 f x 0
7 3x 1 2
Vy bi ton c gii quyt hon ton!.

Bi 20: Chng minh rng:


Tm ti sng to mt mt s cch gii phng trnh v t
2 x2 x 1 4 x2 x 1
f x x 2 2x 0x 0
4 2 2 4 2 2
x x 4 x x2 x 20x 4 x 2x 2
Nguyn Minh Tun
Gii
x 4 20x 2 4 8
1 5 4 2
Vi x 0; x x 4 3
2 2
x x 1 0
Khi :
2 x2 x 1 4 x2 x 1
f x 2 x 2 2x
x2 x 5 x 2x 10
2 2 2
1 7 5 15
x 4 x x 4 12x 2 x x 2 38 x 620
2 6 3 19
0
x x 5 x 2x 10
2 2


x 2 x 1 0

1 5 14
Vi x x 4 x2 4
2 5
4 2 2 11
x 20x 4 x 5x 4
Khi :
2 x2 x 1 4 x2 x 1
f x x 2 2x
24 3
x2 x 2x 2 7x
5 4
6 5 4 3 2
40 x 1 260 x 1 537 x 1 761 x 1 389 x 1 281 x 1 690
0
5x 2
5x 24 8x 2 28x 3
Vy bi ton c gii quyt!
Nhn xt
11
Nhiu bn s t ra cu hi v sao li nh gi c x 4 20x 2 4 x 2 5x . n
4
gin v:
1 5
1. Ta ang cn a v chng minh x 4 20x2 4 x2 ax b vi x a v
2
phng trnh a thc. Li c a 2 20 nn s ly 5.
1 5
2. Do hm ang nghch bin nn im ri s l x . C th kim tra bng Mode 7
2
hoc c th ca hm y f x nh sau:

Nguyn Minh Tun Trang 72


Tm ti sng to mt mt s cch gii phng trnh v t
8

15 10 5 5 10 15

A
2

10

12

14
y = f(x)

1 5 1 5
C th thy im A
2
; f
2 ang nm phn i xung ca th nn f x

11
s nghch bin. Do nhn t l x 4 20x 2 4 x 2 5x . Phn cn li khng phi
4
ni nhiu!

Bi 21: Gii phng trnh: x 5 5x 4 2x 3 2x 2 4x 8 x 4 1 x6 x 2 0


Nguyn Minh Tun
Gii
Ta c:
x 5 5x 4 2x 3 2x 2 4x 8 x 4 1 x6 x 2 0

x 1 x 2 x 3 2 x 4 1
x6 x 2 2x 4 0

x 1 x 2 x 3 2
x 4
1 x 1 x 2 x 4 x 3 3x 2 5x 7
0
x6 x 2 2x 4

3
x 1 x 2 x 2
x 4 1 x 4 x 3 3x 2 5x 7
0
x 6 x 2 2x 4
x 1 x 2 0 x 1 x 2

3 x 4 1 x4 x3 3x2 5x 7 0
f x x 2
x6 x 2 2x 4

1 1
t g x x6 x 2 g ' x 6x 5 1 0 x 5 . g ' x i du khi qua 5 nn
6 6
1 1
g x t cc tiu ti 5 g x g 5 1, 41 0 .
6 6
t h x x6 x 2 2x 4 .
Tm ti sng to mt mt s cch gii phng trnh v t
1. Vi x 2 h x 0 .
2. Vi x 2 x 6 x 2 x6 x 3 h x x x 2 2 0
Khi c:
x 3 2 x 3 2 2x 4 x 4 x 3 3x 2 5x 7
f x
x6 x 2 2x 4
2 9 2 83
3 x 2 x 0,75 x 1, 5
2 16 0
6
x x 2 2x 4
Vy bi ton c gii quyt!

Bi 22: Gii phng trnh: 1 x x 2 1 x 2 x 1 1 x 2 x 2


thi th THPT Quc Gia 2016 Anh Sn 2 Ngh An.
Gii
Ta c:

1 x x2 1 x2 x 1 1 x2 x 2
1 x2 x 1 x x2 1 x2 x 1 x2 x 2 0
x 2 x x 1 2x2 x 2
0
1 x2 x 1 x x2 1 x2 x 1 x2 x 2
2x 2 x 2 x
x 1 0
2 2 2 2
x x 1 x x 1 x x 2 1 x x 1
21 2 3

thy: 1 x x 2 1 x 2 x 1 1 x 2 x 2
4
0x0
Khi ta c:
2x 2 x 2
f x 1
x x 2 1 x 2 x 1 x 2 x 2

2x 2 x 2 x x 2 1 x 2
x 1 x 2 x 2

x x2 1 x 2
x 1 x 2 x 2

2x 2 x 2 x x 2 1 x 2
x 1 x 2 x 2
x x2 1 x 2
x 1 x 2 x 2

2x 2 2x 3 2 x 2
x 1 x 2 x 2
g x
x x2 1 x2 x 1 x2 x 2
1
g x 0
2x 2 2x 3 2
x2 x 1 x2 x 2 x x2 1 x2 x 1 x2 x 2
Li c:
x x2 x 1 1 x
1. 1
1 x2 x 1 1 x2 x 1
2. Xt bt phng trnh: x2 x 1 1 x 0
- Vi x 1 bt phng trnh lun ng.

Nguyn Minh Tun Trang 74


Tm ti sng to mt mt s cch gii phng trnh v t
Vi x 1 ta c: x2 x 1 1 x 0 x 0 lun ng.
-
x
3. Do 1 0.
2
1 x x1
Vy bi ton c gii quyt!

3
a 2 b2 c2
Bi 23: Cho 3 s a, b, c 0 . Chng minh rng: 3 a b c 2abc 11
3 3 3

3

Chng minh
3
a 2 b2 c2
t f a, b, c 3 a b c 2abc 11
3 3 2
. Do f a, b, c l hm thun nht bc
3
3, nn khng mt tnh tng qut ta chun ha a b c 3 v gi s a min a, b, c . Khi
2 2 2

ta cn chng minh f a, b, c 3 a 3 b 3 c 3 2abc 11 0


3
b2 c2
b2 c2 b 2 c2
t f a, , 3a 2
3
a b 2 c 2 11 . Ta c:
2 2 2

2
b c 2
b c 1
2 2
f a.b.c f a,

2
,
2 2

3 2 b3 c3 b2 c2 2 b2 c2 a b c
2


1
2
3 2 b 2 c 2 b c b c 2 b 2 c 2 2bc a b c
2

1
. b c
2
3 b c 2 b 2 c 2 2bc

a


2 2 b c b c
2 2

a 0; 1
Do a min a, b, c nn . Khi ta cn phi chng minh c:
b c 2

g a, b, c

3 b c 2 b 2 c 2 2bc a 0
2 b 2 c2 b c


3 b c 2 b 2 c 2 2bc a 3 b c 2 b2 c 2 2bc 1
2 b 2 c2 b c 2 b 2 c2 b c

3b 3c 1 2 b 2 c2 6bc b c 5 2 b 2 c 2 b c 6bc

2 b2 c2 b c 2 b2 c2 b c
2
b c 6bc
2 b 2 c 2 b c 6bc 2 b2 c2 b c
0
2 b2 c2 b c 2 b2 c2 b c
b2 c2 b2 c2 3 a 2 3 a2
Vy f a, b, c f a, , f a, ,
2 2 2 2

Nguyn Minh Tun Trang 75


Tm ti sng to mt mt s cch gii phng trnh v t
3
2
3 a2 3 a2 3 a 3 a2
Ta c: f a, , 3 a3 2 2a. 11
2 2 2 2

3
3 a2 3 a2
f a, t, t 0 3 a 3 2 2a. 11 0
2 2

3 3

3a 3
3 3 a2 a 3 a 2 11 0
3 3 a2 2
9a 15 2a 4 a 1 0
2 2

2

a 1 2a 4
3 a 4
2a 3
6a 2
14a 23 0 *
3



2 3 a 2 5 3a 2

3
t g a 2a 4 2 3 a 2 2 5 3a 3 a 4 2a 3 6a 2 14a 23


2a 3 4a 2 6a 12 6 2a 2 3a 4 6a3 6a2 38a 29
2
61 22 61 22 1734 2 2684 29
Ta c b : 6 2a 2 a 6 2a 2 a a a 0 .
25 25 25 25 625 625 625
D thy b ng vi a 0; 1 .
Mt khc 2a 3 4a 2 6a 12 2a 1 a a 3 12 0a 0;1 , nn ta c:
61 22
g a 2a 3 4a2 6a 12 a 3a 4 6a 3 6a 2 38a 29
25 25
31a 184a 226a 1052a 7 31a 4 184a 3 215a 2 441a 2 1052a 7
4 3 2

25 25
Do

31a 4 184a 3 215a 2 0a 0; 1


3 a 4 2a 3 6a 2 14a 23
441a 1052a 7 0a 0; 1 g a 0 2a 4
2
3
0

3
2 3 a 2 5 3a 2 0

2

3 a 2
5 3a 2



Vy * lun ng f a, t, t 0 f a, b, c 0
Du = xy ra khi a b c 1
By gi vn t ra l kim u ra ci b kia. lm c iu ny ta s dng tip tuyn
tm ra n di s gip ca CASIO. Nhng tuy nhin lm sao ta c th nh gi c
6 2a 2 ax b khi cha c manh mi g? gii quyt vn ny ta s tm im ri ca
bt ng thc ang cn chng minh. Ta c:
16a 4 24a 3 60a 2 72a 36
g 'a 12a 3 18a 2 12a 38
2
6 2a

Phng trnh g ' a 0 c 1 nghim l :


X A 0, 919459592
By gi ta cn nh gi c biu thc trc cn l
2a 3 4a 2 6a 12 . D thy rng:
2a 3 4a 2 6a 12 a 1 a a 3 12 0
Tm ti sng to mt mt s cch gii phng trnh v t
d
Vy by gi cn chng minh 6 2a 2 ax b . Ta s s dng phm .
dx
d
Ta c a
dx

6 2X 2
X A

0, 8858596433 . By gi ta s ly a lm sao cho bt ng thc

khng b nh gi qu tri v va ng thi l mt s hu t p va v sau d nh gi hn.


22
Do mnh s ly a 0, 88 . By gi cn tm b ,do ang cn chng minh trn on 0; 1
25
22
nn ta s dng MODE 7 tm ra b . MODE 7 vi hm F X 6 2X 2 X trn 0;1 ta
25
c:
22
thy l 6 2x 2 x 2, 449489742 . By
25
gi ta s chn b l mt s hu t va p va gn s
61
2, 449 nht. Ta s chn s b 2, 44 , ng thi
25
thay vo bi ton ta thy nhn t ny tha mn. Vy
22 61
ta tm c nhn t l 6 2a 2 a .
25 25
Ch rng chc my bn khi tm c a th thay lun
X A vo tm ra b v tm c b 2, 884982846
, hin nhin l nhn t ny l sai. i vi bi m
chng minh trn on th ta thng s dng MODE 7
tm ra b, cn nhng bi chng minh trn khong
th ta mi dng cch thay trc tip nhn t vo, nu c thi gian hy th chng minh nhng bi
sau th s r. Ngoi ra nu gii phng trnh o hm bng 0 m nhiu nghim th ta chn nghim
no lm phng trnh u min max

-------------------------------------------------------------- Ht ---------------------------------------------------------------
D. K THUT S DNG TNH N IU.
I. Kin thc cn nh.
nh l 1.
Xt phng trnh f x a . Nu f x xc nh, lin tc v n iu trn TX ca n th phng
trnh f x a c ti a 1 nghim.
nh l 2.
Cho phng trnh f x g x , trong f x ,g x cng xc nh trn D ng thi c tnh n
iu ngc nhau trn D th phng trnh f x g x c ti a 1 nghim.
nh l 3.
Nu hm s f x n iu, khng lin tc trn tp xc nh ca n th phng trnh f x a c
ti a n 1 nghim, vi n l s im gin on ca th hm s.
Tm ti sng to mt mt s cch gii phng trnh v t
II. Bi ton minh ha.
Ni dung ch yu ca phng php ny l ta s chng minh o hm ca hm s ban u mang
mt du ch ra nghim duy nht ca phng trnh u. hiu r hn ta cng i vo cc bi c
th.
Bi 1: Gii phng trnh: x 3 x 2 x 3 4 x 1 3
Phn tch
u tin nh hng hng gii ta s dng my tnh kim tra tnh n iu ca hm s
f x x 3 x 2 x 3 3 4 x 1 bng MODE 7.
Nhp vo my hm s trn ri cho:
START 1
END 20
STEP 1
Ta c bng nh bn. Nhn vo bng ta thy hm c 1
nghim duy nht l x 0 v c v nh ang ng bin
trn 1; cho nn ta c x 0 l nghim duy nht
ca phng trnh.
Bng cc gi tr ca TABLE ta c th v c th ca
hm s nh bn.
Nhn vo th ta c th thy n lun lin tc, ng bin
v ct trc honh ti 1 im duy nht.
Nu mun kim tra k hn ta c th dng my tnh chia
nh khong ra kim chng iu ny.

UA
Bng cc gi tr ca TABLE ta c th v c 8

th ca hm s nh bn.
Nhn vo th ta c th thy n lun lin tc, 6

ng bin v ct trc honh ti 1 im duy nht. 4

Vy khi ta ch cn chng minh o hm ca


hm f x x 3 x 2 x 3 3 4 x 1 ln hn 0
2

l ta c th gii quyt c bi ton ny. 10 5 5 10

Ta c li gii nh sau:
2

Li gii.
KX: x 1; . 4

t f x x 3 x 2 x 3 3 4 x 1 lin tc trn 1; .
3
Ta c: f ' x 3x 2 2x 1 3
0x 1; . Do f x ng bin trn 1; .
4 4
x1
Suy ra phng trnh f x 0 c ti a 1 nghim. Li c f 0 0 nn x 0 l nghim duy nht
ca phng trnh.

Bi 2: Gii phng trnh: 4x 1


x 3 3 3x 5 4x 8
Phn tch

u tin nh hng hng gii ta s dng my tnh kim tra tnh n iu ca hm s


f x 4x 1
x 3 3 3x 5 4x 8 bng MODE 7.
Tm ti sng to mt mt s cch gii phng trnh v t
Nhp vo my hm s trn ri cho:
START 3
END 20
STEP 1
Ta c bng nh hnh bn. Nhn vo bng ta d
thy rng phng trnh u c 2 nghim
x 2; x 1 v khng phi hm n iu.
n y ch c 2 cch l o hm f x ln v xt
du ca f ' x sau ch ra 2 nghim trn 2
khong m ta xt tnh n iu ca n. Cch
th 2 l s dng nh l 2 gii quyt n.

d hnh dung ta c th v th nh sau:


Nhn vo th ta thy f ' x b i du khi qua
4

mt im. gii quyt bi ton bng cch 1 th


kh l kh v: 10 5 5 10 15 20

f ' x 4 x 3 3 3x 5 4

1 1 4
4x 1 8

2 x 3 2


3

3x 5
10

12

Rt kh xt du ca n. Ch cn cch l chng
minh v nghim trn 1 khong cha im ri, cn
14

2 khong cn li ta d dng xt du ca n. Ta s 16

th lm 2 cch. 18

Gii
Cch 1: Hm s.

t f x 4x 1 x 3 3 3x 5 4x 8 lin tc trn 3;

1 1 4
3

Ta c: f ' x 4 x 3 3x 5 4x 1
2 x 3
2


3
3x 5
n y tm khong cha im ri ta s dng CASIO tm n.
Ta c: f ' x 0 x 1, 138426746
n y ta s chng minh phng trnh u v nghim trong
3
khong ; 1 . Ta c:
2
3

Vi x ; 1 th 4x 1 0 , x 3 3 3x 5 0 VT 0 .
2
3 3
Mt khc 8x 4 0x ; 1 nn phng trnh v nghim trn ; 1
2 2
1 1 1 2
t g x 2
g ' x 3
5
0x 1
2 x3 3

3x 5 4 x3 3

3x 5
1
Vi x 1; th 4x 1 0 , g x g 1 1 . Khi ta c:
4
Tm ti sng to mt mt s cch gii phng trnh v t
f ' x 4
x 3 3 3x 5 4x 5 4
1 3 3 3 5 5 4 0
1 1
f x ng bin trn 1; f 1 f x f 18 f x 9 0 . Vy phng trnh
4 4
1
v nghim trn 1; .
4
1

Vi x th 4x 1 0 f ' x 4 x 3 3 3x 5 4 10 0
4

1
f x ng bin trn ; phng trnh f x 0 c ti a 1 nghim. Li c f 1 0
4
nn phng trnh c 1 nghim l x 1 .

4x 1 0
3
Vi x 3; th 2 x 3 2, 5 . Khi ta c:
2 3 2

3x 5 2, 5
4 4x 1

f ' x 4 x 3 3 3x 5 5
4

4
x 3 3 3x 5 9, 6 4
1, 5 3 3 3. 1, 5 5 9, 6 0
3
f x nghch bin trn 3; , do phng trnh f x 0 c ti a 1 nghim thuc
2
3
3; 2 . Li c f 2 0 nn x 2 l 1 nghim ca phng trnh u.
Kt lun: Vy phng trnh c 2 nghim l x 2 v x 1 .
Cch 2: Hm s.
1 1
Do x khng l nghim ca phng trnh nn xt x th phng trnh tng ng:
4 4
4x 8

4x 1 x 3 3 3x 5 4x 8 x 3 3 3x 5
4x 1
4x 8 1 1
t f x x 3 3 3x 5 lin tc trn 3; v ; .
4x 1 4 4
1 1 36 1
C: f ' x 2
2
0 . Suy ra f x ng bin trn 3; v
4x 1 4
2 x3 3
3x 5

1
; . Vy phng trnh f x 0 c ti a 1 nghim trn mi khong trn.
4
T ch ra 2 nghim l x 2 v x 1 l xong bi!
Ta c th thy rng cch 1 c v l di v phc tp hn so vi cch 2 nhng tuy nhin s c nhiu
bi li nhanh hn, sau y l mt v d.

Bi 3: Gii phng trnh: 9x 4 32x 2 5 18 4 3x 0


Thy L Duy Tin - THPT Bnh Minh
thi chn hc sinh gii THPT Bnh Minh kha 2015 2018
Gii
Vn nh nhng bi trc ta s dng MODE 7 kho st v v th ca hm s.
MODE 7 vi hm f x 9x 4 32x 2 5 18 4 3x trn 15; 2 . Ta c:

Nguyn Minh Tun Trang 81


Tm ti sng to mt mt s cch gii phng trnh v t
Nhn vo bng th ta thy hm c v nghch
bin, nhng nu kho st k hn th ta thy rng
n s tng trn khong 1; 3; 1, 1 do hm
4
ny khng h n iu mt chiu trn ; .
3
Ta c th v th ca n nh sau:
Hnh bn l nhnh pha trn ca th. Ta c
th thy rng n khng h n iu mt chiu
v c ti 2 im i chiu. Do gii c
bi ny ta cn phi chng minh n v nghim 46

trn mt khong no . D thy l ; 0 . 44

Xt x ; 0 ta c 4 3x 2 , do 42

40

f x 9x 4 32x 2 5 18 4 3x 38

4 2
9x 32x 36 36

2 34

16 68
9 x2 0 32

9 9 30

Vy phng trnh v nghim trn ; 0 . 28

4
26

Xt x 0; , ta c: 24

27 4
f ' x 36x 3 64x . D thy 36x 3 64x x 36x 2 64 0x 0; nn f ' x 0 .
4 3x 3
4 4
Suy ra f x nghch bin trn 0; , do phng trnh f x 0 c ti a 1 nghim trn 0; .
3 3
Li c f 1 0 nn x 1 l nghim duy nht ca phng trnh.
Ngoi ra nu ai thch cng c th lin hp gii quyt bi ton ny. Ta c:
9x 4 32x 2 5 18 4 3x 0
54
x 1 9x 3 9x 2 23x 23 0
4 3x 1
x 1
54
f x 9x 3 9x 2 23x 23 0 *
4 3x 1
Do ta chng minh phng trnh v nghim trn ; 0 nn ta ch cn chng minh
4 4
f x 0x 0; . iu ny qu l d bi v x 1 9x 2 23 0x 0; do phng
3 3
4
trnh f x 0 s v nghim trn 0; . Bi ton c gii quyt!
3

2
x2 2 109
Bi 4: Gii phng trnh: x 2 2 3x 0
3 3 81
thi th THPT Quc Gia Ph Yn 2015.
Phn tch
Tm ti sng to mt mt s cch gii phng trnh v t
2
X2 2 109
MODE 7 vi hm: F X X 2 2 3X trn 15; 1 ta c:
3 3 81
Nhn vo bng th ta s thy hm c v nghch bin,
2
kho st k hn khong 0; ta s thy phng
3
2
trnh c nghim thuc khong 0; . Gii phng
3
1
trnh ta s c nghim x , do ta s dng
3
phng php hm s gii n. Ta c th v th
ca hm nh hnh bn. Nhn vo th ta thy rng
hm s f x chc chn nghch bin, nn ta s i 8

chng minh iu ny. 6

3
Ta c: f ' x 4x 3 2x . 4

2 2 3x
2 2 3
2

Nu x ; 0; th 4x 2x 0 ,
2 2
10 5 5 10

khi f ' x 0 .
2

2
Nu x ; 0 th 2 2 3x 5 , suy ra:
2
6


3 6 2 3
f ' x 4x 3 2x
g x . C g ' x 0 x . Mt khc ta li c g

5 6 2 5
3 6 27 10 5 2
g 0 v g 0 , nn f ' x g x 0x ; 0 .
5 6 45 2
2 1
Vy f x nghch bin trn ; , do x l nghim duy nht ca phng trnh.
3 3
Bi 5: Gii phng trnh: 2x 4 4x 1 x 3 x 1 3x 3 1 0
Phn tch
Bi ny l mt bi kh l kh, nhng trc ht ta c phi kho st hm s v v th trc .
MODE 7 vi hm F X 2X 4 4X 1 X 3 X 1 3X 3 1 trn 1; 5 ta c.
Nhn vo bng thy rng hm c v nghch
1
bin, nhng kho st k hn trn 3 ; 1 ta
3
s thy hm lc tng lc gim, do nh nhng
bi trn mnh s li phi chng minh v nghim
trn mt khong no ri! D dng d bng
CASIO th ta thy hm s nghch bin trn
9
khong ; , do ta s chng minh
5
1 9
phng trnh v nghim trn 3 ; .
3 5
Trc ht ta v c th nh bn.
Nhn vo th ta c th chc chn rng khi
Tm ti sng to mt mt s cch gii phng trnh v t
9
x ; th hm nghch bin v ct trc 6

5
honh ti duy nht mt im ti x 2 . Sau khi
4

nh hng c hng gii ri ta s i gii 2

quyt n thi! Nhng ta li gp phi mt vn


l biu thc pha trc cn thc l mt a thc 10 5 5 10

bc 3 kh l kh xt du ca n c th kh 2

cn nn ta s chia lm 2 trng hp.


4

x 3 x 1 0
34 10 9
Trng hp 1: Nu 1 9 . Khi 3x 3 1 . Suy ra c:
x 3 ; 25 2
3 5
f x 2x 4 4x 1 x 3 x 1 3x 3 1
9 3 4x 4 9x 3 x 11
2x 4 4x 1
2
x x 1
2
2
9 9 171 2 71 194
4 x2 x x x
8 10 80 10 25
0
2
x 3 x 1 0

Vy phng trnh v nghim khi 9 .
x
5
x 3 x 1 0
3
Trng hp 2: Nu 1 9 . n y nu ta cho 3x 1 0 th khng hn f x 0
x 3 ;
3 5
Do ta cn nh gi 3x3 1 ax b . n y s dng tip tuyn v im ri ca f x .

27x 5 15x 3 3x 2 2 x 0, 678262158 A


Ta c: f ' x 8x 3 4 0 .
2 3x 3 1 x 1, 581810582 B
Ta thy f A 0, 6112465178; f B 2, 256666123 do x A l im ri ca f x .
d 3
Khi a
dx
3x 3 1
x A 2

. Ta s dng MODE 7 tm b .

3 9
MODE 7 vi hm f X 3X 3 1 X ta s tm c min xp x 0, 36 . Vy nh gi cn
2 25
3 9
tm l 3x 3 1 x 1
2 25
1 6
Nu x 3 ; th 1 lun ng.
3 25
6 9 9 27 544
Nu x ; th 1 3x 3 x 2 x 0 g x 0
25 5 4 25 625
Tm ti sng to mt mt s cch gii phng trnh v t
9 27 5 73
- g ' x 9x 2 x 0x
2 25 20
6 9 22831 5 73 5 73
- g 0, 95 0;g 0;g 0, 0388 0 g 0, 97 0
25 5 2500 20 20
Vy 1 lun ng.
3 9
Khi f x 2x 4 4x 1 x 3 x 1 x
2 25
2 2
9 19 107 7863
4 3 2 25x x x 2 71 x
2

25x 18x 75x 107x 68 25 25 142 284
0
25 25
1 9
Vy phng trnh v nghim trn 3 ; .
3 5
9
Trng hp 3: Nu x .
5
Ta c:
27x 5 15x 3 3x 2 2 2 8x 3 4 3x 3 1 27x 5 15x 3 3x 2 2
f ' x 8x 3 4
2 3x 3 1 2 3x 3 1
9
thy: 8x 3 4 0x
5
5 3 2 5 4 3 2
27x 15x 3x 2 27 x 1 135 x 1 255 x 1 222 x 1 84 x 1 7 0
Khi ta ch cn i chng minh 3x 3 1 nh hn mt biu thc no .
3
Nu nh gi 3x3 1 ax b th khng kh thi do deg
3x 3 1
2
- deg l bc ca a

thc ln hn 1. Ta cn phi nh gi 3x3 1 x2 ax b ( y ta chn lun h s ca x 2 l


1 cho d nh gi), nhng ta li phi i tm im ri ca f ' x . Nhng hy khi ta d MODE
7 th dng nh f ' x nghch bin. kim chng ta s kho st tht k v v th hm ny.
Nhn vo th c th thy rng khi 6

9
x 1, 8 th hm f ' x nghch bin. Vy 4

5
9
2

im ri s l x .
5 15 10 5 5 10 15

Khi ta s tm c: 2

d
dx
3x 3 1
x
9
1
4

5
6

b
d 5 8

dx
x2 9 10
x
5
1 3
12

c 3x 3 1 x 2 x 14

5 x 9 2
5 16

1 3
Vy c nh gi 3x 3 1 x 2 x *
5 2
By gi s i chng minh * lun ng.
Ta c:
Tm ti sng to mt mt s cch gii phng trnh v t
1 3
3x 3 1 x 2 x
5 2
2
17 3 76 2 3 5 17 3 2 13
x4 x x x 0 x2 x x 2 0
5 25 5 4 10 20 20
Vy * lun ng.
Khi f ' x 55x 5 16x 4 195x 3 25x 2 8x 50 0 .
9
Suy ra f x nghch bin trn ; . Vy x 2 l nghim duy nht ca phng trnh.
5
5 x 3
Bi 6: Gii phng trnh: x 1 2 4 x 2
2x 18
thi th i hc Chuyn Nguyn Tri Hi Dng 2013
Phn tch
5 x 3
MODE 7 vi hm f x x 1 2 4 x 2 ta c.
2x 18
Nhn vo bng ta thy hm c v ng bin v c
nghim l x 3 . kim chng iu ny ta s v
th ca hm ny.
Nhn vo th trm phn trm l hm ng bin
nn ta s i chng minh iu ny.
5 x 3 6

Ta c: f x x 1 2 4 x 2 lin tc
2x 18 4

trn 1; 4 . 2

2
1 1 10x 60x 90
f ' x 2
By 10 5 5 10

2 x1 4x 2x2 18 2

gi ta cn chng minh f ' x 0x 1; 4 . 4

chng minh iu ny ta cn dng my tnh 6

kho st tm min, max ca f ' x trn 1; 4 .


8

10x 2 60x 90 1
Ta c 2
v
2x 2
18 2
1 1 1
. Ta s i chng minh 2 iu
2 x1 4x 2
ny.
Ta c:
1 1 52 x1 21 4x
1. 0
2 x1 5 10 x 1 10 x 1 5 2 x 1
1 3 10 3 4 x 9x 64
2. 0
4 x 10 10 4 x 10 10 3 4 x 4x
2
15 2412
2 4 2 4x 4 92 x
10x 60x 90 1 4x 92x 120x 144 23 23
3. 2
2
2
0
2x 18 2
2
2 2x 18
2
2 2x 18
2

Vy f ' x 0 f x ng bin trn 1; 4 . Do phng trnh f x 0 c nghim duy nht


x 3.
Tm ti sng to mt mt s cch gii phng trnh v t
Ngoi ra ta cng c th gii bi ny bng phng php nhn lin hp.

5 x 3
x1 2 4x
2x 2 18
x 3
5 x 3 5 x 3
2 1 1
x 1 2 4 x 2x 18 2 *
x 1 2 4 x 2x 18
t f x 2x 2 18 x 1 2 4 x
x 1 3
Vi x 1; 4 th f x 2x2 10 0 .
2 4 x 5
phng trnh * v nghim!
Vy phng trnh c nghim duy nht x 3 .
Bi 7: Gii phng trnh: x 2 8x 16 x 3 3x 1 2 x 7x 2 0
Nguyn Minh Tun
Phn tch
2
Dng MODE 7 kho st hm f x x 8x 16 x 3 3x 1 2 x 7x 2 .
Nhp vo my hm s trn ri cho:
START 1
END 20
STEP 1
Ta c bng nh bn. Nhn vo bng th ta
thy hm c v ng bin v ct trc honh ti
im c honh bng 0. chc chn th ta s
i v th, ta c nh bn. Nhn vo th ta
chc chn rng hm ng bin nn ta s i
chng minh iu ny.
Ta c: 12

9x 7 21x 10 10

f ' x 2x 8
2 3x 1 2 7x 2 8

2 6

chng minh f ' x 0x cng khng


7 4

phi l iu kh khn g. Trc ht ta cn phi 2

chia trng hp th mi lm c. 15 10 5 5 10 15

2 10
Trng hp 1: x ; . 2

7 21 4

21x 10 0 6


Khi th 9x 7 0 . Cho nn f ' x 0 . 8

2x 8 0 10


12

10 7 21x 10 0 21x 10 9x 7
Trng hp 2: x ; . Khi f x 2x 8
21 9 9x 7 0 2 7x 2 2 3x 1
Mt khc ta li c:
21x 10 2 2x 8 7x 2 21x 10 40 21x 10 10 7
2x 8 0x ;
2 7x 2 2 7x 2 2 7x 2 21 9
10 7
f x 0x ;
21 9
Tm ti sng to mt mt s cch gii phng trnh v t
7
Trng hp 3: x . Khi ta c:
9
12x 3 19x 2 224x 64
7
9x 7 2 x 4 3x 1 9x 7 2 x 4 3x 1 9x
1. x 4 0
2 3x 1 2 3x 1 2 3x 1
28x 3 209x 2 512x 128
10
21x 10 2 x 4 7x 2 21x 10 2 x 4 7x 2 21x
2. x 4 0
2 7x 2 2 7x 2 2 7x 2
7
f ' x 0x
9
2
Vy f x ng bin trn ; nn x 1 l nghim duy nht ca phng trnh.
7
3 9x2 8x 4
Bi 8: Gii phng trnh: 2 2 x 1 1
x

3x 2 2x 1
thi th THPT Quc Gia 2016 THPT Trn Hng o knng
Phn tch
Vn nh mi khi ta vn i kho st v v th hm s thi!
3 9x 2 8x 4
MODE 7 vi hm f x 2 2 x 1 1
x

3x 2 2x 1
.

Nhp vo my hm s trn ri cho:


START 1
END 5
STEP 0, 2
Nhn vo bng ta thy lun phng trnh u c
nghim l x 1 v hm c du hiu nghch
bin. bit chnh xc ta s i v th. Ta
thy rng th c du hiu i xung, vy hm
l nghch bin. Do nu ta chng minh c
hm nghch bin l OK.
Ta c: 4

2x 2 3x 6 3 x 1
f ' x 10 5 5 10 15 20

x2 x 1 2


27x 2
12 2x 1 54x 2 52x 8 6

2 8

3x 2 2x 1 2x 1 10

12

Nhn thy chong ri! Vic chng minh 14

khng phi n gin cht no! Nhng hy 16

rng: 18

9x 2 8x 4

3x 2 2x 1 3x 2 2x 1
Khi phng trnh s n gin hn rt nhiu. Ta s khng phi i chng minh con p phy
kia! Ta c:
3 9x 2 8x 4


2
x
2 x 1 1
3x 2 2x 1

3
3x 2 2x 1 3x 2 2x 1
x

2 2 x1 1
3x 2 2x 1
Tm ti sng to mt mt s cch gii phng trnh v t
3

2 2 x 1 1 3x 2 2x 1
x
2x 3 2 x 1 1 3x 2
2x 2x 1

2 2x 3 x 1 2x 2x 1 3x 2 2x 3 0
By gi t f x 2 2x 3 x 1 2x 2x 1 3x 2 2x 3 .
Ta c:
6x 7 6x 2
1. f ' x 6x 2
x1 2x 1
9x2 30x 37 x 1 1
2.
6x 7
3x 1

6 x 1 3x 1 x 1 1
6 x 1 3x 1 0
x1 x1 x1
6x 2 6x 2 3x 1 2x 1 18x 3 33x 2 15x 5x 5
3. 3x 1 0
2x 1 2x 1
6x 2 3x 1 2x 1 2x 1
Suy ra f ' x 0 f x nghch bin trn 1; . Vy x 1 l nghim duy nht ca phng
trnh u.
Bi 9: Gii phng trnh: 3 x 4 3 5x 4 4x 2 18x 12 0
Phn tch
MODE 7 vi hm f x 3 x 4 3 5x 4 4x 2 18x 12
Nhp vo my hm s trn ri cho:
START 0, 8
END 5
STEP 0, 2
Ta c bng nh bn. Nhn vo bng th ta
thy r rng phng trnh khng ch c 1
nghim duy nht x 0 m cn c 1 nghim na
thuc khong 3, 6; 4 . Tt nhin l ta phi chia
3 trng hp ri. Trc tin xc nh cc
trng hp th ta c th dng MODE 7 th c l
hm ny ng bin khi x 3 v nghch bin khi
x1.
V phng trnh u v nghim trn 1; 3 .
3 x 4 8
Vy l ta c nh hng gii bi ny. Nu: x 1; 3 .Khi
3 5x 4 14
2
f x 4x 18x 10 0x 1; 3
Vy phng trnh v nghim trn 1; 3 .
3 15
Ta c: f ' x 8x 18
2 x 4 2 5x 4
Nu x 3 th hin nhin f ' x 0 f x ng bin trn 3; .
4 15
Nu x ; 1 th ta nhn thy rng cha chc hm nghch bin do lim
5 x
4 2 5x 4
5
Tm ti sng to mt mt s cch gii phng trnh v t
Lc ny khng th no c chuyn f ' x 0 c. thy c hm i chiu th hy kho st
0, 1
hm ny trn 0, 8; 0,7 vi STEP ta s thy n i du ngay. Vy khc phc ta li
29
4
chia nh khong ; 1 hn na. Khi ta s xt 2 trng hp sau:
5
4 1 3 x 4 5 4 1
Nu x ; th f x 4x 2 18x 7 0x ;
5 2 3 5x 4 0 5 2

1 2 x 4 3, 5 135
Nu x ; 1 khi f ' x 8x 0 . Vy hm nghch bin trn
2 2 5x 4 2 14
1
2 ; 1 .
1
Vy phng trnh s c 1 nghim trn 3; v 1 nghim trn ; 1 . Lc ny ch ra 2
2
3 21
nghim l x 0 v x l xong!
2
Ngoi cch lm trn ta cng c th lm theo cch lin hp li 1 nghim ra cn mt nghim dng
hm s gii quyt. Ta cng lm th c 2 hng xem sao.
3 21
HNG 1: Li nghim x ra trc.
2
Ta c:
3 x 4 3 5x 4 4x 2 18x 12 0
3 x4 x1 3
5x 4 x 1 4 x 2 3x 3 0
3 x 3x 3
2
3 x 3x 3
2

4 x 2 3x 3 0
x4 x1 5x 4 x 1
2
x 3x 3 0
3 3
f x 40
x4 x1 5x 4 x 1
6 x4 3 6 5x 4 15
C f ' x 2
2
0.
2 x4 x 4 x1 2 5x 4 5x 4 x 1
5
Nn f x ng bin trn ; . Li c f 0 0 nn x 0 l nghim duy nht ca phng
4
trnh f x 0 .
HNG 2: Li nghim x 0 ra trc.
Ta c:
3 x 4 3 5x 4 4x 2 18x 12 0
3 x4 2 3
5x 4 2 4x 2 18x 0
x 0
3 15
f x 4x 18 0
x4 2 5x 4 2
By gi ta s dng c MODE 7 v v th kim tra xem f x c n iu khng.

Nguyn Minh Tun Trang 90


Tm ti sng to mt mt s cch gii phng trnh v t
3 15
MODE 7 vi hm f x 4x 18
x4 2 5x 4 2
Nhp vo my hm s trn ri cho:
START 0, 8
END 5
STEP 0, 2
Ta c bng nh bn. Nhn vo thy rng hm
4
khng n iu trn ; m b i chiu
5
trong khong 0, 8;0 nn ta s phi chng
minh v nghim trn khong ny. hnh dung
r hn ta c th nh bn. 4

Nhn vo th ta thy rng ng l hm b i 2

chiu mt t v cn li l ng bin nn ta s i 10 5 5 10 15 20

chng minh v nghim trc. 2

x 4 1, 5 4

Nu x 0; 8; 0 th: 6

5x 4 0 8

135
f x 4x 0x 0, 8; 0 10

14 12

Nu x 0 ta c: 14

16

18

3 75
f ' x 2
2
4
2 x4 2 x4 2 5x 4 2 5x 4
2 x 4 2 2 x 4 64
D thy vi x 0 th:
.
2

2 5x 4 2

5x 4 64
89
Khi f ' x 0 f x ng bin trn 0; .
32
Vy n y bi ton c gii quyt!

Bi 10: Gii phng trnh: x 4 6x 2 2x 9 x 3 2x 1 x 2 x 1 0


Nguyn Minh Tun
Phn tch
MODE 7 vi hm s f x x 6x 2x 9 x 3 2x 1 x 2 x 1
4 2

Nhp vo my hm s trn ri cho:


START 10
END 10
STEP 1
Ta c bng nh bn. Nu trn my tnh cc
bn th c th thy rng hm ny c v ng
1 5 1 5
bin trn ; v ; . C
2 2
th thy r iu ny khi ta v th. Nhn vo
th ta thy rng hm c nghim duy nht l
Tm ti sng to mt mt s cch gii phng trnh v t
x 2 lin tc v ng bin trn 2

1 5 1 5 10 5 5 10 15

; v ; .
2 2
2

By gi ta s i gii quyt n! 4

C: 6

f ' x 4x3 12x 2 8

8x 4 7x 3 2x 2 4x 5 10


2 x2 x 1
12

14

1 5
Trng hp 1: x ; 16

2
18

Ta cn chng minh f ' x 0 . 20

thy: 8x 7x 2x 4x 5 ' 32x 21x 4x 4 0


4 3 2 3 2


g x
4 3 2
8x 7x 2x 4x 5 1 0
Vy ta cn c mt nh gi kiu nh x2 x 1 ax b
Nu ta chn a 1 th khi lim x2 x 1 x ta khng th xc nh c b
x

1
Nu ta chn a 1 th ta d dng d bng MODE 7 tm c b .
2
1 5 1 5
Vy ta cn nh gi: x 2 x 1 x
0 lun ng vi x ; .
2 4 2
8x 4 10x 3 28x 2 24x 8
Khi f ' x 0
1 2x
2
1 5
28x 24x 8 0x ;
2

1 5
V 1 2x 0x ; .
2

8x 4 10x 3x ; 1 5
2

1 5 1 5
Vy f x ng bin trn ; f x f 0 .
2
2
1 5
Trng hp 2: x .
2
thy:
3 2
g ' x 32x 3 21x 2 4x 4 32 x 1 75 x 1 58 x 1 11 0 g x 0 .
Vn nh trng hp 1 ta vn cn nh gi x2 x 1 ax b .
1
Ln ny chn lun a 1 , dng MODE 7 ta tm c b
2
1 5
Vy ta i chng minh x 2 x 1 x 0 lun ng.
2 4
16x 4 11x 3 22x 2 12x 7 h x
f ' x
2x 1 2x 1
Tm ti sng to mt mt s cch gii phng trnh v t
rng:
3 2
3 3 3 351
h ' x 64x 3 33x 2 44x 12 64 x 255 x 289 x 0
2 2 2 4
1 5 1 5
h x h 17 0 f ' x 0 f x ng bin trn ;
2 2
Vy x 2 l nghim duy nht ca phng trnh.
39x
Bi 11: Gii phng trnh: 21 5 x 3 2x 2 16 5 x 2 0
2
Bi Th Vit Vted.vn
Gii
3 2
x 2x 16 0
KX: x 2
x 2 0
39x
t f x 21 5 x 3 2x 2 16 5 x 2 lin tc trn x0 ; .
2
39 5 3x 2 4x 5
C f ' x
2 2 x 2x 16 2 x 2
3 2

4
Nu x 2; 0 ; th 5 3x 2 4x 0 f ' x 0
3
4 40 3 15
Nu x 0; th 2 x 3 2x 2 16
3 9 2
39 2 3x 4x 24x 2 32x 234 x 2 30
2
5
f ' x 0
2 3 2 x2 12 x 2
49 5 105
Vy f ' x 0 f x ng bin trn 2; . Suy ra x l nghim duy nht ca
8
phng trnh.
Bi 12: Gii phng trnh: x 5 7x 4 12x 3 x 2 3x 6 12 6 5x 0
Nguyn Minh Tun
Gii
Do 6 5x 5 nn iu kin c nghim ca phng trnh s l:
x 5 7x 4 12x 3 x 2 3x 6 0
x 5 7x 4 12x 3 x 2 3x 6 210
x 5 x 4 2x 3 2x 2 9x 42 0

2 2
x 5 do :x 4 2x 3 2x 2 9x 42 x 1 x 4, 5 21,75 0
Trng hp 1: Nu x 5; 4 .
Khi t VT f x th ta c:
f x x5 7x4 12x3 x2 3x 66 x3 x 4 x 3 x2 3x 66 0
Trng hp 2: Nu x 4; 2 th 6 5x 4 . Khi ta c:
f x x 5 7x 4 12x 3 x 2 3x 54
x 5 7x 4 12x 3 20 x 2 3x 34
x 1 x 4 8x 3 20x 2 20x 20 x 2 3x 34 0

Nguyn Minh Tun Trang 93


Tm ti sng to mt mt s cch gii phng trnh v t
x 4 8x 3 20x 2 20x 20 x 2 4x 1 2 2 x 3 2 1 0
V
2
x 3x 34 0x 4; 2
Trng hp 3: Nu x 2; 0 th 6 5x 2 khi :
f x x5 7x 4 12x3 x 2 3x 30 0
x 5 7x 4 12x 3 x 3 x 3 x 4 0
V 2
x 3x 30 x 2 x 5 20 0
6
Trng hp 4: Nu x 0; ta c:
5
30 6
f ' x 5x 4 28x 3 36x 2 2x 3 0x 0;
6 5x 5
30 30 3 6 5x 45x 846
V 3 0
6 5x 6 5x
30 3 6 5x 6 5x
6
f x ng bin trn 0; .
5
Do vy nn phng trnh u s c nghim duy nht x 1 .

1 1
Bi 13: Gii phng trnh: 2x 1 0
x3 1 2 x x2 x 1
Gii

1 1
t: f x 2x 1 lin tc trn 1; 2
3
x 1 2 x x 2 x 1
3x 2 6x 3 3x 2
C f ' x 2 0
2 2 x x2 x 1 2 x3 1
1
Vy x l nghim duy nht ca phng trnh.
2
Bi 14: Gii phng trnh: x 4 x 2 10x 19 x 3 7x 13 x 2 x 1 0
Gii
1 5
t f x x 4 x 2 10x 19 x 3 7x 13 x 2 x 1 lin tc trn ; v
2
1 5
; .
2
8x 4 7x 3 34x 2 5x 27
C f ' x 4x 3 2x 10
2 x2 x 1
1 5
Vi x ta cn chng minh g x 8x 4 7x3 34x 2 5x 27 0
2
8x 4 8 2 8.8.x 4 16x 2
Nu x 0; 1 th theo AM GM ta c .
7x 3 7 14 x 3 14x 2
g x 4x 2 5x 12 0

Nguyn Minh Tun Trang 94


Tm ti sng to mt mt s cch gii phng trnh v t
2
4 8 86
3
Nu x 1 g x 8 x 1 39 x 1 35 x 0
7 7
1 5
Vy f ' x 0x ;
2
1 5
Vi x ta cn chng minh rng:
2
u x 2 4x 3 2x 10 x 2 x 1 8x 4 7x 3 34x 2 5x 27 0
8x 4 7x 3 20x 2 0
1 5 2
Nu x 2; th 14x 5x 27 0 u x 0
2 4x 3 2x 10 0

Nu x 2 .
Ta thy rng 12x 2 5x 27 0 nn ch cn chng minh:
A 2 4x 3 2x 10 x 2 x 1 8x 4 7x 3 22x 2 0

2
thy: 20 x x 1 10x 2

10 x 4 4x 2 4x 4 0 nn li chuyn v chng
2 2
x x x1
minh:
2 4x 3 2x x 2 x 1 8x 4 7x 3 12x 2

x 2 4x 2 2 x 2 x 1 8x 3 7x2 12x 0

2 4x 2 2 x 2 x 1 8x 3 7x2 12x 0
48x 5 143x 4 232x 3 192x 2 16x 16 0
t f1 x 48x5 143x 4 232x 3 192x2 16x 16
C f1 ' x 24x 4 572x 3 696x 2 384x 16
24x 4 16 x 572x 2 696x 384 0x 2
Suy ra f1 x nghch bin f1 x f 2 1152 0 . Suy ra A 0
Vy f ' x 0x 2
Kt lun phng trnh c ti a 2 nghim trn TX l x 1; x 2 .
Nhn chung bi ny nh gi kh l try ci! Ta th lin hp c nhanh khng nh.
x 4 x 2 10x 19 x 3 7x 13 x 2 x 1 0

x 3 7x 13
x 2 x 1 1 x 1 x 2 x 2 3 0

x 3 7x 13
x 1 x 2 x2 3 0
2
x x1 1
3
x 7x 13
t f x x2 3 .
2
x x11
1 5
Nu x th x 3 7x 13 0 (ci ny d t chng minh)
2
1 5
Nu x . thy:
2

Nguyn Minh Tun Trang 95


Tm ti sng to mt mt s cch gii phng trnh v t

- f x
x 3 7x 13 2
x 3

x 2 x 1 2 x 2 2x 7 x 2 x 2 x 1
x2 x 1 1 x2 x 1 1
x 3 19x 2 36x 53
- x 2 2x 7 x 2 x 2 x 1 0
x 2 2x 7 x 2 x 2 x 1
n y bi ton c gii quyt!

--------------------------------------------------------- Ht -----------------------------------------------------------

Nguyn Minh Tun Trang 96


Tm ti sng to mt mt s cch gii phng trnh v t
E. K THUT S DNG BT NG THC
I. CC BT NG THC C BN CN NH.

Bt ng thc Cauchy AM GM .
Cho n s thc dng a 1 , a 2 ,..., a n khi ta c a 1 a 2 ... a n n a 1 .a 2 ...a n
n
1 n2
Bt ng thc AM GM dng cng mu s cho n s thc dng a 1 , a 2 ,..., a n :
i 1 ai
n

a1
i 1

Du = xy ra khi a 1 a 2 ... a n .
Bt ng thc Bunhiacopxki Cauchy Schwarz (BCS).
2
n n n
Cho 2 b s a1 , a2 ,..., an v b1 , b2 ,..., bn . Khi ta c: ai bi ai bi
i 1 i 1 i 1
Ngoi ra cn phi ch n bt ng thc Cauchy Schwarz dng cng mu Engel:
2
n
n ai
a i 2 i 1

i 1 b i
n
bi
i 1
Bt ng thc trn cn c th gi l bt ng thc Svacx.
a a a
Du = xy ra khi 1 2 n . Ring dng cng mu th cn thm iu kin l
b1 b 2 bn
b 1 , b 2 ,..., b n 0
Bt ng thc Minkowski.
Tng qut: Cho mi s thc r 1 v mi s dng a 1 , a 2 ,..., a n , b 1 , b 2 ,..., b n th ta c:
1 1 1
n r r n r r n r r
a i b i a i bi
i 1 i 1 i 1
y ch xt trng hp cho 2 b s a1 , a2 ,..., an v b1 , b2 ,..., bn . Khi ta c:
n n n
2
ai 2
i 1
bi
i 1
a
i 1
i bi

a a a
Du = xy ra khi 1 2 n .
b1 b 2 bn
Bt ng thc Holder.

Cho cc s dng x i , j i 1, m , j 1, n .
j
n m m
n n
Khi vi mi s 1 , 2 ,..., n 0 tha mn i 1 ta c: x i , j xi ,jj
i 1 i 1 j 1 j1 i 1

y ta ch xt trng hp n gin nht cho 3 dy s gm a, b, c ; m, n, p ; x, y,z . Ta c:


3
a 3
b3 c3 x 3 y 3 z 3 m 3 n 3 p 3 axm byn czp
Du = xy ra khi 3 dy tng ng t l.

Nguyn Minh Tun Trang 97


Tm ti sng to mt mt s cch gii phng trnh v t
II. BI.
Bi 1: Gii phng trnh: 13 x2 x 4 9 x 2 x 4 16
Bi 2: Gii phng trnh: x 3 3x 2 8x 40 8 4 4x 4 0
32
Bi 3: Gii phng trnh: x x 6 3 3x 4
x x
Bi 4: Gii phng trnh: x 2 x 1 x 2 x 1 x 2 x 2
2 x2 1 1
Bi 5: Gii phng trnh: x x 1x
3 3
Bi 6: Gii phng trnh: x 4 12x 3 38x 2 12x 67 x 1 7 x 0

Bi 7: Gii phng trnh: 9 x 2


32 x1 x2 24
2
2 x1 x2
Bi 8: Gii phng trnh:
12x2 16x 1 2 24x3 12x2 6x 4 x2 x 4 8x3 9x2 x 0

Bi 9: Gii phng trnh: x 3 x 1 x 2 x 4 2 2 3 x 2 x 3 2x
Bi 10: Gii phng trnh: 4 x 4 x2 1 4 x 4 x2 1 2x
Bi 11: Gii phng trnh: x 4 4x 3 6x 2 3x 3 2x 2 4 2x 1
4
Bi 12: Gii phng trnh: x 3x 2 2 x 3 4 4 x 1 2 x 3 4x
x
Bi 13: Gii phng trnh: 6 x 1 6 4 2x 3 5 5 x 1 2x 3 8x 1
3

5 x 2 4x 9
Bi 14: Gii phng trnh: 2 x 5 1 3x 3x 10
2 10 6x 4 3x 1
Bi 15: Gii phng trnh:
x 1 x 1 2x 1 1 3x 1 3x x 1 2x 1 2x 1 1 3x 3
2x 1 49 x 3 11x 8
Bi 16: Gii phng trnh:
x2 2x 6 9
2
Bi 17: Gii phng trnh: 3x 2 1 x 2 x x x 2 1
4
7x 2 x 4

3x 1 x 3 3 32 x
Bi 18: Gii phng trnh: 1 x
x 5 2 x 1 2 2

Bi 19: Gii phng trnh: 3x 3 x 2 2x 4 2 3 x 5 x 4 1


6
Bi 20: Gii phng trnh: x6 x3 2x 1 6 x6 x 3 2x 1 2x
1 1
Bi 21: Gii phng trnh: x x 1
x x
Bi 22: Gii phng trnh: x 4 x 2 1 3 x7 x 3 x 2


Bi 23: Gii phng trnh: 4x x 2 1 2 x 1 x 2 1 2 x 1 2 5x 1
2 2 x5
Bi 24: Gii phng trnh: x 1 x 2 4 x 2 3x 3 0
4

Nguyn Minh Tun Trang 98


Tm ti sng to mt mt s cch gii phng trnh v t
3x 2 1x
Bi 25: Gii phng trnh: x 2 4x 5 x 1 1
x 2 x 1
2
x x1
2 1 2
Bi 26: Gii phng trnh: 2

x 1 x 2x 1 1 x 2x 1

1 x 1
Bi 27: Gii phng trnh: 1
x11 2x 1 1 3x 1 1

x2 x 1 x2 x 2 x2 x 3
Bi 28: Gii phng trnh: 2
2
2
3
x2 x 1 x2 x 2 x2 x 3
3
Bi 29: Gii phng trnh: x 2 2x 3 85 60x
2
x 2x 2


Bi 30: Gii phng trnh: 1 3x 1 1 x 2 1 5x 1 3 15x 3 35x 2 10x
Bi 31: Gii phng trnh: x x 2 x 2 3x x 5 1 x 2
3 3x7 2x 2 x 2

Bi 32: Gii phng trnh: 2 x 2 5 x 2 x 1 x 2 5 7x 5


2
Bi 33: Gii phng trnh: 1 2x x 2 1 2x x 2 2 x 1 2x 2 4x 1

3x 13
Bi 34: Gii phng trnh: x2 x1 3
4
a b a b
Bi 35: Chng minh rng: 2a, b 0
a 3b b 3a
4 1x 2x 1
Bi 36: Gii phng trnh: 3 3
x 1x
Bi 37: Gii phng trnh: 6 6 2x 1 6 6 2 x 7 7 2x 1 2 x 5
Bi 38: Gii phng trnh:
2x 2 2x 1 2x 2
3 1 x 1 2x 2
3 1 x1 3

3x 3
Bi 39: Gii phng trnh: 2 x 2 4x 5
x3 4 2 x
2 x
1 1 1
Bi 40: Gii phng trnh:
2 2 2
1 x 3x 1 1 x 3x 1 x 1
Bi 41: Gii phng trnh: x 4 x3 6x 4 2 x6 x 1 0
Bi 42: Gii phng trnh: x9 2x 2 x8 x 1 2 x x 3 2x 2 2x 3

Bi 43: Gii phng trnh:



2x 1 x 2x 1 2
x x 1 1 2x 1 2x 1
2
28x 3 9

Bi 44: Gii phng trnh: 1 x 6 1
x 6

1 4
28x 3
256

Nguyn Minh Tun Trang 99


Tm ti sng to mt mt s cch gii phng trnh v t
2
1 1 2x x1
Bi 45: Gii phng trnh: 4 4 2 2
2x x1 x1 2x

Bi 46: Gii phng trnh: 1


2 4
x1 4 3x 1 x1 3x
x1 3x 4
x 1 3 x
Bi 47: Gii phng trnh:
2 2
2 1 2 1
1 3x 1

1 x2
x2
1

17 2 2
3x
4
1
Bi 48: Gii phng trnh:
1

2 x 2x 64
0
4
x 2x
x 2x
Bi 49: Gii phng trnh:
4x 7 2 8x 7 4x 5 2
x 1 2 4 x 1 1 2 8x 7 1
1 x x 1 x
Bi 50: Gii phng trnh: 3
3
2x 1 2x 1 2x 1
2x 1
x 2x 1 x

3
Bi 51: Gii phng trnh: x2 x 1 1 x 3 x2 x 1 x 1 x2 2
4

Bi 52: Gii phng trnh:


2
2
3 2x 3 2 1
x2 4x 1
x 16x 4

Nguyn Minh Tun Trang 100


Tm ti sng to mt mt s cch gii phng trnh v t
III. CC BI TON.
Bi 1: Gii phng trnh: 13 x 2 x 4 9 x 2 x 4 16
Tin s Trn Nam Dng i hc khoa hc t nhin HQG TP.HCM
ngh Olympic 30/4/2011 THPT Chuyn L Hng Phong TH.HCM
Gii
Vi , 0 . Ta s dng bt ng thc AM GM ta c:
13 x 1 x 13 1 x 13
2 2 2 2 2
13
2
13 x x x 1 x
4
2 2 2

2 2
9 x 1 x 9 1 x 9
2 2 2 2 2
13 2 2
2 4
9 x x x 1 x
2

2 2
13 2 1 9 2 1 13 9
2 4 2 4
13 x x 9 x x x2
2 2 2 2

Du = xy ra 2 1 x 2 2 1 x 2 1
2 1 2 1 1

2 VT 16 VP
Chn , 0 tha mn 13 2 1 9 2 1
0 3
2 2 2
2 5
Du = xy ra x .
5
Ngoi ra ta c th dng o hm gii bi ny. t f x 13 x 2 x 4 9 x 2 x 4 hm f x lin
13 2x 3 x 9 2x 3 x
tc trn 0; 1 . Ta c: f ' x .
x2 x4 x2 x4
13 2x 3 x 9 2x 3 x 13 2x 2 1 x 2 1 9 2x 2 1 1 x 2
f ' x 0
x2 x4 x2 x4 x 0 L
2
x 2 ; 1

2
5x 4 200x 160x 22 0
4 2

2 2 5
D thy vi x ; 1 th 200x 4 160x 2 22 108 0 , nn f ' x 0 x .
2 5
2 5 2 5
Do hm f ' x i du t khi qua nn t cc i ti x
5 5
2 5
f x f 16 VP .
5
Bi ton c gii quyt!

Nguyn Minh Tun Trang 101


Tm ti sng to mt mt s cch gii phng trnh v t
Bi 2: Gii phng trnh: x 3 3x 2 8x 40 8 4 4x 4 0
Trch thi HSG Quc Gia Bng A 1995
ngh Olympic 30/4/2014 THPT Chuyn L Qu n Ninh Thun
Gii
Ta d dng thy phng trnh c nghim x 3 nn ta s dng bt ng thc Cauchy kh cn.
Cauchy
Ta c: 8 4 4x 4 4x 4 .16.16.16 x 13 .
4

2
Khi VT x 3 3x 2 8x 40 x 13 x 3 x 3 0x 1 . Vy du = xy ra khi
x3
32
Bi 3: Gii phng trnh: x x 6 3 3x 4
x x
Gii
4
Nhn vo phng trnh ta thy VT 0 nn iu kin c nghim l x .
3
32 x x x x AM GM 32
p dng bt ng thc AM GM ta c: x x 6 x
x x 2 2 x x
2
Khi ta cn chng minh : x 3 3x 4 x x 3x 4 x 2
x 1 0 lun ng.
Vy VT VP . Du = xy ra khi x 4

Bi 4: Gii phng trnh: x2 x 1 x2 x 1 x2 x 2


Gii
ab
Cch 1: Dng bt ng thc dng ab .
2
Cauchy 2
2 x x
x x 1 1. x 2
x 1
2
Ta c: Cauchy 2
x 2 x 1 1. x 2 x 1 x x 2
2
x 2 x 1 1
VT x 1 . Du = xy ra khi x 1.
x 2 x 1 1
2
Li c VP x 2 2x 1 x 1 x 1 x 1 x 1 . Du = xy ra khi x 1 . Nn
VT VP . Du = xy ra ti x 1 nn x 1 l nghim duy nht ca phng trnh.
Cch 2: Dng bt ng thc Bunhiacopxki.
Bunhiacopxki

VT 1. x 2
x 1 1. x 2
x 1 12 12 x2 x 1 x x2 1 2 x
Ta c: 2
VP 2 x x 2 x 2 2 x x 2 2x 1 x 2 x 1 x 1 2 x 1 0

Do du = ch xy ra khi x 1 .
Cch 3: Phn tch tng bnh phng SOS.
Ta c:
2 2
1 1
x2 x 2 x2 x 1 x2 x 1 0 2

x2 x 1 1 2
2
x2 x 1 1 x 1 0

Nguyn Minh Tun Trang 102


Tm ti sng to mt mt s cch gii phng trnh v t
x2 x 1 1


D dng nhn thy VP VT 0 . Nn du = xy ra khi x 2 x 1 1 x 1 .
x 1 0

2 x2 1
1
Bi 5: Gii phng trnh: 1x x x
3 3
Gii
Bi ny kh l c bn p dng bt ng thc Cauchy Schwarz l ra ngay. Ta c:
1 1 2 x2 1
x x
3
1x x 2
1 3
x 1 x


3
VT

x 1 1 1
Du = xy ra khi x x 1 x x3 x2 x 0
1 x1 3 3
x
3
3 1
3x 3 3x 2 3x 1 0 x 1 2x 3 x
1 3 2
Vy bi ton c gii quyt!

Bi 6: Gii phng trnh: x 4 12x 3 38x 2 12x 67 x 1 7 x 0


thi th THPT Quc Gia 2016 Ln 2 Chuyn Vnh Phc
Gii
Cch 1: Dng bt ng thc Cauchy.
1 1 1 x 5 11 x
Ta c: x 1 7 x .2 x 1 .2 7 x 4.
2 2 2 2 2
2 2

Khi VT x 3 x 2 6x 7 x 3 x 1 x 7 0 .

x 1 2
Du = xy ra khi x3
7 x 2
Cch 2: Phn tch tng bnh phng:
Ta c:
x 4 12x 3 38x 2 12x 67 x 1 7 x 0
1 2 1 2

4

x1 2
4
2

7 x 2 x 3 x 1 7 x 0

Khng kh nhn ra vi x 1;7 th VT 0 . Do du = xy ra khi x 3


Vy x 3 l nghim duy nht ca phng trnh.

Bi 7: Gii phng trnh: 9 x 2


32 x1 x2 24
2
2 x1 x2
Gii
tng ca bi ton ny vn nh cc bi ton khc, ta vn s nh gi VT 24 . Nhng tuy
nhin nguyn nh th ny th cha lm n c g, ta s lin hp t ca phn thc.
Ta c:

Nguyn Minh Tun Trang 103


Tm ti sng to mt mt s cch gii phng trnh v t

9 x2
32 x1 x2 24 9 x2
96
24
2 2
2 x1 x2 2 x1 x2 x2 x1
96
6
x2 x1 3 2 x1 x2 2
24
2 x1 x2 x2 x1
3 3
Khi p dng Cauchy cho 4 s: 6a 6 x2 x 1 ; b 2 x 1
2 2
x2
3 3 96 96
2 2

b 2 x1 x2 ; 2

ab 2
ta c:
x2 x1 2 x1 x2
3 3 96 Cauchy 6.3.3.96.a.b.b
6a b b 2 4 4 24 VP
2 2 ab ab2
3
6a 2 b
Vy du = xy ra khi b 4a x 2
6a 96
ab 2
Bi 8: Gii phng trnh:
12x 2 16x 1 2 24x 3 12x 2 6x 4 x 2 x 4 8x 3 9x 2 x 0
Gii
Ta c:
12x 2 16x 1 2 24x 3 12x 2 6x 4 x 2 x 4 8x 3 9x 2 x 0
12x 2 16x 1 2 6x 4x 2 2x 1 2 4 x 2 x 2 4 x 2 x 8x 1 0
6x 4x 2 2x 1
1 2
Phng trnh c nghim x 4 x2 x 1 . Nn ta p dng bt ng thc
2
8x 1 4 x x
2

2 6x 4x 2 2x 1 4x 2 8x 1


AM GM ta c: 2 4 x 2 x 4 x 2 x 1

2 4 x 2 x 8x 1 4x 2 4x 8x 1

1 2
Khi VT 0 . Du = xy ra khi x .
2
Ngoi ra nu khng thch ta c th s dng cch ghp hng ng thc nh sau:
12x 2 16x 1 2 24x 3 12x 2 6x 4 x 2 x 4 8x 3 9x 2 x 0
6x 2 6x 4x 2 2x 1 4x 2 2x 1 1 2 4x 2 4x 4x 2 4x

4x 2 4 2 4x 2
4x 8x 1 8x 1 0
2 2 2
6x 4x 2 2x 1 4x 2 4x 1 4x 2 4x 8x 1 0
n y bi ton c gii quyt!

Nguyn Minh Tun Trang 104


Tm ti sng to mt mt s cch gii phng trnh v t
Bi 9: Gii phng trnh: x 3 x 1 x 2 x 4 2 2 3 x 2 x 3 2x
Gii
y do VT VP 0 m nu s dng Cauchy cho x 1 th bt ng thc b ngc chiu,
khc phc ta s tm nhn t cho x 3 x 1 , ta c x 3 x 1 2 2 x 1 . Nhn t ny
cng chiu vi bi ton nn ta s dng Cauchy chng minh.
3 Cauchy 3
Ta c: x 3 x 1 x1 2 x1 2 2 x1 x 1 2 2 x 1
2
Khi VT VP x 1 2x 2 2 3x 4 0
2
Du = xy ra khi x1 2 x 1 x 1.

Bi 10: Gii phng trnh: 4


x 4 x2 1 4 x 4 x2 1 2x
Gii
4 x 4 x 2 1 a a b 2x 4
Cch 1: t 4 4 4
a b 8 a4 b4
4 x 4 x 2 1 b a b 2x
2

Ta c:
a b
4


a 2
2ab b 2 Cauchy

a 2
b2

a 4 b 4 2a 2 b 2
a4 b4
8 8 2 2
Du = xy ra khi a b 4 x 4 x2 1 4 x 4 x2 1 x 1
Cch 2:
Ta c:
2 Bunhiacopxki
1. 4
x4 x2 1 4 x4 x2 1 2 x4 x2 1 x4 x2 1
Bunhiacopxki
2. 2 x4 x2 1 x4 x2 1 2 2 x 4 x 2 1 x 4 x 2 1 4x 2
3. Do VT VP . Du = xy ra khi x 1 .

Bi 11: Gii phng trnh: x 4 4x 3 6x 2 3x 3 2x 2 4 2x 1


Gii
Cauchy
1 1 4 2x 2 3 x
2 2 x 1 2 2

2
Ta c: Cauchy
4 2x 1 1.1.1. 4 2x 1 2x 1 3 x1

4 2
4
x 3 x 1 2 x 1
Khi VT 0
2 2 2
Du = xy ra khi x 1

4
Bi 12: Gii phng trnh: x 3x 2 2 x 3 4 4 x 1 2 x 3 4x
x
Gii
u tin ta s nhn x ln tin nh gi. Khi phng trnh tr thnh:
3x3 x2 4 2x x3 4 4x x 1 2x x3 4x
By gi ta cn phi chng minh VT VP 0 . Ta s bin i nh sau:

Nguyn Minh Tun Trang 105


Tm ti sng to mt mt s cch gii phng trnh v t
3x 3 x 2 4 2x x 3 4 4x x 1 2x x 3 4x
3x 3 x 2 4 2x x 3 4 4x x 1 2x x 3 4x 0
x 2 2x x 3 4 x 3 4 x 2 4x x 1 4 x 1 2x 3 x 2 4x 12 2x x 3 4x 0
2 2

x x3 4 x 2 x1 2x 3 x 2 4x 12 2x x 3 4x 0

Ta i chng minh 2x3 x2 4x 12 2x x 3 4x 0 .


Tht vy:
2
3 2 3
Cauchy
3 4x 2 x 3 4x x 2 3x 6
2
2x x 4x 12 2x x 4x 2x x 4x 12 0
2 2
x x 3 4 0

Khi VT 0 . Du = xy ra khi x 2 x 1 0 x 2
3
2x x 4x
Vy x 2 l nghim duy nht ca phng trnh. Bi ton c gii quyt!

Bi 13: Gii phng trnh: 6 3 x 1 6 4 2x 3 5 5 x 1 2x 3 8x 1


Gii
6 x 1 2x 2
3


Ta s i chng minh 6 4 2x 3 3x
5
5 x 1 2x 3 3x 1
Cauchy
3 x111
6 x 1 6 2x 2
3
Cauchy
2x 3 3
Tht vy, ta c: 6 4 2x 3 6 .
4
Cauchy
5 x 1 2x 3 3
5 x 1 2x 3 5 3x 1
5
Cng v theo v 3 bt ng thc trn ta s c VT VP
Du = xy ra khi x 2 .

5 x 2 4x 9
Bi 14: Gii phng trnh: 2 x 5 1 3x 3x 10
2 10 6x 4 3x 1
Gii

Ta c:
Bunhiacopxki
1 1
1. 2 10 6x 4 3x 2 10 6x 6x 8 18 4 9
2 2
1 1 5 x 2 4x 9 x 2 4x 9
2. Do
2 10 6x 4 3x 1 10 2 10 6x 4 3x 1 2
2

Li c:
2
x 4x 9
2 x 5 1 3x 3x 10
x 5 2 1 3x 0
2 2
Nn VT VP
Du = xy ra khi x 1

Nguyn Minh Tun Trang 106


Tm ti sng to mt mt s cch gii phng trnh v t
Bi 15: Gii phng trnh:
x 1 x 1 2x 1 1 3x 1 3x x 1 2x 1 2x 1 1 3x 3
Bi Th Vit Vted.vn
Gii
Cch 1.
a x 1

t b 2x 1 a 2 b 2 c 2 3 .

c 1 3x
2
Phng trnh tr thnh: a 3 b ac3 cb 3 3 3 a 3 b ac3 cb 3 a 2 b 2 c 2
b x a
t a v phng trnh bc 2 n a, phng trnh tr thnh :
c y a
x2 xy y 2 a 2 x3 5x2 y 4xy 2 y 3 a x4 3x3 y 2x2 y 2 y 4 0
2
C x 3 5x 2 y 4xy 2 y 3 4 x 2 xy y 2 x 4 3x 3 y 2x 2 y 2 y 4 . Do ang dng ng
x
cp nn ta c th t t . Quy v gii phng trnh:
y
3t 6 6t 5 9t 4 18t 3 6t 2 12t 3 0
Nhng tuy nhin phng trnh ny c nghim thuc phng trnh bc 3 nn c th gi s
2
3t 6 6t 5 9t 4 18t 3 6t 2 12t 3 3 t 3 at 2 bt c
2
ng nht h s ta s c: 3 x 3 x 2 y 2xy y 3 0 . Do VT VP .
Du = xy ra khi x 0
Ngoi cch ng nht h s ta c th tm 3 nghim ca phng trnh ri dng viet cho phng
trnh bc 3
Cch 2.
Bi ton xut pht t ng thc sau:
a2 b2 c2 3 a3 b b3c c3a 21 a2 2ab bc ca c2 0
2 2

cyc

Chng minh:
x a2 ab bc 3 3 3
xy yz xz a b b c c a
t y b 2 cb ca 2 2 2
z c 2 ca ab x y z a b c

Khi c:
2 2
a 2
b 2 c2 3 a 3 b b 3 c c 3a x y z 3 xy yz xz
1 2 1 2 1
x y y z x z 2
2 2 2
1 2 1 2 1 2
a 2 2ab bc ca c 2 b 2 2bc ca ab a 2 c 2 2ca ab bc b 2
2 2 2
By gi ta p dng: t a x 1; b 2x 1; c 1 3x
Khi ta c:

Nguyn Minh Tun Trang 107


Tm ti sng to mt mt s cch gii phng trnh v t
x 1 x 1 2x 1 1 3x 1 3x x 1 2x 1 2x 1 1 3x 3
6 x 1 x 1 2x 1 6 1 3x 1 3x x 1 6 2x 1 2x 1 1 3x 18 0
2

4x 2 x 1 2x 1 x 1 1 3x 2x 1 1 3x
2
x x 1 2x 1 x 1 1 3x 2 2x 1 1 3x
2
5x x 1 2x 1 2 x 1 1 3x 2x 1 1 3x 0
x0
Cch 3.
Ta c:
x 1 x 1 2x 1 1 3x 1 3x x 1 2x 1 2x 1 1 3x
Bunhiacopxki
x 1 2 2x 1 2 1 3x 2 x 1 2x 1 2x 1 1 3x 1 3x x 1

9 7x 2 14x 2 3
Nu 14x 2 3 0 th bi ton coi nh c gii quyt!
1 3
Nu 14x 2 3 0 x ; .
2 14
1. Khi p dng Cauchy ta c:
x 1 x 1 2x 1 1 3x 1 3x x 1 2x 1 2x 1 1 3x
Cauchy
x 1 3x 2 2x 1 2 x
1 3x 1 3x x 1
2 2
x2 1
1 3x 1 3x x 1
2

4x 2 3 7 1 3x x 1 17x 1
8

1 3x x 1 x 1
Cauchy
2 2x
1 3x x 1 x 1 x1 0
2
2. Li c :

7 1 3x x 1 17x 1 0 x 1; 8 14 7
109

3. Nn VT 3 pcm
Cch 4.
Ta c:
2
a2 b2 c2 3 a3 b b3 c c3a a 4 a2 b2 3 a2 b2 a2 bc 3 a3 b a2 bc
cyc cyc cyc cyc cyc cyc


3 a 3 b a 2 bc 3 b 3 c a 2 bc 3 bc a 2 b 2
cyc cyc cyc cyc cyc

3 bc a b ab bc ca a 2 b 2
2 2

cyc cyc

a 2 b 2 ab ca 2bc
cyc

1 2
3 a 2 b 2 a 2 bc ab ac 2bc
cyc cyc 2 cyc
1 2
a 4 a 2 b 2 a2 b2
2 cyc

cyc cyc

Nguyn Minh Tun Trang 108


Tm ti sng to mt mt s cch gii phng trnh v t
Vy ta thu c ng thc:
2
a 2
b 2 c 2 3 a 3 b b 3 c c3 a
1 2 1 2

2 cyc
a 2 b 2 ab ac 2bc a 2 b2 ab ac 2bc
2 cyc cyc

1 2


2 cyc
a 2 b 2 ab ac 2bc 0

Vy bi ton c gii quyt!

2x 1 49 x 3 11x 8
Bi 16: Gii phng trnh:
x2 2x 6 9
Gii
Do bi ton c nghim x 4 nn ta s dng bt ng thc Cauchy nh sau:
Cauchy
1 x5
3 .3. 2x 1
3
Ta c:

Cauchy 49 x 2
49 x 3
2
x 5 49 x 2 2

Khi VT VP 3 2
11x 8

x 4 44x 87
0.
x2 2x 6 9 18 x 2 2x 6
Du = xy ra khi x 4
2
Bi 17: Gii phng trnh: 3x 2 1 x 2 x x x 2 1
4
7x 2 x 4

Gii
Ta s p dng bt ng thc Bunhiacopxki cho ton b v tri ca phng trnh
Ta c:
Bunhiacopxki
3x 2 1 x 2 x x x 2 1 1 2
12 x 2 3x 2 1 x 2 x x 2 1

3x 2 1 x 2 x x x 2 1 5x 2
x x 2 2
1 2 7x 2 x 4
Mt khc 5x 2
x x 2 2
2
5x 2 x .2 x 2 2
2

2
VP

Nn du = ch xy ra khi x 1

3x 1 x 3 3 32 x
Bi 18: Gii phng trnh: 1 x
x 5 2 x 1
2 2

Gii
2
3 32 x 3x 6x 3 1
t f x 1 x f ' x 0 . Nn f x nghch bin trn
2 2 1 x
3
2 x
1
0; 1 f x f 1 .
2
Ta c:
x 5 x 1 Bunhiacopxki x 5 2 x 1
2
Bunhiacopxki Cauchy
1. 3x 1 x 3 2 2 x 1 x3
2 2

Nguyn Minh Tun Trang 109


Tm ti sng to mt mt s cch gii phng trnh v t
1
2. Khi VT VP
2
Du = xy ra khi x 1

Bi 19: Gii phng trnh: 3x 3 x 2 2x 4 2 3 x 5 x 4 1


Gii
Cauchy
D thy nu p dng bt ng thc Cauchy lun th ta c: 2 3 x 5 x 4 1 x 5 x 2 4 .
2

Khi VT VP 3x3 x2 2x 4 x 5 x 2 4 x 1 x 3 3x 2 2x , ta cha th xc nh
c du ca bt ng thc. Do ny ra tng phn tch a thc trong cn thnh nhn t th
khi s d dng dng bt ng thc nh gi c. phn tch c ta s lm nh sau:
Do f x x5 x 4 1 0 khng c nghim hu t tc l khng phn tch c di dng
ax b .f x nn ta s gi s f x x 2 ax 1 x 3 bx c 1 . C th gi s nh vy do:
1. Phng trnh ch c 1 nghim l duy nht m nghim ny khng cng thuc 1 phng
trnh bc 2 no nn nu phn tch c di dng mt a thc bc 2 nhn vi mt a
thc bc 3 th a thc bc 2 phi v nghim.
2. Do h s t do bng 1 v kt hp vi l do trn ta s vit c phng trnh di dng
trn.
Khi c:
x2 ax 1 x3 bx c 1 x 5 x 4 a b x3 ab c 1 x2 ac b 1 x a c 1
a b 1
ab c 1 0 a 1
c b bc c 0 b 1 c 0
ng nht h s ta c: 2
2
b 0
ac b 1 0 2b c c 0 2b c c 0 c 1
a c 0

Vy x 5 x 4 1 x 2 x 1 x 3 x 1
Khi p dng bt ng thc Cauchy ta c:
Cauchy x2 x 1 3 x3 x 1
2 3 x 5 x 4 1 2 3 x 2 x 1 x 3 x 1 2 VT
2

x 0

1 7
Du = xy ra khi x 2 x 1 3 x 3 x 1 x 1 3x 2 2x 2 0 x
3

x 1 7
3
Th li thy tha mn. Vy bi ton c gii quyt!
6
Bi 20: Gii phng trnh: x6 x3 2x 1 6 x6 x 3 2x 1 2x
Gii
6 x6 x 3 2x 1 a a 0 6
t a b 32 a 6 b6
6 x6 x 3 2x 1 b b 0
6

D dng chng minh c bt ng thc sau:


a b 3 3
a 6 b 6 32 a 2 b 2 a b
6

32
3 2
S dng bt ng thc 4 a b 3 3
a b 3 a b a b 0 ta cn chng minh c:

Nguyn Minh Tun Trang 110


Tm ti sng to mt mt s cch gii phng trnh v t
3 3 a 2 b2
8 a 2 b 2 a 2 b 2 2ab . Theo AM GM c ab nn bt ng thc hin nhin
2
ng.
6 1 5
T suy ra c du = xy ra khi x 6 x 3 2x 1 6 x 6 x 3 2x 1 x
2
1 1
Bi 21: Gii phng trnh: x x 1
x x MO Yogoslavia
Gii
1
1 x
1 1 Cauchy x
x 1 x
x x 2
Ta c:
1
Cauchy x1
1 1 x
1 x 1
x x 2
1
1 x x 1 5
Khi VP x . Du = xy ra khi x2 x 1 0 x
1 x 1 2
x
Bi 22: Gii phng trnh: x 4 x 2 1 3 x7 x 3 x 2
Gii
Bi ny tng t nh bi 14. Ta s phn tch x x 3 x 2 x 2 x 2 x 1 x 3 x 2 1
7

Khi ta c:
x 4 x 2 1 3 x7 x 3 x 2

x 2 x 1 x 2 x 1 3x 2 x 2 x 1 x 3 x 2 1

x 2 x 1 x 2 x 1 3x 2 x 3 x 2 1
Li c:
Cauchy 3 x3 1 3 x 1 x2 x 1
1. 3x x x 1
2 3 2

2 2
1 2
3 x 1 x 1
2. 2
x x1 4 0
2 2 3 x 1
x x1
2
3 x 1 x2 x 1
3. x 2 x 1 x 2 x 1 3x 2 x 3 x 2 1
2
Nn x 1 l nghim duy nht ca phng trnh.
Ngoi ra nu ta thy bi ny c th a v bi ton ging bi 15 bng cch t

a x 2 ; b x; c 1 . Khi phng trnh tr thnh: a2 b2 c 2 3 a3 b b3 c c 3a
2 1 2 2
Ta bit rng: a 2
b2 c2 3 a 3 b b 3 c c3a
2
a 2ab bc ca c 2 0
cyc

Vy bi ton c gii quyt!

Nguyn Minh Tun Trang 111


Tm ti sng to mt mt s cch gii phng trnh v t


Bi 23: Gii phng trnh: 4x x 2 1 2 x 1 x 2 1 2 x 1 2 5x 1
Gii
Cauchy
Ta c: 4x 2 5x 1 x 2
Khi ta c:

x 2 1 2 x 1 x 2 1 2 x 1 x 2 2 x 4 2x 3 x 2 2x 1 x 2 2x 2

1
5

Nu x 1 3 ; 1 3 ; th bi ton c gii quyt

1
Nu x ; 1 3 ta c:
5
1
2 x 4 2x 3 x 2 2x 1 x 2 2x 2 x 2 3x 2 12x 4 0x ; 1 3
5
T suy ra VT VP . Du = xy ra khi x 0
Bi ton c gii quyt hon ton!

2 2 x5
Bi 24: Gii phng trnh: x 1 x 2 4 x 2 3x 3 0
4
Gii
Xt x 5 ta c:
4 3 2
2 x 5 4 x 5 103 x 5 1001 x 5 4337 x 5 7046
2
x 1 x 2 0
4 4
Phng trnh v nghim trn ; 5
4 5x
Xt x 5 , ta i chng minh x 2 3x 3
4
Nu x 5 th bi ton c chng minh.
Nu x 5; 5 ta c:
5x 2
4
x 2 3x 3 x 4 20x 3 106x 2 268x 143 0 x 1 x 2 18x 143 0
4
Khng kh nhn ra bt ng thc cui lun ng vi x 5; 5 .
Vy VT 0 . Du = xy ra khi x 1
3x 2 1x
Bi 25: Gii phng trnh: x 2 4x 5 x 1 1
x 2 x 1
2
x x1
thi th THPT Quc Gia 2016 Chuyn Quc hc Hu
Gii
Ta c:
3x 2 1x
x 2 4x 5 x 1 1
x 2 x 1
2
x x1


x 2
4x 5 x 2 x 1 3x

x 1 x2 x 1 2 1 x
2
x x1 2
x x1
2 2

x 4x 5 x x 1 3x x 1 x x 1 2 x 2 x 1 1 x
2

x 4 4x 3 10x 2 6x 6 2 x 1 x 2 x 1 1 x 0

Nguyn Minh Tun Trang 112


Tm ti sng to mt mt s cch gii phng trnh v t
Cauchy
x2 2
thy: 2 x 1 x 2 x 1 1 x 2. x 1 .
2
2
Nn VT x 2 x 2 x 1 0 .
Du = xy ra khi x 2
2 1 2
Bi 26: Gii phng trnh: 2

x 1 x 2x 1 1 x 2x 1
thi th THPT Quc Gia 2015 Ln 1 THPT Minh Chu Hng Yn
Gii
Bi ny c tnh che giu i mu ca phn thc th 2 lm nh lc hng ngi lm do ta
1 2
cn c 1 bc bin i: x 2x 1
2
2x 1 1
1 1 1
Khi phng trnh tng ng: 2
2

x 1
2x 1 1
1 x 2x 1
Nhn th ny ta s ngh ngay ti bt ng thc AM GM dng cng mu s, ta c:
1 1 4
2
2
2 2

x 1
2x 1 1 x 1 2x 1 1
Bi ton s c gii quyt nu ta chng minh c:
4 1
2 2

x 1 2x 1 1 1 x 2x 1


3 3x 4 x 2x 1 2 0 x 2x 1
2 2
1 x 2x 1 x 1 2x 1 1

2 x 2x 1 1 2 x 5 2x 1 1 0
2 2
2 1 x 2x 1 x 1 2x 1 1

2 4x 1
2
x 1
2 x 1

5 2x 1

0
4x 1 2 2
2
2 x 1
2x 1 2 1 x 2x 1




x 1 2x 1 1

1
D thy bt ng thc cui ng vi x nn VT VP 0 , du = xy ra khi x 1
2
x a 1 1 1
Ngoi ra nu ta t th khi ta cn chng minh 2
2
.
2x 1 b a 1 b 1 1 ab
2 2
1 1 1 ab a b 1 ab
Ta c ng thc sau y: 2
2
0 . Vy bt ng
a 1 b 1 1 ab a 1 2 b 1 2 1 ab
thc c chng minh.
Ta cng c th dng bt ng thc Cauchy Schwarz chng minh c. Ta c:
a 2 1 b

ab 1 b 1 a 1 2
a b ab 1
a 1

ab 1 b 1 b 1 2 1
a
2
a
b 1 a b ab 1

Nguyn Minh Tun Trang 113


Tm ti sng to mt mt s cch gii phng trnh v t
Cng li c iu cn chng minh!
Nu p dng lun th phng trnh ban u tr thnh:
2 2
x 2x 1 x 2x 1 1 x 2x 1 0
2 2
x 1 2x 1 1 1 x 2x 1
x 2x 1 0
Du = xy ra khi x 1.
1 x 2x 1 0

1 x 1
Bi 27: Gii phng trnh: 1
x1 1 2x 1 1 3x 1 1
Gii
p dng bt ng thc AM GM dng cng mu ta c:
1 x 1 4 x

x1 1 2x 1 1 3x 1 1 x 1 3x 1 2 2x 1 1
p dng tip bt ng thc Cauchy Schwarz ta c: x 1 3x 1 2 2x 1
2 x x2
Khi VT
2x 1 1 2x 1 1 2x 1 1
By gi nhim v ca chng ta l phi chng minh c:
x2 1 2

2x 1 1
1 x 2 2x 1 1
2
2x 1 1 0
Vy bi ton c gii quyt!
Du = xy ra khi x 0

x2 x 1 x2 x 2 x2 x 3
Bi 28: Gii phng trnh: 2
2
2
3
x 2
x1 x 2
x2 x 2
x3
Gii
2
x x1 2
Ta c: 2
1 x2 x 1 x2 x 1
x2 x 1 x 1 x 1 2 0
2
x x1
Nu x 1 th bt ng thc hin nhin ng.
x3 x2 x 3 x2 1 x x 3
Nu x 1 th : x 1 x 1 2 0
2 1 x x 1 2 1 x x 1
x2 x 1
Do 2
1 . Thit lp cc bt ng thc cn li tng t ta cng s c
x2 x 1
x2 x 2 x2 x 3
2
1; 2
1
x 2
x2 x 2
x3
Cng 3 bt ng thc ta c VT VP . Du = xy ra khi x 0

Nguyn Minh Tun Trang 114


Tm ti sng to mt mt s cch gii phng trnh v t
3
Bi 29: Gii phng trnh: x 2 2x 3 85 60x
2
x 2x 2
Gii
2 x 2 2x 1
x 2x 2.1
Theo bt ng thc AM GM ta c: 2
1
.5 85 60x 11 6x
5
Khi :
6x 3 x 2 28x 17 x 2 2x 1 x 2 2x 3
VT VP
x 2 2x 1


2 x 2 2x 3 6x 15 x 2 2x 3 x 2 10x 31
2
x 2x 1
x 1 6x 15
2
x 2x 3 x 2 10x 31
2


x 2

2x 1 2 x 2 2x 3
6x 15 0
17
thy vi x 1 3 ; th x 2 10x 31 0 nn VT VP 0 .
12 x 2 2x 1 0

Du = xy ra khi x 1

Bi 30: Gii phng trnh: 1 3x 1 1 x 2 1 5x 1 3 15x 3 35x 2 10x
Gii
Nhn thy dng tch l s ngh ngay ti bt ng thc Holder , trc ht ra s i chng minh n.
C b sau Holder : Cho cc s thc dng a, b, c, x, y, z, m, n, p khi ta lun c:
3
a 3
b 3 c3 x 3 y 3 z3 m 3 n 3 p3 axm byn czp
Chng minh: Theo AM GM ta c:
a3 m3 x3 3axm
1. 3
a b3 c3 m 3 n 3 p3 x 3 y 3 z 3 3
a 3
b c 3 3
m 3
n 3 p 3 x 3 y 3 z 3
b3 n3 y3 3byn
2.
a 3 b3 c3 m 3 n 3 p3 x 3 y 3 z 3 3
a 3
b 3 c 3 m 3 n 3 p 3 x 3 y 3 z 3
c3 p3 z3 3cpz
3.
a 3 b3 c3 m 3 n 3 p3 x 3 y 3 z 3 3
a 3
b 3 c 3 m 3 n 3 p 3 x 3 y 3 z 3
Cng 3 bt ng thc trn c iu phi chng minh.
a 3x 1

Quay li bi, ta thy 15x3 35x2 10x 5x 3x 1 x 2 . Nn s t b x 2 . Khi

c 5x
phng trnh tr thnh: a 1 b 1 c 1 1 3 abc
3 3
1 1 3
Theo bt ng thc Holder ta c:
3 3
cyc 2 2
a 1
3 3
abc

Nguyn Minh Tun Trang 115


Tm ti sng to mt mt s cch gii phng trnh v t
1
Vy du = xy ra khi a b c 3x 1 x 2 5x x
2
Bi 31: Gii phng trnh: x x2 x 2 3x x5 1 x 2
3 3x7 2x2 x 2
Gii
p dng bt ng thc Cauchy Schwarz ta c:
x x2 x 2 3x x5 1 x x2 x 2 3. 3x x5 1 x 2
3 3x7 2x 2 x 2 VP
Nhn c v d nhng n y mi c vn ny sinh. Du = xy ra khi:
x2 x 2
x x 5 1 x9 x 4 x 2 x 2 0
x
Phng trnh khng tht s d gii cho lm, chc phi dng n hm s.
t: f x x9 x 4 x 2 x 2 lin tc trn 1; .
f ' x 9x8 4x3 2x 1 9x8 9x3 2x3 2x 3x3 1 0x 1
Vy f x ng bin trn 1; . Li c f 1 0 nn x 1 l nghim duy nht ca phng
trnh.
Bn cnh hm s ta cng c th phn tch f x thnh nhn t. Ta c:
f x x 9 x 4 x 2 x 2 x 1 x 8 x7 x6 x 5 x 4 x 2
x 8 x 7 x 6 x 5 x 4 x 2 x 8 x 7 x 6 x 5 2 x x 1 x 2 x 1 0x 1
Khi phng trnh f x 0 x 1

Bi 32: Gii phng trnh: 2 x 2 5 x 2 x 1 x2 5 7x 5 Trch bo THTT


Gii
Cch 1: Bt ng thc.
Kim tra thy phng trnh c 2 nghim l x 2 & x 1 , mt khc ta li c:
2 x 2 5 x2 x 1 x2 5 7x 5
lim 0
x1 x1
2 x 2 5 x 2 x 1 x 2 5 7x 5
lim 0
x 2 x2
2 x 2 5 x 2 x 1 x 2 5 7x 5
lim 0
x 2
x 2 2

Nn x 2 l nghim kp, x 1 l nghim n, do s lin hp li nghm n ra trc, cn


nghim kp s dng bt ng thc nh gi. Ta c:
2 x 2 5 x 2 x 1 x 2 5 7x 5
2 x 2
5 x 2 2 x 1 x 2 5 3 x 1 0
2 x 2 1 x
x 1 x 2 5 3 0
5 x2 2

x 1

2 x 2 1 x
f x x 2 5 3 0 *
5x 22

Gii phng trnh * , ta c:

Nguyn Minh Tun Trang 116


Tm ti sng to mt mt s cch gii phng trnh v t
2 x 2 1 x 1 2 AM GM
x 2 14 2 x 2 1 x
f x 3.
x 5 3 3
5 x2 2 3 6 5 x2 2

x 2 14 2 x 21 x x 2 5 x 2 10x 16
3 x 2
6 5 x2 2

6 5x 2 2




x 2 9
2
2
x 2
5 x2 1

0

6 5 x2 2
Vy f x 0 , du = xy ra khi x 2
Vy phng trnh c 2 nghim x 1; x 2
Cch 2: Lin hp.
Bc 1:Tm nghim, SOLVE c 2 nghim l:

Bc 2: Kim tra nghim bi: Ta c:


2 x 2 5 x 2 x 1 x 2 5 7x 5
lim 0
x1 x1
2 x 2 5 x x 1 x 2 5 7x 5
2

lim 0
x 2 x2
2 x 2 5 x 2 x 1 x 2 5 7x 5
lim 0
x 2
x 2 2

Vy x 1 l nghim n, x 2 l nghim kp.


Ta s tin hnh nhn lin hp do bi ny chia cn rt l. Ta c:
2 x 2 5 x 2 x 1 x 2 5 7x 5
6 x 2 5 x 2 3 x 1 x 2 5 3 5 7x 0


2 x 2 3 5 x 2 x 5 x 1 3 x 2 5 2x 5 0
20 x 2 x 1 5 x 1x 2
2 2

0
3 5 x2 5 x 3 x 2 5 2x 5
x 2 x 1

5 20
f x 0
3 x 5 2x 5 3 5 x 2 5 x
2

thy:
1 20 3 5 x 2 x 35 10x 2 70x 1180
0
2 3 5 x2 5 x 3 5 x2 5 x
3 5 x 2 x 35 3 5 x 2 5 x
Tm ti sng to mt mt s cch gii phng trnh v t
5 x 2
2
5 1 5 2x 3 x2 5
0
3 x2 5 2x 5 2 3 x2 5 2x 5
3 x 5 2x 5 3 x 5 2x 5
2 2

Do phng trnh f x 0 v nghim.
Cch 3: Phn tch tng bnh phng.
2 x 2 5 x 2 x 1 x 2 5 7x 5
2 x 2 x 1
x 1 3 x 2 5 0
2 5 x 2




x 2 9 x 2 x 2 1 5 x 2


2
x 1 1 3 x2 5 0



6 5x 2
2
6


x 2
2

x 2
2
9
1 5 x2
2
x 1
1
3 x2 5 0 x 1 x 2



6 5x 2
2
6



2
Bi 33: Gii phng trnh: 1 2x x 2 1 2x x 2 2 x 1 2x 2 4x 1
Gii

n gin ta s t 2x x 2 t t 0; 2 x 1 x 2 2x 1 1 t 2
2

Phng trnh tr thnh: 1 t 1 t 2 1 t 2 1 2t 2 *


p dng bt ng thc Cauchy ta c: t 1 1 t 2 t 11 t
Khi ta c:
VT VP 2 t 11 t 2 1 t 2 1 2t 2 2 1 t 2 1 1 2t 2 1 t 2
Mt khc ta lun c:

1 1 2t
AMGM
1 2t 2 t t 5 2t 0
2 2 2 2
2
1t 2

2
1
2
Vy VT VP 0 . Du = xy ra khi x 0
Ngoi ra ta cn c th s dng bt ng thc Cauchy phng trnh * . Ta c:

1. t 1 1 t 2 1 t2
2. 2 1 t 2 2 1 t 2 1 1 t 2 t 2 0
3. Do VT VP 2 1 x 2 1 1 2x 2 2 1 x 2 x 2 0

Du = xy ra khi t 0 2x x2 0 x 0 x 2
Bn cnh vi cch s dng bt ng thc ta c th lm theo 3 cch sau, hi try ci!
Cch 1.

Nguyn Minh Tun Trang 118


Tm ti sng to mt mt s cch gii phng trnh v t
1 2x x2 1 2x x2 2 x 1 2x 2 4x 1
2

1 2x x 2 1 2x x 2


2 1 2x x 2 1 2x x2 2 1 2x x2 1
2x x2 1


a 1 2x x 2
- t a 2 b2 2 . Phng trnh tr thnh:
b 1 2x x 2

a b 2a 2 b 2 2a 2 b 2 1
- Khi ta gii h sau:
a b 2a 2 b 2 2a 2 b 2 1 a b 2 2ab 2

2

a b 2 2 a b 2a 2 b 2 2a 2 b 2 1



a b 2
2

a b 2
2
t2 2 t2 2
ab


2 2 ab

2
2 2

t 2 t 2 1 t 2 t7 2t6 4t 5 8t 4 6t 3 12t 2 4 0 *

2

2
2


t





2

2
f t


Theo gi thit suy ra t 1 . Cn chng minh f t 0 . Ta c:

f t t 1 9 t 1 29 t 1 37 t 1 9 t 1
7 6 5 4 3

7 t 1 9 t 1 5 0
2


25
2
275
9 t 1 7 t 1 9 t 1 t 19 x
3 2
0

18 36
Vy ta c:
a b 2 x 0


* t 2
ab 1
a b 1 1 2x x 2
1 2x x 2

x 2

Cch 2.
1 2x x 2 1 2x x 2 2 x 1 2x 2 4x 1
2

2 2 x 1 2 x 1 2x 2 4x 1
2 2


2
2 2

2 2 x 1 2 x 1 2 x 1 1
2 2 2

x 1 t 0 , phng trnh tr thnh:


2
t t


2 2t 2t 2 2t 2 1 8t 8 8t 6 2t 4 t 1 0
2

t 18t 7 8t 6 2t 3 2t 2 2t 1 0 t 1
x 0
x 1 1 x 2 2x 0
2
Vi t 1 ta c
x 2
Cch 3: Lin hp. Ta c:

Nguyn Minh Tun Trang 119


Tm ti sng to mt mt s cch gii phng trnh v t
1 2x x 2 1 2x x 2 2 x 1 2x 2 4x 1
2

1 2x x 2 1 1 2x x 2 1 x 2 x 4x 2 8x 6 0

2x x 2 2x x 2
x 2 x 4x 2 8x 6 0
1 2x x 1
2
1 2x x 1
2

x 0 x 2

1 1
4x 2 8x 6 2x x 2 0
1 2x x 1
2
1 2x x 1
2

Li c:

1 1 1 2x x 2 1 2x x 2


1 2x x 2 1 1 2x x 2 1 1 2x x 2 1 1 2x x 2 1

2 2x x 2


1 2x x 2 1 1 2x x 2 1 1 2x x 2 1 2x x 2

Do ta cn chng minh:

f x 2x 2 4x 3
2
0



1 2x x 1 1 2x x 1 1 2x x 1 2x x
2 2 2 2

Do 1 2x x 2 0 nn ta c:

2
f x 2x 2 4x 3

1 2x x 2 1 1 2x x 2


2x 2

4x 3 1 2x x 2 2

1 2x x 2 1 1 2x x 2


1 2x x 2 4 2x x 2 2x 2 4x 1 0x 0; 2

1 2x x 2 1 1 2x x 2

Vy bi ton c gii quyt!

Cch 4: Bt ng thc

Nguyn Minh Tun Trang 120


Tm ti sng to mt mt s cch gii phng trnh v t
AMGM
2 x 1
2
Ta c: 1 2x x 2 1 2x x 2

Khi VT VP 2 x 1 2 x 1 2 x 1 1
2 2
2


t x 1 t t 0; 1 , khi cn chng minh:

2t 2t 2 2t 2 1 0 2t t 12t 2 2t 1 0
Vy VT VP 0 . Du = xy ra khi x 0; x 2

3x 13
Bi 34: Gii phng trnh: x 2 x 1 3
4
Gii
cn bc 3 th hi kh nh gi nn ta s ly tha ln. Ta c:
3x 13 3 3x 13
x2 x1 3
4
x2 x1
4
0
Theo AM GM ta c:
3
x1 3x 13 1 2
VT
2
x1
4

8

x 1 2 x 2 4x 1 2 x 1 x 1
n y ta s cn chng minh f x x 4x 1 2 x 1 x 1 0 , nhng c v ta cn tm
2

thm iu kin th mi c th chng minh c. thy x 2 x 1 nn iu kin c


13
nghim s l x 2; .
3
4x 2 25x 7 13
Ta c: f ' x 2x 4 3 x 1 0x 2; t suy ra
2x 4 3 x 1 3
13
13 f 22, 189
f f x f 2 . M 3 nn f x 0 . Vy VT 0 .
3 f 2 3 6 3

Du = xy ra khi x 3
a b a b
Bi 35: Chng minh rng: 2a, b 0
a 3b b 3a
Gii
a
Do bt ng thc ang dng ng cp nn ta s chia c t v mu cho b v t x x 0
b
. Ta cn chng minh bt ng thc sau:
x 1 x 1
x3

3x 1

2 2 x 3 3x 1 x 1 3x 1 x 3 0
Bi ny kh l nhiu cn nn ta s ngh cch lm gim bt cn, tuy nhin kh l b tc. bi ny
s dng bt ng thc a b 2 a 2 b2 . Ta c:

3x 1 x 3 2 4x 4 2 2 x 1
Khi VT 2 x 3 3x 1 2 2 x1
x 1 f x

Nguyn Minh Tun Trang 121


Tm ti sng to mt mt s cch gii phng trnh v t
4

Ta c: f x
4 x 1 0
2 x 3 3x 1 2 2 x1 x 1
Do du = xy ra khi x 1 a b . Vy bt ng thc c chng minh!
Ngoi ra ta c th nh gi bng AM GM nh sau. Ta c:
2 a a ab 2 b 1 2b a b 3 a
;
a 3b a b a 3b a 3b 2 a 3b a 3b 4 2 a b
2 b b ab 2 a 1 2a a b 3 b
;
b 3a a b b 3a b 3a 2 b 3a b 3a 4 2 a b
Cng hai bt ng thc trn vi nhau ta c iu phi chng minh.
Ngoi ra ta c th gii phng trnh bng lin hp nh sau:
Nhn thy phng trnh ang c 3 cn nn c th t a x gim bt cng knh ca
phng trnh.
t a x a 0 phngtrnh tr thnh:

2 3a 4 10a 2 3 a 1
3a 2 1 a 2 3 0 .Ta tm c nghim kp a 1 nn s

3a 2 1 3 a 1
tm c 2 biu thc lin hp l 2 2 . Khi tin hnh nhn lin hp ta c:
2 1 3
a 3 a
2 2
2 3a 4 10a 2 3 a 1 3a 2 1 a 2 3 0
2 3a 4 10a 2 3 2a 2 4a 2
3 1 1 3
a 1 3a 2 1 a a 1 a 2 3 a 0
2 2 2 2
8 a 1 a 2 1 3 a 1a 1 3 a 1a 1
2 2 2

0
2 3a 4 10a 2 3 2a 2 4a 2 3 1 1 3
4 3a 2 1 a 4 a 2 3 a
2 2 2 2
a 1

8 a2 1 3 a 1 3 a 1
0 *
2 3a 4 10a2 3 2a 2 4a 2 4 3a 2 1 6a 2 4 a 2 3 2a 6
Nhim v l s i chng minh phng trnh * v nghim.
thy:
1 3 a 1 48a 2
4. 0 a 0
2 4 3a2 1 6a 2

4 3a2 1 4 8 3a 2 1 12a 4
1 3 a 1 48
5. 0 a 0
2 4 a 3 2a 6
2

4 a 3 4a 8 a 3 4a 12
2 2

6. Cn li ta c:
Tm ti sng to mt mt s cch gii phng trnh v t
8 a 2 1
1
2 3a 4 10a 2 3 2a 2 4a 2
24a 2 24a 1
2

0

2 3a 4 10a 2 3 2a 2 4a 2 6a 2 4a 6 2 3a 4 10a 2 3
Nn do phng trnh * v nghim.
Vy phng trnh c nghim x 1

4 1x 2x 1
Bi 36: Gii phng trnh: 3 3
x 1x
Gii
Cch 1.
u tin nhn thy bi ny c dng phn s nn ta s th dng bt ng thc BCS dng cng mu
2
a2 b2 a b
s Engel . Ta c b sau: .
x y xy
2
2
b a b 2
a 2 b2 1 a
Chng minh:
x y xy
x y2 2

x y xy

p dng vo bi ta c:
2 2 2

VT VP
2 1x
2x 1 3 3
2 1 x 2x 1 3 3
x 1x 1 x 2x 1 1x x 1 x 2x 1
2

Ta cn chng minh f x 2 1 x 2x 1 3 3 1x x 1 x 2x 1 0 .
Ta c:
2

f x 2 1 x 2x 1 3 3 1x x 1 x 2x 1
3x 2 3 3 1 x 4 3x 2 2 27x 9
2


2 2 1 x 2x 1 x 2 6 9x 3 1 x

3 3 1x 4 27x 9
t : g x
2 2 1 x 2x 1 x
2 6 9x 3 1 x


2 3 3 1 x 4 6 9x 3 1 x 2 27x 9 2 1 x 2x 1 x

4 2 1 x 2x 1 x 6 9x 3 1 x
36 3 1 2x 1 x 162x 2 162x 48 54x 2 18x 4 27x 9 1 x 2x 1


4 2 1 x 2x 1 x 6 9x 3 1 x
108x 2 180x 48 1 x 2x 1 36 3 2x 1 4 27x 9

4 2 1 x 2x 1 x 6 9x 3 1 x
Tm ti sng to mt mt s cch gii phng trnh v t

11664x 2 5184
108x 2 180x 48 1 x 2x 1
36 3 2x 1 4 27x 9


4 2 1 x 2x 1 x 6 9x 3 1 x
1
Ta lun c: 108x 2 180x 48 12 3x 4 3x 1 0x ; 1 . Nn g x 0 . Do
2
2
f x 0 . Du = xy ra khi x
3
Cch 2.
2
Bc u tin bao gi cng l i tm nghim. Ta c 1 nghim x .
3
Kim tra nghim bi ta c:
4 1 x 1 x x 2x 1 3 3x 1 x
lim 0
x
2 3x 2
3

4 1 x 1 x x 2x 1 3 3x 1 x
lim 0
3x 2
2 2
x
3

2
x l nghim kp.
3
4 1 x 1 x ax b 0

Tm nhn t tng cm, gi s nhn t c dng: .
x 2x 1 cx d 0


x 2x 1 3x 4 3 0

Khi s tm c 2 nhn t l: 9 .
16 3
4 1 x 1 x 2 3x 0
9
Phn tch nhn t tng cm 1 s dng cng thc chia 2 cn ta c:
16 3 4 2 3

2
1. 4 1 x 1 x 2 3x 1x 3 1x 3
9 9 27

4 3 3 2x 1 3
2. x 2x 1 3x
9 54
Nhn vo r rng thy nhn t th 2 khng c cng du vi bi ton do cha nghim bi 3
nn ta s lm mnh tay hn l phn tch nhn t c cm:
16 3
3 3x 1 x 2 3x x 2x 1 vi hy vng c cm s lun dng. Tht vy ta
9
16 3
c: 3 3x 1 x 2 3x x 2x 1
9

3 9x 4 3
2

2x 1 2 2x 1 0
3 6

Vy li gii l:

Nguyn Minh Tun Trang 124


Tm ti sng to mt mt s cch gii phng trnh v t
4 1 x 1 x x 2x 1 3 3x 1 x

2 3 3 3 9x 4 3
2 2


3
1x

2 3 1 x 1 2x 1
3
2 2x 1
3
0

6
0
0


1 x 3 0
2
D thy VT 0 nn du = xy ra khi 3 x
3 3
2x 1 0
3
Cch 3.
2
1. Tm nghim: Phng trnh c nghim kp x
3
2. Tm nhn t cha nghim: 2 1 x 2x 1 3
3. Chia cn ta c kt qu:

f x 3 x 2 3x 2
x 3 1 x 2 1 x 2x 1 2x 1
2 1 x 2x 1 3
2 2
g x

1
4. thy vi x ; 1 th g x 0 . Nn phng trnh g x 0 v nghim!
2

Cch 4. Lin hp

4 1 x 1 x x 2x 1 3 3x 1 x
4 2 3


2 3
1 3
3x 2 0
2
1x 3 1 x 3 3 2x 1 3
9 27 54 3

3x 2 0
2


12 1 x 2 3 4 3x 2 3
0 *


2 3
3 3 1 x 3 3
3 2x 1 3

1. thy :

4 3x 2 3 12 3x 2 3 3 2x 1 3 36x 15

3
3 3 3
3
2x 1 3 3 3 2x 1 3 3 3 2x 1 3

2. Nn phng trnh * v nghim.


2
3. Vy phng trnh c nghim x .
3
Cch 5. Lin hp

Nguyn Minh Tun Trang 125


Tm ti sng to mt mt s cch gii phng trnh v t
4 1 x 1 x x 2x 1 3 3x 1 x
2 3 3 2 8
3
4 1 x 1 x x x 2x 1 3x 3 6x 8x 0
2 3 3 3
3 2 2
2 2

x 3 x
3 3 2
2
4
4 1 x x 6 3 x 0
2 x 3x 1 3
1 x 3 2x 1 3
3 2 3


x 2
3


f x x1 x
2 x

3x 1
2 3 0 *

1 x 3 2x 1 3
3 2 3

+ Nhn thy
2 x 1
3. g x 1 x 3 nghch bin trn ; 1 nn g x g 1 3
3 2 2 6

3x 1 1 1
ng bin trn ; 1 nn v x v
3
4. v x 2x 1 3
3 2 2 6

x1 x
+ Khi f x 2 3 0
3 3
6 6
Cch 6: Bt ng thc.
a 2x 1
a 4b
t a 2 2b 2 1 . Phng trnh tr thnh: 2 2 3 3
b 1 x b a b2

Ta c:
AMGM
a4 a4 a4 a2 a
27a 4 2 3 3a 2
a .b .b a 2 b 2 b 2 1 b
2 2 2 3 4
b
27
27
AMGM
16b 4 16b 4 16b 4

2b 2 a 2 b 2 a 2 b 2 2b2 a2 a2 b2 b2 2
3 3


3 3
16b2 4b
108b 4 3 3.2b2
a b2 a b2
2 2
2

3 2
Cng li ta c VT 3 3 VP . Du = xy ra khi a b x .
3 3

Nguyn Minh Tun Trang 126


Tm ti sng to mt mt s cch gii phng trnh v t
Bi 37: Gii phng trnh: 6 6 2x 1 6 6 2 x 7 7 2x 1 2 x 5
Gii
Do phng trnh c nghim kp nn ta dng phng php nh gi. Ta c:
1. f x 6 6 2x 1 1 6 6 2 x 1 7 7 2x 1 2 x 7
AM GM
2. 6 6 2x 1 1 7 7 2x 1
AM GM
3. 6 6 2 x 1 7 7 2 x
Do ta c:
f x 7 7 2x 1 7 7 2 x 7 7 2x 1 2 x 7 7 7
2x 1 7 2 x 7 2x 1 2 x 1
2
14 x 1
7 7

2x 1 1 1 7 2 x 6 i 6 j

i 0
7

2x 1 .
j 0
7
2x
7 2x 1 0
M , nn f x 0 . Du = xy ra khi x 1
7
2 x 0

Bi 38: Gii phng trnh: 2x 2 2x 1 2x 2


3 1 x 1 2x 2
3 1 x1 3
Gii

Nhn dng ny ta s dng bt ng thc vect. Cho 2 vect v x; y v u a; b , khi ta c:
2 2
u v u v x2 y2 a 2 b2 x a y b
x y
Du = xy ra khi . Ngoi ra c th m rng cho 3 vect, thm a m; n :
a b
2 2
u v a u v a x2 y 2 a 2 b2 m 2 n 2 x a m y b n
By gi vn l phn tch cc biu thc trong cn thnh cc tng bnh phng nh th no c
th p dng bt ng thc trn c. Ta c:
2
1. 2x2 2x 1 x2 x 2 2x 1 x 2 x 1
2 2 2
1 3 1 3
2. 2
2x 2
2
3 1 x1 x x x 3 x
4
2 2
x


2 2
1 3
2
2
3. 2x 3 1 x 1 x x 1 x x 3 x x
2

2
2

n y nu p dng bt ng thc trn ta s c:


2
1 1 2
VT x x x x 1 x x 18x 2 2
2 2
D thy 18x2 2 cha phi lun ln hn 3, vi li hy im ri ca bt ng thc n khng
phi l x 0 nn ta nh gi sai. nh gi ng hy nu ta p dng cho 2 cn sau th
im ri l x 0 , nn ta s x 2 cn ng sau trc, ta c:
2 2
2 2
1 3 1 3 2 2
x
x x x 4x 4x 4 x 4x 4 x 2
2 2 2 2

Nguyn Minh Tun Trang 127


Tm ti sng to mt mt s cch gii phng trnh v t
3
Ch : Nu ban u ta vit l x
th im ri lc ny cng khng phi l x 0 nn im
2
2 2
3 3
ri l x 0 th ta phi i x x .
2
2
Cn li xut hin con 3 ta cn phi bin i cn cn li, ta c:
2 2
2x2 2x 1 x2 x2 2x 1 x 2 x 1 x2 1 x 1 x
Vy VT x 2 1 x 3 VP
Du = xy ra khi x 0 .
3x 3
Bi 40: Gii phng trnh: 2 x 2 4x 5 x3 4 2 x
2 x
Gii
p dng bt ng thc AM GM ta c:

3x 3 2 x 1 x3 1 1 1
2 x 2 4x 5
2
2
x3 2 2 x 1
x
2 x x x

AM GM
2 2x 1 1 1
2 2x 4 4 x3 4 2 x
2 x x x

Vy VT VP . Du = xy ra khi x 1

1 1 1
Bi 41: Gii phng trnh:
2 2 2
1 x 3x 1 1 x 3x 1 x 1
Din n k2pi.net
Gii
p dng bt ng thc AM GM ta c:

1 1 4
1.
2 2
1 x 3x 1 1 x 3x 1 2 x 3x 1 x 2 3x 1
2

2. x2 3x 1 x2 3x 1 2 2x2 2 2 x2 1

2 1 x2 1 1 x2
3. 0
x2 1 1 x2 1 x2 1 x2 1 1 x2 1 x2 1 1
2

Vy VT VP . Du = xy ra khi x 0 .

Bi 42: Gii phng trnh: x 4 x 3 6x 4 2 x6 x 1 0


Bi Th Vit Vted.vn
Gii

u tin ta dng tip tuyn th s tm c nh gi l: 2 x6 x 1 5x 3 .


3
- Nu x th bt ng thc ng.
5
3 2
- Nu x th 2 x6 x 1 5x 3 x 1 4x 4 8x 3 12x 2 16x 5 0 .
5

Nguyn Minh Tun Trang 128


Tm ti sng to mt mt s cch gii phng trnh v t
D thy bt ng thc cui ng nn ta c :
2
x x 3 6x 4 2 x6 x 1 x 4 x 3 6x 4 5x 3 x 1 x 2 x 1 VP
4

Vy bi ton c gii quyt!


Bi 43: Gii phng trnh: x 9 2x 2 x8 x 1 2 x x 3 2x 2 2x 3
Nguyn Minh Tun
Gii

9 7 5
x 2x 2 2 x 2
Phng trnh ny c nghim l x 1 nn ta s c 2 nh gi:
x8 x 1 7 x 5
2 2
5
Nu x th 2 nh gi l ng.
7
5
Nu x th ta c:
7
7 5 2
1. x 8 x 1 x x 1 4x6 8x 5 12x 4 16x 3 20x 2 24x 21 0
2 2
7 5 2
2. x 9 2x 2 x x 1 4x7 8x 6 12x 5 16x 4 20x 3 24x 2 28x 17 0
2 2

5 20x 2 24x 21 0
D thy rng vi x th ng nhin 2 bt ng thc lun ng v: 2
7 24x 28x 17 0
2 2
Vy VT VP x 1 x 1 0

Du = xy ra khi x 1

Bi 44: Gii phng trnh:



2x 1 x 2x 1 2
x x 1 1 2x 1 2x 1
Gii
a x
t th phng trnh tr thnh: 2
b2 1 1 a2 b 2
b 2x 1 a a 1 1 b3
3
Ta c b : a 1 b 1 c 1 1 3 abc - chng minh bi trn.
p dng ta c:
3
1 a 1 a 1 b 1 a b
3 3 3 2

3
1 1 1 1 a 1 1 a 3

3
1 1 1 b 1 b 1 b
3 3 2

Nhn v theo v ta c:
3 3 3 3 3
8 1 a3 1 b3 1 a 2 b 1 a 1 b2
3 3 3


1 a b 1 a 1 b
2 2

8
1 a b 1 a 1 b 2 1 a b 1 b 2
2 2 2 2

3
1 a 1 b
3 3
3
a 1 b 1 3
a a 1 b 1
3 2 3

Vy bi ton c gii quyt!

Nguyn Minh Tun Trang 129


Tm ti sng to mt mt s cch gii phng trnh v t
2
28x 3 9
Bi 45: Gii phng trnh: 1 x 6 1

x 6

1 4 256
28x 3

Nguyn Minh Tun
Gii
Cch 1: Bt ng thc Holder.
a x 6
Nhn thy dng tch li ngh n bt ng thc Holder. t , phng trnh tr thnh:
b 28x 3
2
b 9
1 a 1 1 256 .
a b
p dng h qu ca bt ng thc Holder cho 4 dy s:
4
1 a 1 b 1 c 1 d 1 4 abcd
4
b 9 9 b 9 9
Ta c: 1 a 1 1 1 1 4 a 256 VP
a b b a b b
Du = xy ra khi a 3; b 9 x 3
Cch 2: Bt ng thc AM GM.
Theo AM GM ta c:
2 2 2 2
b 9 b 9 9 9
1 a 1 1 1 a b 1 2 b b 1 1
2

a b a b b

b 1 1
b

2 2 2
2 9 9 9

b 1 1

b
b b
1 9 2 b
b
10 256


Vy VT VP . Du = xy ra khi x 3 .
Cch 3: Bt ng thc Cauchy Schwarz.
b 9 Cauchy Schwarz 2 9
Ta c: 1 a 1 1
a b
b 1 1


b

256
Vy VT VP . Du = xy ra khi x 3 .
Cch 4:
Ch rng ta lun c ng thc sau:
2 2 4 2 2
b 9 3 3 b 9
1 a 1 1 256 32 4 4 b 4 4 b a 1
a b b b a b
p dng lun vo bi th ta c:
2
28x 3 9

1 x 6 1

x 6
1 4 256
28x 3

2 4 2 2
3 3 28x 3 4 9
32 8 8 28x 3 8 8 28x 3 4 x 6 1 4 0
28x 3 28x 3 x 6
28x 3
Du = xy ra khi x 3
2
1 2x
1 x1
Bi 46: Gii phng trnh: 4 4 2 2
2x x1 x1 2x

Nguyn Minh Tun


Gii

Nguyn Minh Tun Trang 130


Tm ti sng to mt mt s cch gii phng trnh v t
2
a 2 x 1 1 a b
t a 2 b2 1 , phng trnh tr thnh: 2 2
b x 1 a b b a

2
1 1 a b 1 1 a b


2 2 2 2 2 0
a b b a a b b a
a b 2ab 2 2ab 1
2 2
a b a b 1 2 a b 1 2


1 2 a b 2 a b 1 0
Do a 2 b 2 1 a b 1; 2 . Vy bt ng thc cui ng.

Du = xy ra khi a b x 3

Bi 47: Gii phng trnh: 1


2 4
x1 4 3x 1 x1 3x
x1 3x 4
x 1 3 x
Nguyn Minh Tun
Gii
Ta c:

1
2 4
x1 4 3x 1 x1 3x
x1 3x x 1 3 x
4

x1 3 x 4 x 1 3 x 2 4 x 1 4 3 x 4 x 1 3 x
2
x1 3x x1 3x
p dng bt ng thc AM GM ta c:
2
x1 3x
2 x1 3x 4
x 1 3 x
2 2
2 2
x1 3x 4 4 x 1 3 x x1 3x
2
x1 3x 2
2 4
x1 3x 4
4
x 1 3 x
Cng 2 v ta c VT VP . Du = xy ra khi x 3 .
2 2
2 1 2 1
Bi 48: Gii phng trnh: 1 3 x 1

1 x2
x2
1

17 2 2
3x
Nguyn Minh Tun
Gii
p dng bt ng thc Cauchy Schwarz ta c:
2 2
2 1 2 1
1 3x 1

1 x2
x2
1


3x
2
Cauchy Schwarz
1 1 1

2
1 3 x 1


x2
1 x 2 1


3 x

Cch 1: Phn tch nhn t.

Nguyn Minh Tun Trang 131


Tm ti sng to mt mt s cch gii phng trnh v t
2
1 1 1
2

1 3 x 1


x2

1 x 2 1
17 12 2
3 x
2
1 1


1 3 x 1
x2

1 x 2 1


34 24 2
3 x
2
1 1 3x x2
3x x2 2 34 24 2
3x x2 x2 3x
2
1 x 2 2 1 x 2 11 2 3x x2
3x x2 23 2
3x 2 4 x2 2 4 x2 3x
34 24 2
2
2 3x 2 2 2 3x 4 2




2 x2 2
2 x 2 4

3x

x2

2 3 2

8 3x 8 x2 x2 3x

34 24 2
2
3x x2
Bi ton s c gii quyt nu ta chng minh c: 2 3 2 34 24 2
x2 3x
Tht khng may l:
2
3x x2 3x
AM GM
x2 2

x2

3x
2 23 2 43 2 34 24 2 VP
x2 3x
Vy bi ton c gii quyt!

5
Du = xy ra khi x
2
Cch 2: Bt ng thc.
2
a 3 x 1 1 1
t a b 1 , ta i chng minh 1 a 1 1 b 1 VP
2 2

b x 2 2 b a
2
1 1
1 a 1 b 1 b 1 a

2
2 1 1 1 1
a b b a 2
2 2 a b
2
1 1 a b 2 2
2 2
a b b a ab b a

Nguyn Minh Tun Trang 132


Tm ti sng to mt mt s cch gii phng trnh v t
2
b2 a2
3a 3 3b 3

AM GM
4 4 a b 2
ab b a



2
2
a 2
b 2
6 ab 3 2
AM GM
4 4 6 6

ab
4 6 6
16 ab
2
6 6 a2 b2
2 2 17 12 2


16
2
Vy bi ton c gii quyt!
4

Bi 49: Gii phng trnh:


1

1

2 x 2x 64
0
4
x 2x
x 2x
Nguyn Minh Tun
Gii
4

1
Ta c:
1

2 x 2x 64
0
1

1
2
64
0
4 2 2 4
x 2x
x 2x
x 2x x 2x
2 2
32 x 2 x
1 1 4
2
2 2 2 4
x 2 x x 2x x 2x
1 1 4
Li c: 2
2
2 2
x x 2 x
2x
2
2 2
x 2x
2 2
4 x 2 x
2 2
2 2 2 2 2 2
x 2 x x 2 x x 2 x x 2x

Theo bt ng thc Cauchy Schwarz ta c:
2
2 2
2x

x
x 2 x 4
2 2

2 2 2
2 2 2 2 2
x 2 x x 2 x x 2 x x

2x

2
x 2 2x 2 2 x 2x
x 2x
4

2 2 2

2 2 2 2 2
x 2x 2
2 x
2x
x 2x x 2x

Ta s chng minh:

Nguyn Minh Tun Trang 133


Tm ti sng to mt mt s cch gii phng trnh v t
2 2
32 x 2 x
4
x 2x

2 2 2 2 4
2 x 2x x 2x
x 2 x
8 2 2 2 2 2

x 2x 64 x 2 x x 2 x
4 2 2 4
x 2x 8 x 2 x x 2 x x 2x 0

Vy VT VP . Du = xy ra khi x 1
Bi 50: Gii phng trnh:

4x 7 2 8x 7 4x 5 2 x 1 2 4 x 1 1 2 8x 7 1
Nguyn Minh Tun
Gii
Ta c:

4x 7 2 8x 7 4x 5 2 x 1 2 4 x 1 1 2 8x 7 1
8x 7 3 8x 7
2 3 1 8x 7 1
x 1 x 1 2 x 1 2
4 4 4 4 2 4 2

Ch rng:
2
1 2 1 2 8x 7 3 8x 7 1
x1 2 0

x1
4
x1 x1
4
x1
4 4

2
2 2 2
8x 7 1 8x 7 1 8x 7 8x 7 3 8x 7 1
x 1 x1
4 2 4 4 4 4 4 4 2

2
8x 7 1
Do VT x 1 . M theo AM GM ta lun c:
4 2

2
8x 7
2 x 1 2 1 2
1 8x 7 1 4 8x 7 1
2 x 1 2 x1 VT
2 4 2 4
4 2

8x 7 3
Du = xy ra khi x 1 .
4 4

1 x x 1 x
Bi 51: Gii phng trnh: 3
3
2x 1 2x 1 2x 1
2x 1
x 2x 1 x

Nguyn Minh Tun


Gii
p dng bt ng thc AM GM ta c:

Nguyn Minh Tun Trang 134


Tm ti sng to mt mt s cch gii phng trnh v t
3

1
11
3
;
x 11
3 x
3 3
x x 2x 1 2x 1

3 1 x
2x 1 1 1 3 2x 1;
x

2x 1
2x 1 3

1 x 1 x
Cng li ta c: VT 6 3 2x 1 2x 1 VP
x 2x 1 x 2x 1
Vy du = xy ra khi x 2x 1 1 x 1
Bi 52: Gii phng trnh: 3 x2 x 1 1 x 3 x 2 x 1 x 1 x2 2
Din n k2pi.net.vn
Gii
p dng bt ng thc AM GM pht mt l ra. Ta c:
3 2 3 2
VT 6 x 2 x 1 1 x 6 x 2 x 1 x 1
3 x2 x 1 2 1 x 1 3 x2 x 1 2 x 1 1
x2 2
6 6

Du = xy ra khi x 0 .
4
2
Bi 52: Gii phng trnh: 3 2x 3 2 1
x 2 4x 1
2

x 16x 4
ngh 30/4/2014 THPT Chuyn L Qu n Vng Tu
Gii

3 2x 2 2 x 2
6 6 6 6
KX: x ; ; . Theo AM GM ta c: 2 1 . T suy ra:
2 3 3 2 3 2 2 2
x x
4 2

VP VT
x 2
4x 1 1
2
x 2 3
x 1 x6 14x5 87x 4 116x3 87x 2 14x 1
16x 4 x 16x6

Li c: x 6 14x 5 87x 4 116x 3 87x 2 14x 1


2
58 2813 2 6 6 6 6
x 4 x 2 14x 16 71x 2 x x 14x 1 0x ; ;
71 71 2 3 3 2

Vy VP VT . Du = xy ra khi x 1 .

Nguyn Minh Tun Trang 135


Tm ti sng to mt mt s cch gii phng trnh v t
TI LIU THAM KHO
1. Tuyn chon 410 h phng trnh Nguyn Minh Tun
2. T duy sng to, tm ti li gii PT HPT BPT L Vn on.
3. Tuyn tp phng trnh h phng trnh Din n Boxmath
4. Tuyn tp phng trnh h phng trnh Din n K2pi.net
5. Nhng vin kim cng trong bt ng thc ton hc Trn Phng.
6. Sng to phng trnh bt phng trnh, h phng trnh Nguyn Ti Chung
7. Phng php s dng my tnh CASIO trong gii ton phng trnh, bt phng trnh, h
phng trnh on Tr Dng, Bi Th Vit.

Nguyn Minh Tun Trang 136

You might also like